Fortgeschrittene Geometrie für Mathematikolympioniken

Werbung
Fortgeschrittene Geometrie für
Mathematikolympioniken
Yimin Ge
Version: 12. Februar 2007
Inhaltsverzeichnis
1 Basics
1.1 Grundlagenwiederholung . . . . . . . . . . . . . . . . . . . . . . . . . . . . .
1.2 In- und Ankreise . . . . . . . . . . . . . . . . . . . . . . . . . . . . . . . . .
1.3 Südpolsatz . . . . . . . . . . . . . . . . . . . . . . . . . . . . . . . . . . . . .
1
1
8
15
2 Fortgeschrittene Grundlagen
2.1 Sätze von Ceva und Menelaos
2.2 Die Potenz eines Punktes . . .
2.3 Potenzgeraden . . . . . . . . .
2.4 Simson-Geraden . . . . . . . .
2.5 Lotfußpunktdreiecke . . . . .
2.6 Schmetterlinge . . . . . . . .
2.7 Tangentenvierecke . . . . . . .
.
.
.
.
.
.
.
19
19
27
33
38
41
45
47
.
.
.
.
.
.
.
.
.
.
.
.
.
.
.
.
.
.
.
.
.
.
.
.
.
.
.
.
.
.
.
.
.
.
.
.
.
.
.
.
.
.
.
.
.
.
.
.
.
.
.
.
.
.
.
.
.
.
.
.
.
.
.
.
.
.
.
.
.
.
.
.
.
.
.
.
.
.
.
.
.
.
.
.
.
.
.
.
.
.
.
.
.
.
.
.
.
.
.
.
.
.
.
.
.
.
.
.
.
.
.
.
.
.
.
.
.
.
.
.
.
.
.
.
.
.
.
.
.
.
.
.
.
.
.
.
.
.
.
.
.
.
.
.
.
.
.
.
.
.
.
.
.
.
.
.
.
.
.
.
.
.
.
.
.
.
.
.
.
.
.
.
.
.
.
3 Mehr zu Dreiecken
3.1 Die Eulersche Gerade
3.2 Der Feuerbachkreis .
3.3 Isogonal Konjugierte
3.4 Symmedianen . . . .
3.5 Satz von Stewart . .
.
.
.
.
.
.
.
.
.
.
.
.
.
.
.
.
.
.
.
.
.
.
.
.
.
.
.
.
.
.
.
.
.
.
.
.
.
.
.
.
.
.
.
.
.
.
.
.
.
.
.
.
.
.
.
.
.
.
.
.
.
.
.
.
.
.
.
.
.
.
.
.
.
.
.
.
.
.
.
.
.
.
.
.
.
.
.
.
.
.
.
.
.
.
.
.
.
.
.
.
.
.
.
.
.
.
.
.
.
.
.
.
.
.
.
.
.
.
.
.
.
.
.
.
.
.
.
.
.
.
.
.
.
.
.
.
.
.
.
.
.
.
.
.
.
.
.
.
.
.
.
.
.
.
.
52
52
56
59
62
65
4 Transformationen
4.1 Translation . . . . .
4.2 Rotation . . . . . . .
4.3 Geradenspiegelung .
4.4 Zentrische Streckung
4.5 Drehstreckung . . . .
.
.
.
.
.
.
.
.
.
.
.
.
.
.
.
.
.
.
.
.
.
.
.
.
.
.
.
.
.
.
.
.
.
.
.
.
.
.
.
.
.
.
.
.
.
.
.
.
.
.
.
.
.
.
.
.
.
.
.
.
.
.
.
.
.
.
.
.
.
.
.
.
.
.
.
.
.
.
.
.
.
.
.
.
.
.
.
.
.
.
.
.
.
.
.
.
.
.
.
.
.
.
.
.
.
.
.
.
.
.
.
.
.
.
.
.
.
.
.
.
.
.
.
.
.
.
.
.
.
.
.
.
.
.
.
.
.
.
.
.
.
.
.
.
.
.
.
.
.
.
.
.
.
.
.
67
67
70
72
75
78
5 Inversive Geometrie
5.1 Inversion . . . . . . . . . . . . . . . . . . . . . . . . . . . . . . . . . . . . . .
5.2 Eigenschaften der Inversion . . . . . . . . . . . . . . . . . . . . . . . . . . .
5.3 Anwendungen der Inversion . . . . . . . . . . . . . . . . . . . . . . . . . . .
83
83
84
88
i
6 Geometrische Ungleichungen
6.1 Dreiecksungleichung . . . .
6.2 Algebraische Techniken . . .
6.3 Jensen-Ungleichung . . . . .
6.4 Satz von Ptolemäus . . . . .
.
.
.
.
.
.
.
.
.
.
.
.
.
.
.
.
.
.
.
.
.
.
.
.
.
.
.
.
.
.
.
.
.
.
.
.
.
.
.
.
.
.
.
.
.
.
.
.
.
.
.
.
.
.
.
.
.
.
.
.
.
.
.
.
.
.
.
.
.
.
.
.
.
.
.
.
.
.
.
.
.
.
.
.
.
.
.
.
.
.
.
.
.
.
.
.
.
.
.
.
.
.
.
.
.
.
.
.
94
94
96
101
104
A Orientierte Winkel modulo 180◦
108
A.1 Definition und Eigenschaften von Kreiswinkeln . . . . . . . . . . . . . . . . . 108
A.2 Anwendungen von Kreiswinkeln . . . . . . . . . . . . . . . . . . . . . . . . . 112
B Beispiele, Beispiele, Beispiele, . . .
116
ii
Vorwort
Dieses Skriptum ist eine Sammlung geometrischer Erfahrung, die ich in drei Jahren Mathematikolympiade, darunter zwei Trainingskurse in Raach und zwei IMOs (Internationale
Mathematikolympiade), gesammelt habe.
Der Schwerpunkt dieses Skriptums liegt bei Aufgaben von vergangenen IMOs und IMOShortlists sowie synthetischen Techniken, um diese zu meistern. Es werden gewisse Vertrautheiten mit den grundlegenden Techniken und Sätzen der ebenen Euklidischen Geometrie
vorausgesetzt, welche am Anfang in Form von gelösten Beispielen wiederholt werden.
In diesem Skriptum werden einige Techniken sowohl theoretisch besprochen, als auch in
Form von gelösten Beispielen demonstriert. Das Wichtigste, um erfolgreich bei mathematischen Wettbewerben abzuschneiden, ist meines Erachtens nach allerdings das selbstständige
Lösen von Bespielen. Zu diesem Zweck befindet sich in Anhang B ab Seite 116 eine kleine
Sammlung von Übungsaufgaben, wem diese nicht genug sind, dem möchte ich die Seiten
www.mathlinks.ro und www.kalva.demon.co.uk sehr ans Herz legen. Des Weiteren sind
einige gute Einführungen in die Geometrie im Literaturverzeichnis auf Seite 125 angegeben.
[1] ist sicherlich ein Standardwerk in diesem Gebiet, [2] und [3] sind ebenfalls sehr gute und
sehr umfangreiche Einführungen, die kostenlos als Ebooks verfügbar sind.
Zur Notation sei noch angemerkt, dass ich mich nicht immer an die in der Literatur oder
in der Schule üblichen Konventionen halte. Beispielsweise schreibe ich aus Bequemlichkeitsgründen für die Länge einer Strecke AB üblicherweise bloß AB, anstatt, wie sonst üblich,
AB. Mit [X] ist, wenn nicht anders angemerkt, der Flächeninhalt von X gemeint.
Bevor es nun wirklich geometrisch wird, möchte ich noch all jenen Personen danken,
die direkt oder indirekt zur Entstehung dieses Skriptums beigetragen haben, insbesondere
also auch jenen Personen, denen ich überhaupt erst meine Begeisterung für die Mathematikolympiade (und damit auch für die Mathematik selbst) zu verdanken habe. In diesem Sinne
seien der wissenschaftliche Leiter der Österreichischen Mathematikolympiade, Gerd Baron,
sowie meine bisherigen Kursleiter, Karl Czakler, Gerhard Pillwein und Richard Henner, dankend erwähnt. Weiters möchte ich allen Vortragenden des Vorbereitungstrainings in Raach
während der Jahre 2005 und 2006 danken, insbesondere Erich und Gerhard Windischbacher
sowie Gottfried Perz, den Vortragenden der Geometrie in diesen Jahren. Ich bedanke mich
weiters bei Michael Mosshammer für die Unterstützung und die hilfreichen Vorschläge. Für
die zahlreichen Kommentare und Verbesserungsvorschläge zu einer früheren Version dieses
Skriptums, die für das Entstehen dieser Version wesentlich waren, möchte ich mich herzlichst
bei Gerhard Kirchner bedanken.
Wien, Februar 2007
Yimin Ge
iii
Kapitel 1
Basics
In diesem Kapitel sollen die wichtigsten Grundlagen der Olympiadengeometrie, mit deren
Hilfe bereits eine Vielzahl an Aufgaben gelöst werden können, behandelt werden.
1.1
Grundlagenwiederholung
Zunächst sollen die wichtigsten Sätze und Techniken wie Peripheriewinkelsatz, Strahlensatz,
Winkeljagd, ähnliche Dreiecke u.s.w. anhand einiger Beispiele wiederholt werden.
Beispiel 1.1.1. Es seien k1 und k2 zwei Kreise mit den Mittelpunkten A und B, die sich in
C und D schneiden. Der Umkreis k des Dreiecks ABC schneide k1 in E und k2 in F , wobei
_
E und F von C verschieden sind und der Bogen EF, der C nicht enthält, außerhalb von k1
_
und k2 liegt. Zeige, dass der Bogen EF, der C nicht enthält, durch CD halbiert wird.
C
A
B
D
F
E
Abbildung 1.1: Zeichnung zu Beispiel 1.1.1
1
Lösung. Am selben Kreis sind zwei Kreisbögen genau dann gleich groß, wenn die Peripheriewinkel über diesen gleich groß ist. Es genügt daher, zu zeigen, dass CD die Winkelsymmetrale
von ∠ECF ist.
Nach dem Peripheriewinkelsatz über AC an k gilt
∠AF C = ∠ABC.
Nach dem Peripheriewinkel-Zentriwinkelsatz über CD an k2 gilt
∠DF C =
∠DBC
= ∠ABC.
2
Somit gilt ∠AF C = ∠DF C, womit die Punkte A, D, F auf einer Geraden liegen. Analog
liegen auch B, D, E auf einer Geraden. Somit gilt
∠DCE =
∠F AE
∠F BE
∠F BD
∠DAE
=
=
=
= ∠F CD,
2
2
2
2
also ist CD die Winkelsymmetrale von ∠ECF .
Beispiel 1.1.2 (Mediterranean MC 2005). Seien k und k 0 zwei konzentrische Kreise mit
Mittelpunkt O und den Radien R bzw. R0 , wobei R < R0 . Eine Gerade durch O schneidet k
in A und k 0 in B, wobei O bzwischen A und B liegt. Eine weitere Gerade durch O, die von
AB verschieden ist, schneidet k in E und k 0 in F , wobei E zwischen O und F liegt. Zeige,
dass die Umkreise der Dreiecke OAE und OBF , der Kreis mit Durchmesser EF und der
Kreis mit Durchmesser AB alle durch einen gemeinsamen Punkt gehen.
F
T
k'
E
k
O
B
A
Abbildung 1.2: Zeichnung zu Beispiel 1.1.2 (Mediterranean MC 2005)
2
Lösung. Sei T der Schnittpunkt der Umkreise von 4OAE und 4OBF (verschieden von O).
Sei ∠AOE = α und ∠BOF = β. Es gilt α + β = 180◦ . Die Dreiecke OBF und OAE sind
gleichschenklig, somit gilt
α
∠OEA = ∠OAE = 90◦ −
2
und
β
∠OF B = ∠OBF = 90◦ − .
2
BOT F ist ein Sehnenverieck, somit gilt
∠OT F = 180◦ − ∠OBF = 90◦ +
β
.
2
AOET ist ebenfalls ein Sehnenviereck, somit gilt
∠OT E = ∠OAE = 90◦ −
α
.
2
Somit gilt
α
β
− 90◦ + = 90◦
2
2
und daher liegt T auf dem Thaleskreis mit Durchmesser EF .
Auf ähnliche Weise zeigt man, dass ∠AT B = 90◦ . BOT F ist ein Sehnenviereck, somit
gilt
β
∠OT B = ∠OF B = 90◦ − .
2
AOET ist ein Sehnenviereck, somit gilt
∠ET F = ∠OT F − ∠OT E = 90◦ +
∠OT A = ∠OEA = 90◦ −
α
.
2
Somit gilt
α
β
+ 90◦ − = 90◦ .
2
2
und daher liegt T auf dem Thaleskreis mit Durchmesser AB.
∠AT B = ∠OT A + ∠OT B = 90◦ −
Beispiel 1.1.3 (Türkei 1994). Sei ABCD ein Sehnenviereck mit ∠BAD < 90◦ und ∠BCA =
∠DCA. Sei E auf der Strecke AD so gewählt, dass BD = 2 · DE gilt. Die zu CD parallele
Gerade durch E schneidet die Diagonale AC in F . Zeige, dass AC · BD = 2 · AB · F C gilt.
Lösung. Wegen ∠BCA = ∠DCA gilt AB = AD, denn zu gleich großen Peripheriewinkeln
gehören am selben Kreis gleich lange Sehnen. Nach dem Strahlensatz gilt wegen EF k DC
somit
FC
AC
AC
=
=
.
ED
AD
AB
Somit gilt
AC · BD
F C · AB = AC · DE =
.
2
3
D
C
E
F
B
A
Abbildung 1.3: Zeichnung zu Beispiel 1.1.3 (Türkei 1994)
Beispiel 1.1.4 (Asian Pacific MO 1998). Sei ABC ein Dreieck und D der Fußpunkt der
Höhe durch A auf BC. Seien E und F Punkte auf einer Geraden durch D, sodass AE ⊥ BE
und AF ⊥ CF , E und F verschieden von D. Seien M und N die Mittelpunkte der Strecken
BC und EF . Man zeige, dass AN auf N M normal steht.
E
A
N
F
D
B
M
C
Abbildung 1.4: Zeichnung zu Beispiel 1.1.4 (Asian Pacific MO 1998)
Lösung. Da AEBD wegen ∠AEB = ∠ADB = 90◦ ein Sehnenviereck ist, gilt ∠AED =
∠ABD = β. AF DC ist ebenfalls ein Sehenviereck, daher gilt ∠AF E = ∠ACD = γ. Somit
sind die Dreiecke ABC und AEF ähnlich. Da M bzw. N aber die Strecken BC bzw. EF
halbieren, sind auch die Dreiecke ABM und AEN ähnlich. Wir erhalten daher
AN
AE
=
AM
AB
sowie ∠BAM = ∠EAN , also auch ∠N AM = ∠EAB. Somit sind die Dreiecke AM N und
ABE ähnlich, womit ∠AN M = ∠AEB = 90◦ gilt.
4
Beispiel 1.1.5 (IMO 1979). Zwei in einer Ebene liegende Kreise k1 und k2 schneiden einander in den Punkten A und B. Zwei Punkte P1 und P2 bewegen sich auf k1 und k2 mit
konstanten Geschwindigkeiten und im gleichen Drehsinn. Sie beginnen gleichzeitig in A und
treffen nach einem Umlauf wieder gleichzeitig in A ein. Ermittle die Ortslinie des Halbierungspunktes H von P1 P2 und zeige, dass die Streckensymmetrale von P1 P2 stets durch einen
fixen Punkt P geht.
A
O1
O2
M
P1
H
B
P2
Abbildung 1.5: Zeichnung zu Beispiel 1.1.5 (IMO 1979)
Lösung. Es seien O1 und O2 die Mittelpunkte von k1 und k2 und M der Halbierungspunkt von
O1 O2 . Da ∠AO1 P1 = ∠AO2 P2 wegen der konstanten Geschwindigkeiten gilt, sind die Punkte
P1 , B und P2 kollinear, da die Winkel ∠ABP1 und ∠ABP2 wegen der gleichen Zentriwinkel
über AP1 bzw AP2 supplementär, falls B zwischen P1 und P2 liegt, bzw ansonsten gleich
sind.
Es sei nun ∠AO1 O2 = ∠BO1 O2 = ε und ∠AO2 O1 = ∠BO2 O1 = ϕ. Aus dem Peripheriewinkelsatz über AB an k1 und k2 folgt
∠AP1 B = ε,
∠AP2 B = ϕ.
Somit sind die Dreiecke AP1 P2 und AO1 O2 ähnlich.
Da H und M die Strecken P1 P2 und O1 O2 halbieren, sind auch die Dreiecke AM O2 und
AHP2 ähnlich.
Somit gilt
∠AM O2 = ∠AHP2 .
5
Da aber ∠AM O2 = ∠BM O2 , gilt somit
∠AM B = 2 · ∠AHB.
Da M aber auf der Streckensymmetrale von AB liegt, folgt somit aus dem Peripheriewinkelsatz, dass M der Umkreismittelpunkt des Dreiecks ABH ist. Da A, B und M aber fix
sind, ist die Ortslinie von H somit der Kreis mit Mittelpunkt M und Radius M A. Nach dem
Satz von Thales geht die Streckensymmetrale von P1 P2 somit immer durch den gespiegelten
Punkt von B an M .
Beispiel 1.1.6 (IMO 1990). Die Sehnen AB und CD eines Kreises schneiden sich innerhalb
des Kreises im Punkt E. M sei ein beliebiger Punkt auf der Strecke AB, der verschieden von
E ist. Die Tangente in E an den Umkreis von 4DEM schneided AC in G und BC in F .
Gegeben sei nun AM/AB = t. Man drücke EG/EF in Abhängigkeit von t aus.
B
F
D
M
E
G
C
A
Abbildung 1.6: Zeichnung zu Beispiel 1.1.6 (IMO 1990)
Lösung. Da ein Peripheriewinkel auch an der Tangente zu sehen ist, gilt
∠DM E = ∠DEG = ∠CEF.
Nach dem Peripheriewinkelsatz über BD gilt
∠ECF = ∠DCB = ∠DAB = ∠DAM.
Somit sind die Dreiecke ADM und CF E ähnlich und daher gilt
DM
AM
=
.
CE
FE
6
(1.1)
Weiters gilt nach dem Peripheriewinkelsatz über AD
∠DBM = ∠DBA = ∠DCA = ∠ECG.
Weiters gilt wegen ∠DEG = ∠DM E auch
∠CEG = ∠BM D.
Somit sind die Dreiecke BDM und CGE ähnlich und es gilt
BM
DM
=
.
CE
GE
(1.2)
Dividieren wir (1.1) durch (1.2), so erhalten wir
EG
AM
t
=
=
.
EF
BM
1−t
Beispiel 1.1.7 (IMO 1997). Es sei ∠BAC der kleinste Winkel im Dreieck ABC. Die Punkte
B und C teilen den Umkreis des Dreiecks ABC in zwei Bögen. Sei U ein Punkt auf dem
_
Bogen BC, der A nicht enthält. Die Streckensymmetralen von AB bzw AC schneiden AU
in den Punkten V bzw W . Die Geraden BV und CW schneiden sich in T . Zeige, dass
AU = T B + T C gilt.
Y
A
W
X
T
V
B
C
U
Abbildung 1.7: Zeichnung zu Beispiel 1.1.7 (IMO 1997)
Lösung. Die Geraden BV bzw CW schneiden den Umkreis des Dreiecks ABC ein weiteres
Mal in X bzw Y .
7
Da AU und CY symmetrisch bezüglich der Streckensymmetrale von AC liegen, gilt
AU = CY . Wegen CY = T Y + T C genügt es daher zu zeigen, dass
TB = TY
gilt.
Aufgrund der Symmetrie bezüglich der Streckensymmetrale von AB gilt
∠ABT = ∠ABV = ∠BAV.
Aufgrund der Symmetrie bezüglich der Streckensymmetrale von AC und aufgrund des Peripheriewinkelsatzes über Y A gilt
∠Y BA = ∠Y CA = ∠W CA = ∠W AC.
Somit gilt
∠Y BT = ∠Y BA + ∠ABT
= ∠W AC + ∠BAV = ∠W AC + ∠BAW = ∠BAC.
Andererseits gilt nach dem Peripheriewinkelsatz über BC
∠BAC = ∠BY C = ∠BY T.
Somit gilt ∠Y BT = ∠BY T , womit das Dreieck T BY gleichschenklig ist. Somit gilt, wie zu
zeigen war, T B = T Y .
1.2
In- und Ankreise
Aus dem Schulunterricht ist bekannt, dass sich die Innenwinkelsymmetralen eines Dreiecks
ABC im Inkreismittelpunkt schneiden. Dieser Punkt, den wir mit I bezeichnen, hat von allen
Dreiecksseiten den gleichen Normalabstand. Er ist daher der Mittelpunkt eines Kreises, des
sogenannten Inkreises, der alle Dreiecksseiten von innen berührt.
Bevor wir uns genauer mit dem Inkreis auseinandersetzen, sei noch eine wichtige Eigenschaft der Innenwinkelsymmetralen erwähnt.
Satz 1.1. In einem Dreieck teilt jede Innenwinkelsymmetrale die gegenüberliegende Seite im
Verhältnis der anliegenden Seiten.
Anders ausgedrückt: Sei L der Schnittpunkt der Innenwinkelsymmetrale von ∠CAB mit
der Seite BC. Dann gilt
AB
BL
=
.
LC
AC
8
A
B
C
L
Abbildung 1.8
Beweis. Wenden wir den Sinussatz auf die Dreiecke ABL und ALC (deren supplementäre
Winkel bei L gleiche Sinuswerte haben) an, so erhalten wir
AB
BL
=
∠A
sin ∠L
sin 2
und
LC
AC
=
,
∠A
sin ∠L
sin 2
insgesamt also
BL
AB
c
=
= .
LC
AC
b
Damit zeigt man leicht die Existenz des Inkreismittelpunktes (d.h. dass sich die Innenwinkelsymmetralen auch wirklich in einem Punkt schneiden), man benötigt lediglich den
Satz von Ceva (siehe Kapitel 2.1).
Es sei nun ABC ein Dreieck mit Inkreismittelpunkt I. Die Inkreisberührpunkte auf BC,
CA, AB seien X, Y , Z (siehe Abbildung 1.9).
Da zwei Tangenten an einen Kreis von einem beliebigen äußeren Punkt immer gleich lang
sind, gilt AY = AZ =: x, BZ = BX =: y, CX = CY =: z. Wir erhalten also
y + z = a,
z + x = b,
x + y = c.
Durch Addition erhalten wir somit (s sei der halbe Dreiecksumfang):
2x + 2y + 2z = a + b + c = 2s
und somit
x + y + z = s.
Daraus folgt sofort
Satz 1.2.
x = s − a,
y = s − b,
9
z = s − c.
A
x
x
Z
Y
I
y
z
B
y
X
z
C
Abbildung 1.9: Inkreis und Inkreisberührpunkte
Das Dreieck IBC hat zur Seite a die Höhe r (r sei der Inkreisradius des Dreiecks ABC),
daher gilt [IBC] = ar/2. Summieren wir zyklisch auf, so erhalten wir
[ABC] = [IBC] + [ICA] + [IAB] = (a + b + c)r/2 = s · r.
Wir erhalten daher eine Formel für den Inkreisradius:
Satz 1.3.
r=
[ABC]
.
s
Jeder Winkel eines Dreiecks hat allerdings nicht nur eine Innenwinkelsymmetrale, sondern auch eine Außenwinkelsymmetrale, welche den Außenwinkel, also den zum Innenwinkel
supplementären Winkel, halbiert. Diese steht, wie man leicht sieht, normal auf die Innenwinkelsymmetrale.
Es sei nun Ia Ib Ic das Dreieck, das aus den Außenwinkelsymmetralen des Dreiecks ABC
gebildet wird (Ia liegt A gegenüber, Ib liegt B gegenüber und Ic liegt C gegenüber).
Jeder Punkt auf der Geraden Ic Ia ist von AB und BC gleich weit entfernt. Auch ist jeder
Punkt auf der Geraden Ia Ib von BC und CA gleich weit entfernt. Somit ist der Schnittpunkt
der Geraden Ic Ia und Ia Ib , also Ia , von allen Dreiecksseiten gleich weit entfernt. Da aber Ia
von AB und CA gleich weit entfernt ist und offensichtlich nicht auf Ib Ic liegt, muss Ia
daher auf der Innenwinkelsymmetrale durch A liegen. Analoges Vorgehen bei den anderen
Dreiecksseiten liefert daher
Satz 1.4. In einem Dreieck schneiden sich die Außenwinkelsymmetralen von zwei beliebigen
Winkeln und die Innenwinkelsymmetrale des dritten Winkels in einem gemeinsamen Punkt.
10
Yc
Ic
Zb
A
Ib
Yb
Zc
I
Xb
Xc
B
C
Xa
Za
Ya
Ia
Abbildung 1.10: Ankreise
Diese Punkte Ia , Ib , Ic heißen Ankreismittelpunkte des Dreiecks ABC. Sie sind die Mittelpunkte jener Kreise, der sogenannten Ankreise des Dreiecks ABC, die alle Dreiecksseiten
von außen berühren, je zwei Seiten in der Verlängerung und eine als Strecke.
Es seien nun die Ankreisberührpunkte wie in Abbildung 1.10 benannt.
Da zwei Tangenten an einem Kreis von einem beliebigen äußeren Punkt gleich lang sind,
gilt
AYa = AZa
sowie
AYa + AZa = AB + BZa + Ya C + CA
= AB + BXa + Xa C + CA
=a+b+c
= 2s.
Analog erhalten wir auch
AYa = AZa = BZb = BXb = CXc = CYc = s.
Es gilt ebenfalls CXb = BXb − BC = s − a. Durch analoge Überlegungen erhalten wir
11
(1.3)
Satz 1.5.
BXc = BZc = CXb = CYb = s − a,
CYa = CXa = AYc = AZc = s − b,
AZb = AYb = BZa = BXa = s − c.
Man kann (1.3) aber auch einfacher und leichter verständlich in Worte fassen:
Satz 1.6. In einem Dreieck ist der Abstand von einem Eckpunkt zu den Ankreisberührpunkten
des gegenüberliegenden Ankreises an den anliegenden Seiten gleich dem halben Umfang.
Übrigens lässt sich Satz 1.1 auch leicht auf Außenwinkelsymmetralen übertragen:
Satz 1.7. Sei ABC ein Dreieck mit AB 6= AC und sei L der Schnittpunkt der Außenwinkelsymmetrale von ∠CAB mit BC. Dann gilt
BL
AB
=
.
LC
AC
Der Beweis dieses Satzes verläuft analog zum Beweis von Satz 1.1.
Zusammen mit dem Satz von Ceva (siehe Kapitel 2.1) liefert dies einen weiteren Beweis
für Satz 1.4.
Beispiel 1.2.1. Zeige, dass die 4 Normalprojektionen des Eckpunktes A des Dreiecks ABC
auf die Innen- und Außenwinkelhalbierenden von ∠ABC und ∠ACB auf einer Geraden
liegen.
R
C
P
S
I
A
B
T
U
Abbildung 1.11: Zeichnung zu Beispiel 1.2.1
Lösung. Es seien S und T die Normalprojektionen von A auf die Innenwinkelsymmetralen
und U bzw. R jene auf die Außenwinkelsymmetrale von ∠ABC und ∠BCA. Es sei weiters
12
I der Inkreismittelpunkt des Dreiecks ABC und P der Schnittpunkt der Außenwinkelsymmetralen von ∠ABC und ∠BCA. P ist sicherlich ein Ankreismittelpunkt von 4ABC und
liegt daher auf der Geraden AI (I sei der Inkreismittelpunkt von 4ABC.).
Das Viereck IBP C ist ein Sehnenviereck, denn es hat zwei gegenüberliegende rechte
Winkel. Somit gilt
γ
∠IP B = ∠ICB = .
2
Das Viereck ARP U ist aufgrund zwei gegenüberliegender rechter Winkel ebenfalls ein Sehnenviereck. Somit gilt
γ
∠ARU = ∠AP U = ∠IP B = .
2
Das Viereck ARCT ist aber sicherlich ein Rechteck, denn es hat vier rechte Winkel, daher
gilt
γ
∠ART = ∠ACT = = ∠ARU.
2
Somit sind R, T , U kollinear. Analog sind auch R, S, U kollinear. Insgesamt sind also R, S,
T , U kollinear.
Beispiel 1.2.2 (IMO Shortlist 2003). Sei ABC ein Dreieck und sei P ein Punkt im Inneren
des Dreiecks. Seien D, E, F die Fußpunkte von P auf BC, CA, AB. Es sei bekannt, dass
AP 2 + P D2 = BP 2 + P E 2 = CP 2 + P F 2 .
Seien weiters Ia , Ib , Ic die Ankreismittelpunkte des Dreiecks ABC. Zeige, dass P der Umkreismittelpunkt des Dreiecks Ia Ib Ic ist.
Ib
C
Ia
D
E
P
A
F
B
Ic
Abbildung 1.12: Zeichnung zu Beispiel 1.2.2
13
Lösung. Aus AP 2 + P D2 = BP 2 + P E 2 folgt
AP 2 − P E 2 = BP 2 − P D2 .
Nach Pythagoras gilt allerdings
AP 2 − P E 2 = AE 2 ,
BP 2 − P D2 = BD2 .
Daher gilt also AE = BD und analog auch BF = CE und CD = AF . Sei nun x = BF =
CE, y = CD = AF , z = AE = BD, a = BC, b = CA, c = AB, s = (a + b + c)/2. Wir
erhalten somit x = s − a, y = s − b, z = s − c und daher sind D, E, F die Punkte, in
denen die Ankreise des Dreiecks ABC die Dreiecksseiten berühren. Somit sind also jeweils
die Punkte P , D, Ia , die Punkte P , E, Ib sowie die Punkte P , F , Ic kollinear.
Aufgrund der Winkelsumme in den Dreiecken AEIb und AIc F folgt, dass
∠EIb A = ∠AIc F = ∠CAB/2
womit das Dreieck P Ib Ic gleichschenklig ist. Analog sind auch die Dreiecke P Ic Ia und P Ia Ib
gleichschenklig und daher gilt P Ia = P Ib = P Ic .
Beispiel 1.2.3 (IMO Shortlist 1998). Sei ABCD ein konvexes Sehnenviereck. Seien E und
F variable Punkte auf den Seiten AB und CD, sodass AE/EB = CF/F D gilt. Sei P ein
Punkt auf der Strecke EF , sodass P E/P F = AB/CD gilt. Zeige, dass das Verhältnis der
Flächen der Dreiecke AP D und BP C nicht von der Wahl von E und F abhängt.
G
D
F
C
P
A
E
B
Abbildung 1.13: Zeichnung zu Beispiel 1.2.3 (IMO Shortlist 1998)
14
Lösung. Nehmen wir zunächst an, die Geraden AD und BC sind nicht parallel und schneiden
sich in einem Punkt G. Es gilt AE/CF = BE/DF = (AE + BE)/(CF + DF ) = AB/CD.
Da aber ABCD ein Sehnenviereck ist, sind die Dreiecke GAB und GCD ähnlich und es gilt
daher AB/CD = GA/GC. Somit gilt
AB
GA
AE
=
=
CF
CD
GC
womit wegen ∠GAE = ∠GCF die Dreiecke AEG und CF G ähnlich sind. Somit gilt
GE
AE
AB
EP
=
=
=
GF
CF
CD
PF
womit GP im Dreieck EGF die Seite EF im Verhältnis der anliegenden Seiten teilt und
daher die Winkelsymmetrale von ∠EGF ist. Allerdings gilt, da die Dreiecke AEG und CF G
ähnlich sind, auch ∠AGE = ∠CGF , also ist GP auch die Winkelsymmetrale von ∠AGB.
Somit ist P von AD und BC gleich weit entfernt, d.h. die Dreiecke AP D und BP C haben
eine gleiche Höhe. Somit ist das gesuchte Verhältnis nur von den Seiten abhängig, d.h. es
gilt
AD
[AP D]
=
[BP C]
BC
und das ist ein fixes Verhältnis.
Nehmen wir nun an, dass AD und BC parallel sind. Da ABCD aber ein Sehnenviereck
ist, muss es somit ein gleichschenkliges Trapez sein und es gilt daher AB = CD und somit
auch AE = CF und EB = F D. Es folgt weiters, dass P der Mittelpunkt von EF ist. Seien
nun M und N die Mittelpunkte von AB und CD. Dann gilt M E = N F und sicherlich sind
E und F von M N gleich weit entfernt. Somit liegt P , also der Mittelpunkt von EF , auf
M N , also ist P von AD und BC gleich weit entfernt, womit wir wiederum
AD
[AP D]
=
[BP C]
BC
erhalten.
1.3
Südpolsatz
Satz 1.8 (Südpolsatz). In jedem Dreieck ABC schneiden sich die Winkelsymmetrale von
∠CAB und die Streckensymmetrale von BC auf dem Umkreis von 4ABC.
Beweis. Sei A1 der Schnittpunkt der Streckensymmetrale von BC mit dem Umkreis von
4ABC. Sicherlich gilt A1 B = A1 C. Zu gleich langen Sehnen gehören am selben Kreis gleich
große Peripheriewinkel, daher gilt ∠BAA1 = ∠CAA1 , womit A1 auf der Winkelsymmetrale
von ∠CAB liegt.
15
A
I
B
C
A1
Abbildung 1.14: Südpolsatz
Seien nun A1 , B1 und C1 jeweils die Schnittpunkte der Streckensymmetralen von BC,
CA und AB mit den Winkelsymmetralen von ∠CAB, ∠ABC und ∠BCA. Dann heißen
die Punkte A1 , B1 und C1 , welche, wie wir soeben bewiesen haben, auf dem Umkreis von
4ABC liegen, auch Südpole von A, B und C in Bezug zum Dreieck ABC.
Diese Südpole haben noch einige andere nützliche Eigenschaften:
Satz 1.9. Sei I der Inkreismittelpunkt von 4ABC. Dann ist A1 der Umkreismittelpunkt des
Dreiecks BCI, B1 der Umkreismittelpunkt des Dreiecks CAI und C1 der Umkreismittelpunkt
des Dreiecks ABI.
Beweis. Sei α = ∠CAB, β = ∠ABC, γ = ∠BCA. Der Winkel ∠A1 IB ist im Dreieck AIB
ein Außenwinkel, daher gilt ∠A1 IB = ∠IAB + ∠ABI = α/2 + β/2, denn in einem Dreieck
ist die Summe zweier Winkel gleich dem Außenwinkel des dritten Winkels. Andererseits gilt
∠A1 BI = ∠A1 BC + ∠CBI = ∠A1 AC + ∠CBI = α/2 + β/2. Somit gilt ∠A1 IB = ∠A1 BI,
also ist das Dreieck A1 IB gleichschenklig. Analog ist auch das Dreieck A1 IC gleichschenklig.
Somit gilt A1 B = A1 I = A1 C, also ist A1 der Umkreismittelpunkt von 4BIC. Analog sind
auch B1 bzw C1 die Umkreismittelpunkte von 4CAI bzw 4ABI.
Beispiel 1.3.1 (IMO 2004). Sei ABC ein spitzwinkliges Dreieck mit AB 6= AC. Der Kreis
mit Durchmesser BC schneidet die Seiten AB und AC in M und N . Sei O der Mittelpunkt
der Seite BC. Die Winkelhalbierenden der Winkel ∠BAC und ∠M ON schneiden sich in R.
Zeige, dass die Umkreise der Dreiecke BM R und CN R einen gemeinsamen Punkt auf der
Seite BC haben.
Lösung. Sei L der zweite Schnittpunkt der Umkreise der Dreiecke BM R und CN R. Wir
wollen zeigen, dass ∠BLC = ∠BLR + ∠CLR = 180◦ gilt.
Da BLRM ein Sehnenviereck ist, gilt
∠BLR = 180◦ − ∠BM R = ∠AM R.
16
A
M
N
R
B
O
L
C
Abbildung 1.15: Zeichnung zu Beispiel 1.3.1 (IMO 2004)
Analog gilt
∠CLR = ∠AN R.
Es bleibt somit zu zeigen, dass ∠AM R + ∠AN R = 180◦ . Dies ist genau dann der Fall,
wenn AM RN ein Sehnenviereck ist.
Da O der Umkreismittelpunkt von BCN M ist, ist 4M ON gleichschenklig und daher
ist die Winkelsymmetrale von ∠M ON auch gleichzeitig die Streckensymmetrale von M N .
Da sich aber in jedem Dreieck die Winkelsymmetrale und die Streckensymmetrale der gegenüberliegenden Seite auf dem Umkreis schneiden (Südpolsatz), muss AM RN ein Sehnenviereck sein.
Beispiel 1.3.2 (IMO 2006). Es sei ABC ein Dreieck mit dem Inkreismittelpunkt I. Für
einen Punkt P im Innern des Dreiecks gelte:
∠P BA + ∠P CA = ∠P BC + ∠P CB.
Man beweise die Ungleichung AP ≥ AI mit Gleichheit genau dann, wenn P = I gilt.
Lösung. Betrachten wir die Winkelsumme im Dreieck P BC, so gilt
∠BP C = 180◦ − ∠P BC − ∠P CB
∠P BC + ∠P CB + ∠P BA + ∠P CA
= 180◦ −
2
β+γ
= 180◦ −
= ∠BIC,
2
17
A
I
P
B
C
A1
Abbildung 1.16: Zeichnung zu Beispiel 1.3.2 (IMO 2006)
womit P auf dem Umkreis von 4BIC liegt. Sei nun A1 der Südpol von A in Bezug zum
Dreieck ABC. Dieser liegt auf der Winkelsymmetrale von ∠CAB, also AI. Andererseits
ist A1 aber der Mittelpunkt des Kreises durch B, C, P und I. Die Verlängerung der
kürzestmöglichen Strecke von einem fixen Punkt (A) außerhalb eines Kreises zu einem Punkt
auf dem Kreis verläuft aber durch den Kreismittelpunkt, woraus die Behauptung folgt.
18
Kapitel 2
Fortgeschrittene Grundlagen
Nun wollen wir einen Schritt in eine größere Welt machen und fortgeschrittene, aber wichtige
Themen behandeln.
2.1
Sätze von Ceva und Menelaos
Wir beginnen mit einem wichtigen Satz, dem Satz von Ceva. Dieser bildet eine wichtige
Grundlage für die Dreiecksgeometrie, ist aber auch oft bei Aufgaben essenziell. Doch zunächst
müssen wir den Begriff der gerichteten Länge erklären.
Üblicherweise ist die Länge einer Strecke vorzeichenlos, d.h. es gilt AB = BA. Die Idee
ist nun, eine Streckenlänge mit einem Vorzeichen zu versehen, sodass AB = −BA gilt. Die
Verwendung von gerichteten Längen ist nur dann sinnvoll, wenn mehrere Strecken im Spiel
sind, die auf einer Gerade liegen oder zumindest parallel sind.
Wir definieren somit das gerichtete Produkt oder den gerichteten Quotienten von zwei
gericheten Längen, die auf einer Gerade liegen oder parallel sind, als positiv bzw. negativ,
jenachdem, ob sie dieselbe oder entgegengesetzte Orientierung besitzen.
Satz 2.1 (Satz von Ceva). Sei ABC ein Dreieck und seien X, Y , Z Punkte auf den Geraden
BC, CA und AB. Dann gehen die Geraden AX, BY und CZ dann und nur dann durch
einen Punkt, wenn
BX CY AZ
·
·
= 1,
(2.1)
XC Y A ZB
wobei die Streckenverhältnisse gerichtet zu nehmen sind.
Beweis. Nehmen wir zunächst an, dass die Geraden AX, BY , CZ durch einen gemeinsamen Punkt T gehen. Dann ist das Verhältnis BX/XC nach dem Strahlensatz gleich dem
Verhältnis der Normalabstände von B und C zur Gerade AT . Letzteres Verhältnis ist aber
gleich dem Flächenverhältnis [AT B]/[CT A], denn diese Dreiecke haben die Seite AT gemeinsam. Durch analoge Überlegungen für die anderen Verhältnisse erhalten wir
[AT B] [BT C] [CT A]
BX CY AZ
·
·
=
·
·
= 1.
XC Y A ZB
[CT A] [AT B] [BT C]
19
C
X
Y
T
A
B
Z
Abbildung 2.1: Satz von Ceva
Nehmen wir nun umgekehrt an, die Gleichung (2.1) sei erfüllt. Die Geraden AX und BY
schneiden sich im Punkt T und CT schneide AB in Z 0 . Nach Ceva in die andere Richtung
(die wir soeben bewiesen haben), gilt
BX CY AZ 0
·
·
= 1.
XC Y A Z 0 B
Zusammen mit (2.1) gilt somit
AZ
ZB
AZ + ZB
⇔
ZB
AB
⇔
ZB
⇔ ZB
AZ 0
Z 0B
AZ 0 + Z 0 B
=
Z 0B
AB
= 0
ZB
= Z 0B
=
und daher Z = Z 0 .
Anmerkung: Die gerichteten Längen sind notwendig, weil die Anordnung der Punkte
wesentlich ist. Alternativ kann man ohne Verwendung von gerichteten Längen den Satz von
Ceva so formulieren: Unter der Voraussetzung, dass keiner oder genau zwei der Punkte X, Y ,
Z außerhalb des Dreiecks ABC liegen, gehen AX, BY , CZ genau dann durch einen Punkt,
wenn (2.1) erfüllt ist.
Übrigens heißen die Geraden AX, BY , CZ auch Ecktransversalen.
Der Satz von Ceva hat auch noch eine manchmal sehr nützliche trigonometrische Form:
Satz 2.2. Die Geraden AX, BY und CZ schneiden sich dann und nur dann in einem
Punkt, wenn
sin ∠ACZ sin ∠BAX sin ∠CBY
·
·
= 1.
(2.2)
sin ∠ZCB sin ∠XAC sin ∠Y BA
20
Beweis. Wir wollen beweisen, dass
sin ∠ACZ sin ∠BAX sin ∠CBY
·
·
=1
sin ∠ZCB sin ∠XAC sin ∠Y BA
genau dann gilt, wenn
AZ BX CY
·
·
= 1.
ZB XC Y A
Wenden wir den Sinussatz auf die Dreiecke ABX und ACX an, so folgt
AB
BX
=
,
sin ∠BAX
sin ∠AXB
XC
CA
=
.
sin ∠XAC
sin ∠CXA
Die Winkel ∠AXB und ∠CXA sind entweder supplementär oder gleich (jenachdem, ob
X innerhalb oder außerhalb des Dreiecks liegt), daher gilt sin ∠AXB = sin ∠CXA. Durch
Division dieser Gleichungen erhalten wir somit
BX
AB · sin ∠BAX
=
XC
CA · sin ∠XAC
und analog auch
CY
BC · sin ∠CBY
=
,
YA
AB · sin ∠Y BA
AZ
CA · sin ∠ACZ
=
.
ZB
BC · sin ∠ZCB
Durch Multiplikation dieser Gleichungen erhalten wir somit
sin ∠ACZ sin ∠BAX sin ∠CBY
AZ BX CY
·
·
=
·
·
.
ZB XC Y A
sin ∠ZCB sin ∠XAC sin ∠Y BA
Ein weiterer wichtiger Satz, der meist mit dem Satz von Ceva in einem Atemzug genannt
wird, ist der Satz von Menelaos, welcher als Gegenstück zum Satz von Ceva die Kollinearitätsbedingung von X, Y , Z beschreibt.
Satz 2.3 (Satz von Menelaos). Sei ABC ein Dreieck und seien X, Y , Z Punkte auf den
Geraden BC, CA und AB. Dann liegen die Punkte X, Y , Z dann und nur dann auf einer
Geraden, wenn
BX CY AZ
·
·
= −1,
(2.3)
XC Y A ZB
wobei die Streckenverhältnisse gerichtet zu nehmen sind.
Beweis. Nehmen wir zunächst an, die Punkte X, Y , Z liegen auf einer Gerade. Seien x, y,
z die gerichteten Abstände von A, B, C zur Gerade durch X, Y , Z. Dann gilt nach dem
Strahlensatz BX/XC = −y/z, CY /Y A = −z/x, AZ/ZB = −x/y. Somit gilt
BX CY AZ
yzx
·
·
= (−1)(−1)(−1)
= −1.
XC Y A ZB
zxy
Die Umkehrung des Satzes erfolgt analog zum Satz von Ceva.
21
C
Y
X
Z
A
B
Abbildung 2.2: Satz von Menelaos
Anmerkung: Der Sinn der gerichteten Längen liegt auch hier wieder in der Anordnung
der Punkte. Alternativ kann man ohne Verwendung von gerichteten Längen den Satz von
Menelaos so formulieren: Unter der Voraussetzung, dass genau einer oder alle drei der Punkte
X, Y , Z außerhalb des Dreiecks ABC liegen, sind X, Y , Z genau dann kollinear, wenn (2.3)
(mit 1 statt −1) erfüllt ist.
Bei Aufgaben besteht sowohl beim Satz von Ceva als auch beim Satz von Menelaos meist
keine Notwendigkeit, mit gerichteten Größen zu arbeiten, da oftmals die Anordnung der
Punkte offensichtlich bzw. aus der Angabe herauszulesen ist.
Beispiel 2.1.1 (Asian Pacific MO 1992). Sei k ein Kreis mit Mittelpunkt O und Radius r
und seien k1 und k2 zwei Kreise mit den Mittelpunkten O1 und O2 sowie Radien r1 und r2 ,
sodass k1 den Kreis k von Innen im Punkt A1 , k2 den Kreis k von Innen im Punkt A2 und
die Kreise k1 und k2 sich von Außen im Punkt A berühren. Zeige, dass die Geraden OA,
O1 A2 und O2 A1 durch einen gemeinsamen Punkt gehen.
k
A1
A2
O1
A
O
O2
k2
k1
Abbildung 2.3: Zeichnung zu Beispiel 2.1.1 (Asian Pacific MO 1992)
22
Lösung. Sicherlich liegt A auf der Strecke O1 O2 , O1 auf der Strecke OA1 und O2 auf der
Strecke OA2 . Nach dem Satz von Ceva, angewandt auf das Dreieck OO1 O2 , schneiden sich
OA, O1 A2 und O2 A1 genau dann in einem Punkt, wenn
OA2 O2 A O1 A1
·
·
= 1.
A2 O2 AO1 A1 O
Allerdings gilt
OA2
A2 O 2
womit sich OA, O1 A2 und O2 A1
O 2 A O 1 A1
r r2 r1
·
=
· ·
=1
AO1 A1 O
r2 r1 r
in einem Punkt schneiden.
·
Beispiel 2.1.2. Sei ABC ein Dreieck und D, E, F Punkte auf den Seiten BC, CA, AB,
sodass die Ecktransversalen AD, BE, CF durch einen gemeinsamen Punkt gehen. M , N ,
P seien Punkte auf EF , F D, DE. Zeige, dass sich AM , BN , CP dann und nur dann in
einem Punkt schneiden, wenn sich DM , EN , F P in einem Punkt schneiden.
A
Z
Y
F
M
N
E
T
P
B
X
D
C
Abbildung 2.4: Zeichnung zu Beispiel 2.1.2
Lösung. AM schneide BC in X, BN schneide CA in Y und CP schneide AB in Z. Nach
der trigonometrischen Form des Satzes von Ceva (Satz 2.2) schneiden sich AM , BN und
CP genau dann in einem Punkt, wenn
sin ∠ACZ sin ∠BAX sin ∠CBY
·
·
= 1.
sin ∠ZCB sin ∠XAC sin ∠Y BA
Wir wollen zeigen, dass dies genau dann gilt, wenn sich DM , EN , F P in einem Punkt
schneiden, wenn also
F M EP DN
·
·
= 1.
ME PD NF
23
Wenden wir den Sinussatz auf die Dreiecke AF M und AEM an, so folgt
FM
FM
AF
=
=
,
sin ∠BAX
sin ∠F AM
sin ∠AM F
ME
ME
EA
=
=
sin ∠XAC
sin ∠M AE
sin ∠EM A
und wegen sin ∠AM F = sin ∠EM A folgt durch Division
AF · sin ∠BAX
FM
=
ME
EA · sin ∠XAC
und analog auch
EP
CE · sin ∠ACZ
=
,
PD
DC · sin ∠ZCB
DN
BD · sin ∠CBY
=
.
NF
F B · sin ∠Y BA
Durch Multiplikation erhalten wir
F M EP DN
sin ∠ACZ sin ∠BAX sin ∠CBY
·
·
=
·
·
·
ME PD NF
sin ∠ZCB sin ∠XAC sin ∠Y BA
BD CE AF
·
·
DC EA F B
.
Allerdings schneiden sich laut Voraussetzung die Ecktransversalen AD, BE, CF in einem
Punkt, somit gilt nach dem Satz von Ceva
BD CE AF
·
·
=1
DC EA F B
und daher
sin ∠ACZ sin ∠BAX sin ∠CBY
F M EP DN
·
·
=
·
·
.
ME PD NF
sin ∠ZCB sin ∠XAC sin ∠Y BA
Beispiel 2.1.3 (IMO Shortlist 1991). Sei ABC ein spitzwinkeliges Dreieck. M sei der Mittelpunkt der Seite BC und P sei jener Punkt auf der Strecke AM , sodass M B = M P . H
sei der Lotfußpunkt von P auf BC. Das Lot auf P B durch H schneide AB im Punkt Q, das
Lot auf P C durch H schneide AC in R. Zeige, dass BC eine Tangente in H an dem Kreis
durch Q, H, R ist.
Lösung. Wegen BC ⊥ P H genügt es zu zeigen, dass der Umkreismittelpunkt des Dreiecks
QHR auf P H liegt.
Seien X der Schnittpunkt von P B und HQ und sei Y der Schnittpunkt von P C und
HR. Wegen BM = M P = M C ist M der Umkreismittelpunkt des Dreiecks BP C, welcher
allerdings ein Halbkreis über BC ist. Somit ist ∠BP C = 90◦ . Somit hat das Viereck P XHY
drei rechte Winkel und ist daher ein Rechteck. Somit ist der Mittelpunkt von XY der Umkreismittelpunkt dieses Rechtecks. Da aber ∠QHR = 90◦ gilt, genügt es, zu zeigen, dass QR
zu XY parallel ist.
Sei nun D der Schnittpunkt von CP und AB und E der Schnittpunkt von BP und
AC. Wenden wir den Satz von Ceva auf das Dreieck ABC an, so folgt BM/M C · CE/EA ·
AD/DB = 1 und wegen BM = M C gilt daher
AE
EC
=
AD
DB
24
A
D
E
Q
P
X
R
Y
B
M
H
C
Abbildung 2.5: Zeichnung zu Beispiel 2.1.3 (IMO Shortlist 1991)
womit DE zu BC parallel ist. Nach dem Strahlensatz gilt somit
PD
PE
=
.
PC
PB
Da aber HQ zu CD und HR zu BE parallel ist, gilt
XQ
PD
=
,
PC
XH
PE
YR
=
PB
YH
und somit
YR
XQ
=
XH
YH
womit, wie behauptet, QR zu XY parallel ist.
Beispiel 2.1.4 (USA 2003). Es sei ABC ein Dreieck. Ein Kreis durch A und B schneide
die Seiten AC und BC jeweils in D und E. Seien weiters F der Schnittpunkt von AB
und DE, und M der Schnittpunkt von BD und CF . Beweise, dass M genau dann der
Halbierungspunkt von CF ist, wenn M B · M D = M C 2 .
Lösung. Im Dreieck BCF schneiden sich die Ecktransversalen BM , CA und F E im gemeinsamen Punkt D, daher gilt nach dem Satz von Ceva
CM F A BE
·
·
= 1.
M F AB EC
Somit gilt CM = M F genau dann, wenn
F A BE
·
= 1 bzw.
AB EC
25
BA
BE
=
AF
EC
C
E
M
D
F
B
A
Abbildung 2.6: Zeichnung zu Beispiel 2.1.4 (USA 2003)
was nach Strahlensatz genau dann der Fall ist, wenn die Geraden AE und F C parallel sind.
Dies ist allerdings genau dann der Fall, wenn ∠DAE = ∠DCF . Allerdings gilt ∠DCF =
∠DCM und nach Peripheriewinkelsatz gilt ∠DAE = ∠DBE = ∠M BC, daher ist ∠DAE =
∠DCF äquivalent zu ∠M BC = ∠DCM und dies ist genau dann der Fall, wenn die Dreiecke
M CD und M BC ähnlich sind, wenn also gilt
MC
MB
=
MC
MD
was gleichbedeutend mit M B · M D = M C 2 ist.
Beispiel 2.1.5 (Hong Kong 2006). Seien A und B feste Punkte in der Ebene und sei L eine
Gerade, die durch A und nicht durch B geht. C sei ein variabler Punkt auf einer der durch
A begrenzten Halbgeraden, die auf L verlaufen. Der Inkreis von 4ABC berühre BC in D
und AC in E. Zeige, dass DE durch einen festen Punkt geht.
Lösung. Der Inkreis von 4ABC brühre AB in F . S sei jener Punkt auf dem Strahl AC,
sodass AS = AB und P sei der Mittelpunkt der Strecke BS. Wir zeigen, dass DE immer
durch P geht.
Offensichtlich liegen D innerhalb der Strecke BC und P innerhalb der Strecke BS. E
liegt außerhalb der Strecke CS, denn es gilt AE < AC sowie nach Satz 1.2 und der Dreiecksungleichung, angewandt auf das Dreieck ABC,
AE =
AB + AC − BC
< AB = AS.
2
Wenden wir nun den Satz von Menelaos (mit ungerichteten Größen) auf das Dreieck
BCS an, so liegen die Punkte D, E, P genau dann auf einer Geraden, wenn
BD CE SP
·
·
= 1.
DC ES P B
26
C
S
E
P
L
A
F
D
B
Abbildung 2.7: Zeichnung zu Beispiel 2.1.5 (Hong Kong 2006)
Da zwei Tangenten von einem beliebigen äußeren Punkt an einen Kreis immer gleich lang
sind, gilt
DC = CE
(2.4)
sowie
BD = BF
und
AE = AF.
Somit gilt wegen AS = AB und AE = AF auch ES = BF und daher auch
ES = BD.
(2.5)
P ist per Definition der Mittelpunkt von BS, daher gilt
SP = P B.
(2.6)
Aus (2.4), (2.5) und (2.6) folgt somit, wie behauptet,
BD CE SP
·
·
= 1.
DC ES P B
2.2
Die Potenz eines Punktes
Satz 2.4 (Satz von der Potenz). Gegeben seien ein Kreis k und ein Punkt P . Eine Gerade
g durch P schneide k in den Punkten A und B. Dann hängt das Produkt P A · P B nur von
der Wahl von P und k, nicht aber von der Geraden g durch P ab.
Beweis. Eine weitere Gerade g 0 durch P schneide k in C und D (oBdA sei die Anordnung
der Punkte wie in Abbildung 2.8). Nach dem Peripheriewinkelsatz über AC gilt ∠P AC =
∠BAC = ∠BDC = ∠BDP , somit sind die Dreiecke P AC und P DB ähnlich und es gilt
P A/P D = P C/P B ⇔ P A · P B = P C · P D.
27
C
A
P
B
P
A
B
O
C
O
D
D
Abbildung 2.8: Potenz eines Punktes
Das gerichtete Produkt P A · P B wird auch als Potenz des Punktes P bezüglich des
Kreises k bezeichnet. Die Vorzeichenkonvention hat den Sinn, dass die Potenz von P zu k
negativ bzw. positiv ist, falls P innerhalb bzw. außerhalb von k liegt.
Indem man als Gerade P O (wobei O der Mittelpunkt und r der Radius von k ist) wählt,
kann man P A · P B auch berechnen als
P A · P B = (P O − r)(P O + r) = P O2 − r2 .
Falls P außerhalb von k liegt, so wird im Extremfall, wenn A und B zusammenfallen, aus
der Gerade die Tangente. Es gilt also
Satz 2.5. Ist T der Tangentenberührpunkt von P an k, so gilt
P A · P B = P T 2.
Der Satz von der Potenz eines Punktes hat eine sehr nützliche Umkehrung:
Satz 2.6. Schneiden sich die Geraden AB und CD im Punkt P , und gilt P A·P B = P C ·P D
(mit gerichteten Längen), so liegen die Punkte A, B, C, D auf einem Kreis.
Beweis. Wegen P A · P B = P C · P D gilt auch P A/P D = P C/P B, womit die Dreiecke P AC
und P DB ähnlich sind. Somit gilt ∠BAC = ∠P AC = ∠BDP = ∠BDC, also liegen die
Punkte A, B, C, D auf einem Kreis.
Beispiel 2.2.1. Gegeben sei ein rechtwinkliges Dreieck ABC mit der Hypotenuse AB. Der
Punkt F sei der Fußpunkt der Höhe auf die Seite AB. Ein Kreis berührt die Strecke F B
im Punkt P , die Strecke F C im Punkt Q und den Umkreis des Dreiecks ABC im Punkt R.
Man beweise, dass die Punkte A, Q, R auf einer Geraden liegen und die Strecken AP und
AC gleich lang sind.
28
C
R
Q
N
M
F
P
A
B
Abbildung 2.9: Zeichnung zu Beispiel 2.2.1
Lösung. Sei M der Umkreismittelpunkt von ABC (welcher der Halbierungspunkt von AB
ist) und N der Umkreismittelpunkt von P QR. Sicherlich sind M, N und R kollinear. Andererseits ist F C eine Tangente am Umkreis von 4P QR, daher gilt QN ⊥ F C und somit
QN k AB und daher gilt
∠AM R = ∠QN R.
Da aber die Dreiecke AM R und QN R beide gleichschenklig sind, sind sie somit ähnlich, d.h.
∠ARM = ∠QRN = ∠QRM,
womit A, Q, R kollinear sind.
Die Dreiecke AF Q und ARB sind ähnlich, denn sie haben den gemeinsamen Winkel bei
A und den rechten Winkel bei F bzw. R (Thaleskreis). Somit gilt
AF
AQ
=
AB
AR
⇔
AQ · AR = AF · AB.
Nach der Potenz von A in Bezug zum Umkreis von 4P QR gilt
AQ · AR = AP 2 .
Andererseits gilt nach dem Kathetensatz
AF · AB = AC 2 .
Insgesamt gilt daher AP 2 = AC 2 und folglich AP = AC.
Beispiel 2.2.2. Sei C ein Punkt auf einem Halbkreis mit dem Durchmesser AB und sei
_
D der Mittelpunkt des Kreisbogens AC. Sei E die Normalprojektion von D auf die Gerade
BC und sei F der Schnittpunkt von AE mit dem Halbkreis. Zeige, dass BF die Strecke DE
halbiert.
29
E
M
C
D
A
F
O
B
Abbildung 2.10: Zeichnung zu Beispiel 2.2.2
Lösung. Sei O der Mittelpunkt des Halbkreises. Wir zeigen zunächst, dass DE eine Tangente
am Halbkreis ist.
_
Da D der Mittelpunkt des Kreisbogens AC ist, gilt sicherlich OD ⊥ AC. Sicherlich ist
aber ∠BCA ein rechter Winkel (Satz von Thales), somit ist DE parallel zu AC und somit
gilt DE ⊥ OD, womit DE also tatsächlich eine Tangente am Halbkreis ist.
Sei nun M der Schnittpunkt von BF und DE. Verwenden wir die Potenz von M in Bezug
zum Halbkreis, so folgt
M D2 = M F · M B.
Andererseits ist aber 4M BE ein rechtwinkliges Dreieck mit Hypotenuse M B und Höhe
EF , somit gilt nach Kathetensatz
EM 2 = M F · M B,
also
EM 2 = M D2 .
und folglich EM = M D.
Beispiel 2.2.3 (ÖMO 2005). Im spitzwinkeligen Dreieck ABC wird über der Seite AC als
Durchmesser der Kreis k1 und über der Seite BC als Durchmesser der Kreis k2 gezeichnet.
Sei E der Fußpunkt der Höhe hb auf AC und F der Fußpunkt der Höhe ha auf BC. Seien L
und N die Schnittpunkte der Geraden BE mit dem Kreis k1 (L auf der Strecke BE) und K
und M die Schnittpunkte der Geraden AF mit dem Kreis k2 (K auf der Strecke AF ). Man
zeige: KLM N ist ein Sehnenviereck.
Lösung. Sei G der Fußpunkt der Höhe hc auf AB und H der Höhenschnittpunkt des Dreiecks
ABC. Sicherlich liegt G auf k1 und k2 (Satz von Thales über AC und BC). Da KGM C ein
Sehnenviereck ist, gilt aufgrund der Potenz von H in Bezug zum Kreis k2
HC · HG = HK · HM.
30
N
C
k1
k2
E
M
F
H
K
L
A
G
B
Abbildung 2.11: Zeichnung zu Beispiel 2.2.3 (ÖMO 2005)
Analog gilt auch
HC · HG = HL · HN.
Insgesamt gilt also
HK · HM = HL · HN,
womit KLM N nach Satz 2.6 ein Sehnenviereck ist.
Beispiel 2.2.4 (IMO Shortlist 2004). Seien k ein Kreis und g eine Gerade, die keinen
Schnittpunkt haben. Sei AB der zu g normale Kreisdurchmesser von k, wobei B näher bei
g liegt als A. Sei C ein beliebiger Punkt auf k verschieden von A und B. Die Gerade AC
schneidet g in D, die Gerade DE sei Tangente am Kreis k im Punkt E, wobei B und E auf
derselben Seite von AC liegen. BE schneidet g in F und AF schneidet k in G 6= A. Zeige,
dass das Spiegelbild von G an AB auf der Geraden CF liegt.
Lösung. Sei H der Schnittpunkt von CF und k. Da AB ⊥ g, bleibt zu zeigen, dass GH k g.
Dies ist genau dann der Fall, falls ∠AGH = ∠AF D. Es gilt aber
∠AGH = ∠ACH = ∠DCF.
Die Dreiecke DF A und DCF haben den Winkel ∠CDF gemeinsam. Es bleibt somit zu
zeigen, dass die Dreiecke DF A und DCF ähnlich sind.
Sei X der Schnittpunkt von AB und g. Nach Thales gilt
∠AEF = ∠AEB = 90◦ .
Somit ist AEXF ein Sehnenviereck und daher gilt
∠XAE = ∠XF E = ∠DF E.
31
X
D
F
C
B
E
G
H
A
Abbildung 2.12: Zeichnung zu Beispiel 2.2.4 (IMO Shortlist 2004)
Da sich der Periepheriewinkel über einer Sehne auch an der Tangente befindet, gilt
∠DEF = ∠BAE = ∠XAE = ∠DF E.
Somit ist 4DF E gleichschenklig mit DF = DE.
Aus der Potenz von D bezüglich k folgt DF 2 = DE 2 = DC · DA und daher
DF
DC
=
.
DA
DF
Somit sind 4DF A und 4DCF ähnlich.
Beispiel 2.2.5 (IMO 2000). Zwei Kreise k1 und k2 schneiden einander in den Punkten M
und N . Sei AB die gemeinsame Tangente an diesen zwei Kreisen in A und B, sodass M
näher bei AB liegt als N und A auf k1 und B auf k2 liegt. Sei CD die zu AB Parallele durch
M , wobei C auf k1 und D auf k2 liegt. Die Geraden AC und BD schneiden einander in E,
AN schneidet CD in P , BN schneidet CD in Q. Zeige, dass EP = EQ gilt.
Lösung. Die Geraden AB und M N schneiden einander in K. Aufgrund der Potenz von K
zu den Kreisen k1 und k2 gilt
KA2 = KM · KN = KB 2 ,
also gilt KA = KB. Wegen P Q k AB gilt somit auch
P M = QM.
Nach dem Peripheriewinkelsatz gilt
∠EAB = ∠ECD = ∠ACM = ∠M AB
32
E
A
K
C
B
P
M
D
Q
N
Abbildung 2.13: Zeichnung zu Beispiel 2.2.5 (IMO 2000)
sowie
∠EBA = ∠EDC = ∠BDM = ∠M BA.
Somit sind die Dreiecke AEB und AM B ähnlich und aufgrund der gemeinsamen Seite AB
auch kongruent, daher ist AEBM ein Deltoid und es gilt EM ⊥ AB und daher auch
EM ⊥ P Q. Somit sind die Dreiecke EP M und EQM kongruent, daher gilt EP = EQ.
2.3
Potenzgeraden
Es seien k1 und k2 zwei nicht-konzentrische Kreise mit den Mittelpunkten O1 und O2 sowie
Radien r1 und r2 . Wir suchen nun den geometrischen Ort aller Punkte, die bezüglich k1 und
k2 dieselbe Potenz haben.
Sei dazu P ein Punkt, der bezüglich k1 und k2 dieselbe Potenz hat, d.h. es gilt P O12 −r12 =
P O22 − r22 und daher
P O12 − P O22 = r12 − r22 .
(2.7)
Sei nun Q die Normalprojektion von P auf O1 O2 (siehe Abbildung 2.14). Nach dem Satz
von Pythagoras ist (2.7) äquivalent zu
QO12 − QO22 = r12 − r22 .
(2.8)
Durch (2.8) ist der Punkt Q auf O1 O2 eindeutig, somit liegt jeder Punkt P , der zu k1 und
k2 gleiche Potenz hat, auf der Normalen zu O1 O2 durch Q. Wir kommen daher zu folgendem
Schluss:
Satz 2.7. Die Menge aller Punkte, die zu zwei nicht-konzentrischen Kreisen dieselbe Potenz
hat, ist eine Gerade, die normal auf die Verbindungsgerade der Kreismittelpunkte steht.
33
P
k1
k2
Q
O2
O1
Abbildung 2.14: Potenzgerade
Diese Gerade heißt auch Potenzgerade dieser zwei Kreise. Falls sich die beiden Kreise
schneiden, so ist die Potenzgerade offensichtlicherweise die Verbindungsgerade der Schnittpunkte (falls sie sich berühren, so ist die Potenzgerade natürlich die gemeinsame Tangente
im Berührpunkt).
Seien nun k1 , k2 , k3 drei Kreise, deren Mittelpunkte nicht kollinear sind. Sei Z der Schnittpunkt der Potenzgerade von k1 und k2 und der Potenzgerade von k2 und k3 . Dieser Punkt Z
hat zu k1 und k2 gleiche Potenz, da er auf ihrer Potenzgerade liegt. Andererseits hat Z auch
zu k3 die gleiche Potenz, denn er liegt auf der Potenzgerade von k2 und k3 . Somit hat er auch
zu k3 und k1 gleiche Potenz, womit Z auf ihrer Potenzgerade liegt. Wir fassen zusammen:
Z
k2
k1
k3
Abbildung 2.15: Potenzzentrum
Satz 2.8. Sind k1 , k2 , k3 Kreise mit nicht-kollinearen Mittelpunkten, so gehen die Potenz34
geraden von k1 und k2 , k2 und k3 sowie k3 und k1 durch einen gemeinsamen Punkt. Dieser
Punkt hat bezüglich k1 , k2 , k3 die gleiche Potenz.
Dieser Punkt wird auch als Potenzzentrum dieser drei Kreise bezeichnet.
Beispiel 2.3.1 (IMO 1995). Es seien A, B, C, D vier paarweise verschiedene Punkte auf
einer Gerade in dieser Reihenfolge. Die Kreise mit den Durchmessern AC bzw BD schneiden
einander in X bzw Y . XY schneidet BC in Z. Sei P ein beliebiger Punkt auf XY verschieden
von Z. Die Gerade CP schneidet den Kreis mit Durchmesser AC in C und M , die Gerade
BP schneidet den Kreis mit Durchmesser BD in B und N . Zeige, dass die Geraden AM ,
DN und XY durch einen Punkt gehen.
M
N
X
P
A
B
Z
C
D
Y
Abbildung 2.16: Zeichnung zu Beispiel 2.3.1 (IMO 1995)
Lösung. Verwenden wir die Potenz von P bezüglich der zwei Kreise, so folgt
P M · P C = P X · P Y = P N · P B.
Somit ist BCN M ein Sehnenviereck und es gilt
∠M N B = ∠M CB.
Die Dreiecke AM C und BN D sind rechtwinklig, somit gilt
∠M AD = ∠M AC = 90◦ − ∠M CB = 90◦ − ∠M N B = 180◦ − ∠M N D
womit ADN M ebenfalls ein Sehnenviereck ist. Die Geraden AM, DN und XY sind aber die
Potenzgeraden der Umkreise von ACM , BDN und ADM N und gehen somit durch einen
gemeinsamen Punkt, das Potenzzentrum dieser Kreise.
35
Beispiel 2.3.2 (Indien 1995). Es seien ABC ein Dreieck und D und E Punkte auf den
Seiten AB bzw. AC, sodass DE zu BC parallel ist. Sei P ein beliebiger Punkt im Inneren
des Dreiecks ADE und es seien F bzw G die Schnittpunkte von DE mit BP bzw CP . Sei Q
der zweite Schnittpunkt der Umkreise der Dreiecke P DG und P F E. Zeige, dass die Punkte
A, P und Q kollinear sind.
A
H
P
I
D
F
G
E
Q
B
C
Abbildung 2.17: Zeichnung zu Beispiel 2.3.2 (Indien 1995)
Lösung. Es seien H der zweite Schnittpunkt des Umkreises von 4P DG mit AB, und I der
zweite Schnittpunkt des Umkreises von 4P F E mit AC.
Falls H zwischen A und D liegt, so gilt, da DGP H ein Sehenviereck ist,
∠DGP = 180◦ − ∠DHP.
Da DG k BC gilt
∠BCP = ∠DGP = 180◦ − ∠DHP = 180◦ − ∠BHP.
Somit ist BCP H ein Sehnenviereck.
Ansonsten, falls D zwischen A und H liegt, so gilt
∠DHP = ∠DGP
und da DG k BC, gilt somit
∠BCP = ∠DGP = ∠DHP = 180◦ − ∠BHP
womit BCP H ein Sehnenviereck ist.
Somit liegt H auf dem Umkreis von 4BCP . Analog liegt auch I auf dem Umkreis
von 4BCP . Somit ist BCIH ein Sehnenviereck und da BC k DE ist auch DEIH ein
Sehnenviereck. Somit gilt AH · AD = AI · AE. Daraus folgt, dass A auf der Potenzgerade
der Umkreise von 4P DG und 4P F E, also auf P Q, liegt.
36
Beispiel 2.3.3 (Taiwan 2000). In einem spitzwinkeligen Dreieck ABC mit AC > BC sei M
der Mittelpunkt von AB. Ferner seien P und Q die Höhenfußpunkte auf BC und AC. Sei
H der Schnittpunkt von AP und BQ. AB schneide P Q in R. Man zeige, dass die Geraden
RH und CM aufeinander normal stehen.
C
Q
N
A
P
H
B
M
R
Abbildung 2.18: Zeichnung zu Beispiel 2.3.3 (Taiwan 2000)
Lösung. Es sei N die Normalprojektion von H auf AM . Wir beweisen, dass P Q, AB und
HN durch einen gemeinsamen Punkt gehen.
Wegen ∠AP B = ∠AQB = 90◦ ist ABP Q ein Sehnenviereck mit Umkreismittelpunkt
M , daher gilt
∠CAB = ∠QAB = ∠CP Q = α
und
M A = M Q = M P = M B.
Da das Dreieck AM Q gleichschenklig ist, gilt ebenfalls
∠M AQ = ∠AQM = α.
Wegen ∠HP C = ∠HQC = ∠HN C = 90◦ liegen die Punkte C, Q, N , H, P auf einen Kreis
und es gilt
∠CP Q = ∠CN Q = α.
Somit ist AM N Q ein Sehnenviereck und es gilt
∠AQM = ∠AN M = α.
Aufgrund der Winkelsumme im Dreieck ABQ gilt aber ∠ABQ = 90◦ − α. Es gilt also
insgesamt
∠AN H + ∠ABH = ∠AN M + ∠M N H + ∠ABQ = α + 90◦ + 90◦ − α = 180◦
37
womit ABHN ein Sehnenviereck ist.
Insgesamt gibt es also einen Kreis durch A, B, H, N , einen Kreis durch A, B, P , Q sowie
einen Kreis durch P , Q, N , H. Die Geraden AB, P Q und HN sind somit die Potenzgeraden
dieser Kreise, die sich in dessen Potenzzentrum R schneiden.
2.4
Simson-Geraden
Satz 2.9 (Satz von Simson). Es sei 4ABC ein Dreieck und P ein beliebiger Punkt. Seien
X, Y und Z die Lotfußpunkte von P auf BC, CA und AB (in dieser Reihenfolge). Dann
liegen die Punkte X, Y und Z dann und nur dann auf einer Geraden, wenn P auf dem
Umkreis von 4ABC liegt.
X
P
C
Y
A
B
Z
Abbildung 2.19: Satz von Simson
Beweis. Um Fallunterscheidungen bezüglich der Anordnung der Punkte zu ersparen, seien in
diesem Beweis alle Winkel orientierte Winkel modulo 180◦ (siehe Anhang A). Da ]P XB =
]P ZB = 90◦ , liegen die Punkte P , X, Z, B auf einem Kreis. Nach Peripheriewinkelsatz
gilt somit ]ZXP = ]ZBP = ]ABP . Analog erhält man ]Y XP = ]Y CP = ]ACP . Die
Punkte X, Y , Z liegen genau dann auf einer Geraden, wenn ]ZXY = 0◦ . Nun gilt aber
]ZXY = ]ZXP + ]P XY . Daher gilt:
]ZXY
⇔ ]ZXP + ]P XY
⇔ ]ZXP − ]Y XP
⇔ ]ABP − ]ACP
⇔ ]ABP
= 0◦
= 0◦
= 0◦
= 0◦
= ]ACP.
Nun gilt aber ]ABP = ]ACP genau dann, wenn A, B, C, P auf einem Kreis liegen.
38
Falls P auf dem Umkreis von 4ABC liegt, so bezeichnet man die Gerade, die durch X,
Y und Z geht, auch als Simson-Gerade oder Wallace-Gerade von P in Bezug zum Dreieck
ABC.
Beispiel 2.4.1 (IMO 2003). Es sei ABCD ein Sehnenviereck. Ferner seien P , Q und R
die Fußpunkte der Lote von D auf die Geraden BC, CA und AB (in dieser Reihenfolge).
Man beweise, dass P Q = QR dann und nur dann gilt, wenn sich die Winkelhalbierenden der
Winkel ∠ABC und ADC auf der Geraden AC schneiden.
P
D
C
Q
A
B
R
Abbildung 2.20: Zeichnung zu Beispiel 2.4.1 (IMO 2003)
Lösung 1. 1 Die Punkte P , Q und R sind kollinear, da diese auf der Simson-Gerade von D
bezüglich 4ABC liegen.
Da ∠ARD = ∠AQD = 90◦ , ist ARQD ein Sehnenviereck und es gilt ∠DAC = ∠DAQ =
∠DRQ = ∠DRP . Analog gilt ∠DCA = ∠DP R. Somit gilt 4DAC ∼ 4DRP und daher
CD
DP
=
.
DR
AD
Da ARQD und ABCD Sehnenvierecke sind, gilt weiters ∠DRQ = ∠DAQ = ∠DAC =
∠DBC und ∠QDR = ∠QAR = ∠CAB = ∠CDB. Somit gilt 4DRQ ∼ 4DBC und daher
DR
BD
=
QR
BC
und analog auch
DP
BD
=
.
PQ
AB
Somit gilt
AB DP
AB CD
PQ
=
·
=
·
.
QR
BC DR
BC AD
1
Eine alternative Lösung dieser Aufgabe befindet sich auf Seite 44.
39
Somit gilt P Q = QR genau dann, falls AB/BC = AD/CD. Da aber in jedem Dreieck die
Winkelsymmetrale die gegenüberliegende Seite im Verhältnis der anliegenden Seiten teilt
(Satz 1.1), gilt AB/BC = AD/CD genau dann, wenn die Winkelsymmetrale von ∠ABC
und ∠ADC die gemeinsame Seite AC der Dreiecke 4ABC und 4ADC im selben Verhältnis
teilen, sich also auf AC schneiden.
Beispiel 2.4.2. Sei P ein Punkt auf dem Umkreis des Dreiecks 4ABC. Die Spiegelbilder
von P an den Seiten BC, CA und AB seien R, S und T . Man beweise, dass die Punkte R,
S, T und der Höhenschnittpunkt H des Dreiecks 4ABC auf einer Gerade liegen.
T
A
Z
P
H'
Y
H
S
X
B
C
L
R
Abbildung 2.21: Zeichnung zu Beispiel 2.4.2
Lösung. Die Punkte R, S und T liegen sicherlich auf einer Geraden, denn diese entstehen
durch die zentrische Streckung mit Zentrum P und Faktor 2 aus den Fußpunkten von P auf
BC, CA und AB, welche auf der Simsongerade von P bezüglich 4ABC liegen. Wir zeigen
im Folgenden, dass die Simsongerade die Strecke P H halbiert, woraus die Behauptung folgt
Seien dazu X, Y , Z die Lotfußpunkte von P auf BC, CA und AB (in dieser Reihenfolge).
Sei weiters L der Schnittpunkt von CH und der Simson-Geraden durch X, Y , Z und sei
weiters H 0 das Spiegelbild von H an AB. Dieser liegt bekanntlich auf dem Umkreis von
4ABC (siehe Satz A.12). Da AZY P und ABCP Sehnenvierecke sind, gilt
∠LZP = ∠Y ZP = ∠Y AP = ∠CAP = ∠CH 0 P = ∠LH 0 P
40
womit LP ZH 0 ein Sehnenviereck ist. Somit gilt, unter Verwendung der Symmetrie von H
und H 0 bezüglich AB, ∠P LH = ∠P LH 0 = 180◦ − ∠H 0 ZP = ∠H 0 ZT = ∠HZP und da
wegen P Z ⊥ AB und LH ⊥ AB die Geraden P Z und LH parallel sind, folgt, dass P ZHL
ein Parallelogramm ist. In einem Parallelogramm halbieren sich aber die Diagonalen, somit
halbiert LZ, also die Simsongerade, die andere Diagonale P H.
2.5
Lotfußpunktdreiecke
Sei 4ABC ein Dreieck und P ein beliebiger Punkt. Seien X, Y und Z die Lotfußpunkte von
P auf BC, CA und AB (in dieser Reihenfolge).
Das Dreieck 4XY Z heißt Lotfußpunktdreieck des Punktes P in Bezug auf das Dreieck
4ABC.
C
C
Y
Y
P
X
P
X
A
Z
A
B
B
Z
Abbildung 2.22: Lotfußpunktdreiecke
Satz 2.10. Für die Seitenlängen des Lotfußpunktdreieckes gilt:
YZ =
AP · BC
,
2R
ZX =
BP · CA
,
2R
XY =
CP · AB
,
2R
wobei R der Umkreisradius von 4ABC ist.
Beweis. Die Punkte A, Z, P , Y liegen auf dem Thaleskreis über AP , somit ist AP/2 der
Umkreisradius von 4AZY . Nach dem Sinussatz, angewandt auf 4AZY , gilt daher
sin α =
YZ
.
AP
Andererseits gilt nach dem Sinussatz, angewandt auf 4ABC
sin α =
41
BC
.
2R
Somit gilt
AP · BC
.
2R
Die Beweise der anderen Gleichungen erfolgen analog.
YZ =
Das Lotfußpunktdreieck kann auch zu einer Strecke entarten, was allerdings am Beweis
von Satz 2.10 nichts ändert. Diese Entartung tritt nach Satz 2.9 genau dann ein, wenn P
auf dem Umkreis von 4ABC liegt.
Beispiel 2.5.1 (IMO 1993). Sei D ein Punkt im spitzwinkeligen Dreieck 4ABC sodass
∠ADB = ∠ACB + 90◦ und AC · BD = AD · BC. Berechne (AB · CD)/(AC · BD) und
zeige, dass sich die Umkreise der Dreiecke ACD und BCD rechtwinkelig schneiden.
C
M
N
Y
X
D
A
Z
B
Abbildung 2.23: Zeichnung zu Beispiel 2.5.1 (IMO 1993)
Lösung. Sei 4XY Z das Lotfußpunktdreieck von D in Bezug auf das Dreieck 4ABC, wobei
X dem Punkt A, Y dem Punkt B und Z dem Punkt C gegenüberliegt. Aus AC · BD =
AD·BC folgt nach Satz 2.10, dass ZX = ZY , d.h. das Lotfußpunktdreieck ist gleichschenklig.
Weiters folgt nach Satz 2.10, dass das gesuchte Verhältnis (AB · CD)/(AC · BD) = XY /ZX
ist. Bezeichnen wir ∠ACB = γ, dann gilt nach Vorraussetzung
∠ADB = 90◦ + γ.
Betrachten wir nun die Winkelsumme im Viereck ADBC, so gilt
∠DAC + ∠DBC = 360◦ − γ − (360◦ − 90◦ − γ) = 90◦ .
Andererseits gilt aber, da AZDY und BZDX Sehnenvierecke sind,
∠DAC = ∠DAY = ∠DZY
sowie
42
∠DBC = ∠DBX = ∠DZX.
Somit gilt
∠XZY = ∠XZD + ∠Y ZD = 90◦ ,
(2.9)
womit das Lotfußpunktdreieck 4XY Z ein rechtwinklig-gleichschenkliges Dreieck mit Hypotenuse XY ist. In einem rechtwinklig-gleichschenkligen
Dreieck ist das Verhältnis von Hypo√
√
tenuse zur Kathete bekanntlich 2 : 1, daher gilt XY /ZX = (AB · CD)/(AC · BD) = 2.
Nun bleibt noch zu zeigen, dass sich die Umkreise der Dreiecke ACD und BCD rechtwinklig schneiden. Dies ist genau dann der Fall, wenn ∠M DN = 90◦ gilt, wobei M und N
die Umkreismittelpunkte der Dreiecke ACD und BCD sind.
Das Dreieck M DC ist gleichschenklig, daher gilt
∠CM D
.
2
Aufgrund des Peripheriewinkel-Zentriwinkelsatzes gilt allerdings
∠M DC = 90◦ −
∠CM D
= ∠CAD = ∠Y AD = ∠Y ZD,
2
also insgesamt
∠M DC = 90◦ − ∠Y ZD
und analog auch
∠CDN = 90◦ − ∠DZX.
Somit folgt unter Verwendung von (2.9)
∠M DN = ∠M DC + ∠CDN
= (90◦ − ∠Y ZD) + (90◦ − ∠DZX)
= 180◦ − (∠Y ZD + ∠DZX)
= 180◦ − ∠Y ZX = 90◦ .
Beispiel 2.5.2 (IMO 1996). Sei P ein Punkt innerhalb des Dreiecks 4ABC, sodass ∠AP B−
∠ACB = ∠AP C − ∠ABC. Die Punkte D und E seien die die Inkreismittelpunkte der Dreiecke AP B und AP C. Zeige, dass die Geraden AP , BD und CE einen gemeinsamen Punkt
haben.
Lösung 1. 2 Sei 4XY Z das Lotfußpunktdreieck von P in Bezug auf das Dreieck 4ABC,
wobei X dem Punkt A, Y dem Punkt B und Z dem Punkt C gegenüberliegt. Die Geraden
BD und CE sind die Winkelsymmetralen von ∠ABP und ∠ACP . In jedem Dreieck teilt eine
Innenwinkelsymmetrale die gegenüberliegende Seite im Verhältnis der anliegenden Seiten
(Satz 1.1), daher teilt die Gerade BD die Strecke AP im Verhältnis AB/BP und die Gerade
CE teilt die Strecke AP im Verhältnis AC/CP . Die Geraden BD und CE schneiden sich
genau dann auf AP , wenn sie die Strecke AP im gleichen Verhältnis teilen, wenn also gilt:
AB
AC
=
BP
CP
2
bzw.
AB · CP = AC · BP.
Eine alternative Lösung dieser Aufgabe befindet sich auf Seite 89.
43
C
Y
X
E
P
D
A
Z
B
Abbildung 2.24: Zeichnung zu Beispiel 2.5.2 (IMO 1996)
Nach Satz 2.10 gilt AB · CP = AC · BP allerdings genau dann, wenn XY = XZ. Wir
beweisen die dazu äquivalente Aussage ∠XY Z = ∠XZY .
Da AY P Z und CY P X Sehnenvierecke sind, gilt:
∠P Y Z = ∠P AZ = ∠P AB
sowie
∠P Y X = ∠P CX = ∠P CB.
Betrachten wir nun die Winkelsumme im Viereck ABCP , so gilt
∠P AB + ∠P CB = 360◦ − ∠ABC − (360◦ − ∠AP C) = ∠AP C − ∠ABC.
Nun gilt aber
∠P AB + ∠P CB = ∠P Y Z + ∠P Y X = ∠XY Z
und daher folgt
∠XY Z = ∠AP C − ∠ABC.
Analog erhalten wir
∠XZY = ∠AP B − ∠ACB.
Aus der Voraussetzung ∠AP B − ∠ACB = ∠AP C − ∠ABC folgt somit, wie behauptet,
∠XZY = ∠XY Z.
Beispiel 2.5.3 (IMO 2003). Sei ABCD ein Sehnenviereck. Ferner seien P , Q und R die
Fußpunkte der Lote von D auf die Geraden BC, CA und AB (in dieser Reihenfolge). Man
beweise, dass P Q = QR dann und nur dann gilt, wenn sich die Winkelhalbierenden der
Winkel ∠ABC und ∠ADC auf der Geraden AC schneiden.
44
P
D
C
Q
A
B
R
Abbildung 2.25: Zeichnung zu Beispiel 2.5.3 (IMO 2003)
Lösung 2. 3 4P QR ist das (in diesem Fall zu einer Strecke entartete) Lotfußpunkdreieck
des Punktes D in Bezug auf das Dreieck 4ABC. Nach Satz 2.10 gilt somit
PQ =
DC · AB
2R
sowie
QR =
AD · BC
2R
wobei R der Umkreisradius von 4ABC ist. Somit gilt P Q = QR dann und nur dann, wenn
DC · AB = AD · BC, wenn also gilt:
AB
AD
=
.
BC
DC
Da nach Satz 1.1 in jedem Dreieck eine Innenwinkelsymmetrale die gegenüberliegend Seite
im Verhältnis der anliegenden Seiten teilt, gilt AB/BC = AD/DC dann und nur dann, wenn
die Winkelsymmetralen von ∠ABC und ∠ADC die gemeinsame Seite AC der Dreiecke ABC
und ADC im gleichen Verhältnis teilen, sich also auf AC schneiden.
Anmerkung: Wie diese Lösung zeigt, ist die Bedingung, dass D auf dem Umkreis von
4ABC liegt, irrelevant.
2.6
Schmetterlinge
Ein bekannter und manchmal recht nützlicher Satz ist der sogenannte Schmetterlingssatz.
Wer genügend künstlerische Kreativität besitzt, kann auch versuchen, sich über die Herkunft
dieser Bezeichnung Gedanken zu machen (kleiner Tipp: Abbildung 2.26 betrachten).
Satz 2.11 (Schmetterlingssatz). Sei k ein Kreis und P Q eine Sehne von k. Durch den
Mittelpunkt M von P Q lege man zwei weitere Sehnen AB und CD. AD bzw. BC schneiden
P Q in X bzw. Y . Dann ist M auch der Mittelpunkt von XY .
3
Eine alternative Lösung dieser Aufgabe befindet sich auf Seite 39.
45
A
C
y2
x1
P
X
x
M
Y
y
Q
y1
x2
O
D
B
Abbildung 2.26: Schmetterlingssatz
Beweis. Wir fällen die Lote x1 und y1 von X und Y auf AB, und x2 und y2 von X und Y
auf CD. Sei a = P M = M Q, x = XM , y = M Y . Aufgrund ähnlicher Dreiecke gilt
x
x1
x2
=
= ,
y
y1
y2
x1
AX
,
=
y2
CY
x2
XD
,
=
y1
YB
daher gilt
x2
x1 x2
AX · XD
P X · XQ
=
=
=
2
y
y1 y2
CY · Y B
PY · Y Q
2
a − x2
a2
(a − x)(a + x)
= 2
=
=1
=
(a + y)(a − y)
a − y2
a2
und somit x = y.
Beispiel 2.6.1 (IMO Shortlist 1996). Sei ABC ein spitzwinkeliges Dreieck mit BC > CA.
Sei O der Umkreismittelpunkt und H der Höhenschittpunkt des Dreiecks ABC. Ferner sei F
der Höhenfußpunkt der Höhe CH auf der Seite AB. Die Normale auf OF durch F schneide
AC in P . Beweise, dass ∠F HP = ∠CAB.
Lösung 1. 4 Sei D der Schnittpunkt der Höhe CH mit dem Umkreis von ABC. Die Spiegelpunkte des Höhenschnittpunktes bezüglich der Dreiecksseiten liegen am Umkreis (vergleiche
Satz A.12 auf Seite 112), somit ist F der Mittelpunkt von HD. Sei Q der Schnittpunkt von
F P und der Sehne DB.
4
Eine alternative Lösung dieser Aufgabe befindet sich auf Seite 58.
46
C
O
H
P
A
B
F
Q
D
Abbildung 2.27: Zeichnung zu Beispiel 2.6.1 (IMO Shortlist 1996)
Wegen OF ⊥ P Q ist F der Mittelpunkt der Kreissehne durch P Q. Nach dem Schmetterlingssatz ist somit F auch der Mittelpunkt der Strecke P Q. Es gilt somit F P = F Q und
F H = F D, sowie ∠HF P = ∠DF Q. Somit sind die Dreiecke P HF und QDF kongruent
und es gilt
∠F HP = ∠F DQ.
Aufgrund des Peripheriewinkelsatzes über BC gilt allerdings
∠F DQ = ∠CDB = ∠CAB
womit also insgesamt ∠F HP = ∠CAB gezeigt ist.
2.7
Tangentenvierecke
Ein Viereck heißt Tangentenviereck, wenn es einen Inkreis hat.
Satz 2.12. Ein konvexes Viereck ABCD ist genau dann ein Tangentenviereck, wenn AB +
CD = BC + DA gilt.
In Worten: Ein konvexes Viereck hat genau dann einen Inkreis, wenn die Summe gegenüberliegender Seiten gleich ist.
Beweis. Der Fall, dass ABCD ein Parallelogramm ist, ist trivial, wir könnn also annehmen, dass zumindest zwei gegenüberliegende Vierecksseiten nicht parallel sind. OBdA seien
AB und CD nicht parallel und R der Schnittpunkt dieser Geraden. Wir können weiters
annehmen, dass A zwischen R und B und D zwischen R und C liegt.
Das Viereck ABCD hat dann und nur dann einen Inkreis, wenn der Inkreis des Dreiecks
RBC mit dem Ankreis des Dreiecks RDA zusammenfällt. Seien X bzw. X 0 die Berührpunkte
47
C
Z
D
Y
W
I
A
B
X
Abbildung 2.28: Tangentenviereck
auf RB mit dem Inkreis von 4RBC bzw. dem Ankreis von 4RDA. Da sowohl der Inkreismittelpunkt von 4RBC als auch der Ankreismittelpunkt von 4RDA auf der Winkelsymmetrale von ∠BRC liegen und sowohl RB als auch RC an beide Kreise Tangenten sind,
fallen die zwei Kreise genau dann zusammen, wenn X = X 0 gilt. Dies ist ganau dann der
Fall, wenn RX = RX 0 . Nach Satz 1.2 und Satz 1.5 gilt
RA + AB + RD + CD − BC
RB + RC − BC
=
,
2
2
RA + RD + DA
RX 0 =
.
2
Somit gilt RX = RX 0 genau dann, wenn AB + CD = BC + DA gilt.
RX =
Ein klassischer Satz zu Tangentenvierecken:
C
Z
D
Y
P
W
I
A
B
X
Abbildung 2.29
Satz 2.13. Sei ABCD ein Tangentenviereck, dessen Inkreis die Seiten AB, BC, CD, DA
jeweils in X, Y , Z, W berührt. Dann schneiden sich AC, BD, XZ, Y W in einem Punkt.
In Worten: In einem Tangentenviereck schneiden sich die Diagonalen und die Verbindungsgeraden gegenüberliegender Inkreisberührpunkte in einem Punkt.
48
Beweis. Dieser Satz ist (fast) trivial, wenn man den Satz von Brianchon kennt. Darauf wird
im Rahmen dieses Skriptums aber nicht näher eingegangen. Stattdessen werde ich einen
etwas elementareren Beweis angeben.
C
Z
D
Y
P
W
I
A
B
X
Abbildung 2.30
Sei P der Schittpunkt von AC und XZ. Wenden wir den Sinussatz auf die Dreiecke AP X
und CP Z an, so folgt
AX
AP
=
,
sin ∠P XA
sin ∠AP X
CP
CZ
=
.
sin ∠P ZC
sin ∠CP Z
(2.10)
(2.11)
Offensichtlich gilt ∠AP X = ∠CP Z. Da ein Peripheriewinkel auch an der Tangente zu sehen
ist, gilt sicherlich ∠AXP = ∠DZP , womit die Winkel ∠AXP und ∠P ZC supplementär
sind und daher gleiche Sinus haben. Dividieren wir daher (2.10) durch (2.11), so erhalten
wir
AP
AX
=
.
CP
CZ
Ist P 0 der Schnittpunkt von AC und Y W , so gilt analog auch
AP 0
AW
=
.
CP 0
CY
Allerdings gilt sicherlich AX = AW und CY = CZ, denn Tangentenabschnitte sind gleich
lang, daher teilen P und P 0 die Diagonale AC im gleichen Verhältnis, also fallen P und P 0
zusammen. Somit schneiden sich also XZ, Y W und AC in einem Punkt.
Anders ausgedrückt: Die Diagonale AC geht durch den Schnittpunkt P von XZ und
Y W . Was wir für eine Diagonale zeigen können, können wir für die andere natürlich genauso
zeigen, daher geht die Diagonale BD analog dazu auch durch P .
Insgesamt gehen also AC, BD, XZ, Y W durch einen Punkt P .
Übrigens haben wir gleich eine weitere Eigenschaft von Tangentenvierecken mitbewiesen:
49
Satz 2.14. Ist P der Schnittpunkt von AC, BD, XZ, Y W , so gilt
AP
AX
=
CP
CZ
und
BP
BY
=
.
DP
DW
Beispiel 2.7.1 (IMO 1962). Gegeben seien ein Kreis k und die Punkte A, B, C auf k. Man
konstruiere einen Punkt D auf k, sodass dem entstehenden Viereck ein Kreis eingeschrieben
werden kann.
D
E
Q
P
O
C
A
B
Abbildung 2.31: Zeichnung zu Beispiel 2.7.1 (IMO 1962)
Lösung. Seien zunächst zwei der drei Strecken AB, BC, CA gleich lang, oBdA sei AB = BC.
D sei der B auf k diametral gegenüberliegende Punkt. Dann ist ABCD ein Deltoid, welches
nach Satz 2.12 einen Inkreis besitzt.
Sei nun oBdA CA > AB > BC. Ein konvexes Viereck ABCD ist genau dann ein
Tangentenviereck, wenn AB + CD = BC + DA bzw.
AB − BC = AD − DC.
_
Wir wollen nun einen Punkt D auf dem Bogen AC, der B nicht enthält, konstruieren, sodass
AD − DC = AB − BC gilt. Ein solcher Punkt existiert aufgrund der Annahme, dass CA >
AB > BC.
Wir konstruieren nun einen Kreis k1 mit dem Mittelpunkt A und der gewünschten Diffe_
renz AB − BC als Radius. Gesucht sind nun ein Punkt D auf dem Bogen AC und ein Punkt
_
P auf k1 , sodass DC = DP und A, P , D kollinear sind. Dazu wählen wir auf dem Bogen AC
(der B nicht enthält) einen Punkt E mit EA > EC. Auf AE wählen wir nun einen Punkt
Q, sodass EC = EQ, wobei Q zwischen A und E liegt.
50
Wir konstruieren nun den Umkreis des Dreiecks ACQ. Dieser schneide k1 in P , wobei P
und B nicht auf derselben Seite von AC liegen. AP schneide k in D (verschieden von A).
Wir behaupten, dass nun DP = DC gilt.
Sicherlich gilt
∠P DC = ∠ADC = ∠AEC = ∠QEC
sowie
∠AP C = ∠AQC
und daher auch
∠DP C = ∠EQC.
Somit sind die Dreiecke DP C und EQC ähnlich, und daher gilt auch DP = DC.
Insgesamt gilt nun
AD + BC = AP + P D + BC = AB − BC + CD + BC = AB + CD.
51
Kapitel 3
Mehr zu Dreiecken
3.1
Die Eulersche Gerade
Aus dem Schulunterricht ist oftmals bekannt, dass in einem Dreieck der Höhenschnittpunkt
H, der Schwerpunkt S und der Umkreismittelpunkt O auf einer Geraden liegen, der sogenannten Eulerschen Geraden. Manchmal ist ebenfalls bekannt, dass HS : SO = 2 : 1. Der
Beweis dieses wahrlich schönen Satzes bleibt üblicherweise leider aus.
C
Hb
Mb
Ma
O
Ha
S
H
A
Mc
Hc
B
Abbildung 3.1: Die Eulersche Gerade
Es seien Ma , Mb , Mc die Mittelpunkte der Dreiecksseiten BC, CA, AB. Nach dem Strahlensatz ist Mb Mc zu BC parallel und es gilt 2 · Mb Mc = BC. Analoges Vorgehen für die
anderen Dreiecksseiten bringt uns zur Erkenntnis, dass das Dreieck Ma Mb Mc zum Dreieck
ABC ähnlich mit einem Ähnlichkeitsfaktor von 1/2 ist.
Weiters halbieren nach dem Strahlensatz die Schwerlinien AMa die Strecke Mb Mc , die
Schwerlinie BMb die Strecke Mc Ma und die Schwerlinie CMc die Strecke Ma Mb . Somit
52
C
Mb
Ma
S
A
B
Mc
Abbildung 3.2: Mittendreieck
sind die Schwerlinien AMa , BMb , CMc des Dreiecks ABC auch die Schwerlinien des Dreiecks Ma Mb Mc und daher ist auch der Punkt S nicht nur Schwerpunkt des Dreiecks ABC,
sondern auch Schwerpunkt des Dreiecks Ma Mb Mc . Somit entspricht die Strecke AS im
Dreieck ABC der Strecke Ma S im Dreieck Ma Mb Mc und da diese beiden Dreiecke mit
einem Ähnlichkeitsfaktor 1 : 2 ähnlich sind, gilt AS = 2 · SMa und analog natürlich auch
BS = 2 · SMb und CS = 2 · SMc . Wir fassen zusammen:
Satz 3.1. In einem Dreieck ABC mit Schwerpunkt S und den Seitenhalbierungspunkten
Ma , Mb , Mc gilt
AS = 2 · SMa , BS = 2 · SMb , CS = 2 · SMc .
In Worten: Jede Schwerlinie wird vom Schwerpunkt im Verhältnis 2 : 1 geteilt.
Das Dreieck Ma Mb Mc heißt übrigens auch Mittendreieck des Dreiecks ABC.
C
Mb
Ma
O
A
Mc
Abbildung 3.3
53
B
Der Umkreismittelpunkt O liegt auf der Streckensymmetrale von BC und daher steht
OMa normal auf BC. Da aber Mb Mc zu BC parallel ist, steht OMa auch auf Mb Mc normal.
Analog steht auch OMb auf Mc Ma normal und OMc steht auf Ma Mb normal. Es gilt somit:
Satz 3.2. Die Seitensymmetralen eines Dreiecks ABC sind die Höhen seines Mittendreiecks
Ma Mb Mc , insbesondere ist der Umkreismittelpunkt O von 4ABC der Höhenschnittpunkt
von 4Ma Mb Mc .
Betrachten wir nun die zentrische Streckung mit Zentrum S und Faktor −1/2, so geht
bei dieser Streckung das Dreieck ABC in sein Mittendreieck Ma Mb Mc über. Somit geht auch
der Höhenschnittpunkt des Dreiecks ABC in den Höhenschnittpunkt des Dreiecks Ma Mb Mc
über. Nach Satz 3.2 geht somit H in O über. Bei einer zentrischen Streckung liegen allerdings
jeder Punkt, das Abbild des Punktes und das Streckungszentrum in der Proportion des
Streckungsfaktors auf einer Geraden. Wir erhalten daher
Satz 3.3 (Satz von Euler). H, S und O liegen in dieser Reihenfolge auf einer Geraden, der
Eulerschen Geraden. Weiters gilt HS = 2 · SO.
Übrigens geht bei dieser zentrischen Streckung die Strecke AH in die Strecke Ma O, BH
in die Strecke Mb O und CH in die Strecke Mc O über, daher gilt
C
O
H
A
Mc
B
Abbildung 3.4
Satz 3.4.
AH = 2 · OMa ,
BH = 2 · OMb ,
CH = 2 · OMc .
In Worten: Die Verbindungsstrecke eines Eckpunkts zum Höhenschnittpunkt ist doppelt so
lange wie der Abstand vom Umkreismittelpunkt zur gegenüberliegenden Seite.
Die Eulersche Gerade hat noch viel mehr interessante Eigenschaften, beispielsweise wird
die Strecke OH vom Mittelpunkt des Feuerbachkreises (siehe Kapitel 3.2) halbiert, welcher
ebenfalls auf der Eulerschen Gerade liegt. Betrachten wir aber zunächst einige OlympiadeAufgaben:
54
Beispiel 3.1.1 (Asian Pacific MO 2004). Seien O der Umkreismittelpunkt und H der
Höhenschnittpunkt des spitzwinkeligen Dreiecks ABC. Man zeige, dass der Flächeninhalt
eines der Dreiecke AOH, BOH, COH die Summe der Flächeninhalte der beiden anderen
Dreiecke ist.
A
Mc
Mb
O
B
S
H
C
Ma
Abbildung 3.5: Zeichnung zu Beispiel 3.1.1 (Asian Pacific MO 2004)
Lösung. Aufgrund der Dreiecksflächenformel genügt es, zu zeigen, dass einer der Abstände
d(A, OH), d(B, OH), d(C, OH) die Summe der anderen beiden ist.
Betrachten wir nun die zentrische Streckung mit Zentrum S (Schwerpunkt von ABC)
und Faktor −1/2. Diese führt die Gerade OH in sich selbst über (denn S liegt auf OH, der
Eulerschen Gerade), und die Punkte A, B, C in die Mittelpunkte Ma , Mb , Mc der Seiten
BC, CA, AB über. Des Weiteren verkleinert diese Streckung alle Distanzen um den Faktor
1/2.
Ohne Beschränkung der Allgemeinheit schneide OH die Strecken AB und AC. Dann gilt
d(Ma , OH) =
d(A, OH)
2
denn Ma , OH sind das Abbild von A, OH durch die Streckung. Andererseits gilt offensichtlich
d(Ma , OH) =
d(B, OH) + d(C, OH)
2
und daher folgt, wie behauptet, d(A, OH) = d(B, OH) + d(C, OH).
Beispiel 3.1.2 (Balkan 1984). Sei ABCD ein Sehnenviereck und seien Ha , Hb , Hc , Hd
jeweils die Höhenschnittpunkte der Dreiecke BCD, CDA, DAB, ABC. Man zeige, dass die
Vierecke Ha Hb Hc Hd und ABCD kongruent sind.
55
Hb
Ha
D
C
E
O
Hc
Hd
A
B
Abbildung 3.6: Zeichnung zu Beispiel 3.1.2 (Balkan 1984)
Lösung. Es sei O der Umkreismittelpunkt des Sehnenvierecks ABCD und X der Mittelpunkt
von AB. Nach Satz 3.4 gilt CHd = 2 · OX und DHc = 2 · OX und somit CHd = DHc . CHd
und DHc sind aber nicht nur gleich lang, sondern auch parallel, denn beide stehen normal
auf AB. Somit ist CDHc Hd ein Parallelogramm. Sei E dessen Diagonalenschnittpunkt.
Auf analoge Weise zeigt man, dass auch DAHd Ha und BCHb Hc Parallelogramme sind.
Diese haben ebenfalls E als Diagonalenschnittpunkt, denn E ist per Definition der Halbierungspunkt von DHd und CHc . Somit halbiert E die Strecken AHa , BHb , CHc , DHd .
Eine zentrische Streckung mit Zentrum E und Faktor −1 führt somit ABCD in Ha Hb Hc Hd
über.
3.2
Der Feuerbachkreis
Satz 3.5 (Satz vom Feuerbachkreis). In einem Dreieck ABC mit Höhenschnittpunkt H
liegen die Seitenmittelpunkte Ma , Mb , Mc , die Höhenfußpunkte Ha , Hb , Hc und die Halbierungspunkte Pa , Pb , Pc der Strecken AH, BH, CH auf einem Kreis, dem sogenannten
Feuerbachkreis des Dreiecks ABC.
Beweis. Es gibt viele Beweise zu diesem Satz. Ein einfacher und leicht zu merkender sei im
Folgenden angegeben:
Wir beweisen zunächst, dass die Höhenfußpunkte Ha , Hb , Hc und die Seitenhalbierungspunkte Ma , Mb , Mc auf einem Kreis liegen.
56
A
Pa
Hc
Mb
Mc
Hb
H
Pb
B
Pc
Ma
Ha
C
Abbildung 3.7: Der Feuerbachkreis
Sicherlich sind die Dreiecke AMb Mc und Ma Mb Mc kongruent und daher sind auch ihre
Umkreise gleich groß. Da diese aber die Punkte Mb und Mc gemeinsam haben, sind diese
beiden Kreise bezüglich Mb Mc symmetrisch. Per Definition gilt AHa ⊥ BC. Da aber BC
zu Mb Mc parallel ist, gilt auch AHa ⊥ Mb Mc . Laut Strahlensatz ist somit Mb Mc die Streckensymmetrale von AHa und somit geht bei der Spiegelung an Mb Mc der Punkt A in den
Punkt Ha und der Umkreis von 4AMb Mc in den Umkreis von 4Ma Mb Mc über. Da aber A
auf dem Umkreis von 4AMb Mc liegt, muss das Abbild von A, also Ha , auf dem Abbild des
Umkreises von 4AMb Mc , also dem Umkreis von 4Ma Mb Mc , liegen. Analog liegen auch Hb
und Hc auf dem Umkreis von 4Ma Mb Mc .
A
Mb
Mc
B
Ma
Ha
C
Abbildung 3.8
Wir haben also somit bewiesen, dass in jedem Dreieck die Seitenhalbierungspunkte und
die Höhenfußpunkte auf einem Kreis liegen. Dies gilt für jedes Dreieck und daher auch für das
Dreieck BCH. Sicherlich gilt HHa ⊥ BC, BHc ⊥ CH, CHb ⊥ BH. Somit sind Ha , Hb , Hc
57
auch die Höhenfußpunkte des Dreiecks BCH und daher liegen die Seitenhalbierungspunkte
dieses Dreiecks, also Pb , Pc (und analog auch Pa ) auf dem Umkreis von 4Ha Hb Hc .
Insgesamt liegen also alle neun Punkte auf dem besagten Kreis.
Betrachten wir nun die zentrische Streckung mit Zentrum H und Faktor 1/2. Diese führt
das Dreieck ABC in das Dreieck Pa Pb Pc über, daher geht auch der Umkreismittelpunkt O
des Dreicks ABC in den Umkreismittelpunkt N des Dreiecks Pa Pb Pc über, der allerdings
der Mittelpunkt des Feuerbachkreises ist. Wir erhalten somit:
A
Pa
Hc
Mb
Mc
O
Hb
N
H
Pb
B
Pc
Ma
Ha
C
Abbildung 3.9: Mittelpunkt des Feuerbachkreises
Satz 3.6. Der Mittelpunkt N des Feuerbachkreises liegt auf der Eulerschen Geraden und
halbiert die Strecke OH.
Beispiel 3.2.1 (IMO Shortlist 1996). Sei ABC ein spitzwinkeliges Dreieck mit BC > CA.
Sei O der Umkreismittelpunkt und H der Höhenschittpunkt des Dreiecks ABC. Ferner sei
F der Fußpunkt der Höhe CH auf der Seite AB. Die Normale auf OF durch F schneide
AC in P . Beweise, dass ∠F HP = ∠CAB.
Lösung 2.
1
Sei M der Mittelpunkt von AC und Q der Mittelpunkt von OP . Dann gilt
∠OF P = ∠OM P = 90◦
womit Q der Mittelpunkt des Kreises durch O, F , P , M ist. Sei N der Halbierungspunkt
von OH. Nach Satz 3.6 ist dies der Mittelpunkt des Feuerbachkreises durch F , M (und
1
Eine alternative Lösung dieser Aufgabe befindet sich auf Seite 46.
58
C
H
M
N
P
Q
O
A
B
F
Abbildung 3.10: Zeichnung zu Beispiel 3.2.1 (IMO Shortlist 1996)
andere Punkte). Sowohl N als auch Q liegen auf der Streckensymmetrale von F M , denn
F M ist die gemeinsame Sehne der beiden Kreise. Somit steht N Q normal auf F M . Da N
der Mittelpunkt von OH und Q der Mittelpunkt von OP ist, folgt nach dem Strahlensatz,
dass N Q zu HP parallel ist. Somit steht HP normal auf F M , daher gilt
∠F HP = 90◦ − ∠M F H.
Allerdings ist M der Umkreismittelpunkt des Dreiecks AF C, daher gilt
∠M F H = ∠M F C = ∠M CF = 90◦ − ∠CAB.
Insgesamt erhalten wir daher ∠F HP = ∠CAB, wie zu zeigen war.
3.3
Isogonal Konjugierte
Es sei ABC ein Dreieck und N und M Punkte auf der Geraden BC, sodass ∠BAN =
−∠CAM , d.h. die Geraden AM und AN gehen bei der Spiegelung an der Winkelsymmetrale
von ∠CAB ineinander über.
Dann heißen die Geraden AN und AM isogonal zum Winkel CAB.
Satz 3.7. Sei ABC ein Dreieck und M und N Punkte auf der Geraden BC. Dann sind die
Geraden AM und AN genau dann zum Winkel CAB isogonal, wenn
BM · BN
AB 2
=
,
CM · CN
AC 2
wobei die Strecken auf der linken Seite der Gleichung gerichtet zu nehmen sind.
59
(3.1)
A
B
M
N
C
Abbildung 3.11: Isogonale Geraden
Beweis. Nehmen wir zunächst an, die Geraden AM und AN seien zum Winkel CAB isogonal. Wenden wir den Sinussatz auf 4ABM und 4ABN an, so erhalten wir
AB
BM
=
sin ∠BAM
sin ∠AM B
und
BN
AB
=
.
sin ∠BAN
sin ∠AN B
Durch Multiplikation erhalten wir also
AB 2
BM · BN
=
sin ∠BAM · sin ∠BAN
sin ∠AM B · sin ∠AN B
(3.2)
CM · CN
AC 2
=
.
sin ∠CAM · sin ∠CAN
sin ∠AM C · sin ∠AN C
(3.3)
und analog auch
Wegen ∠BAN = −∠CAM gilt auch ∠BAM = −∠CAN und daher
sin ∠BAM · sin ∠BAN = sin ∠CAM · sin ∠CAN.
Andererseits gilt
sin ∠BM A · sin ∠BN A = sin ∠CM A · sin ∠CN A
denn die Winkel ∠BM A und ∠CM A sowie ∠BN A und ∠CN A sind entweder beide gleich
oder supplementär. Dividieren wir daher (3.2) durch (3.3), so erhalten (3.1).
Nehmen wir nun an, (3.1) sei erfüllt. M 0 sei jener Punkt auf BC, sodass AN und AM 0
zum Winkel CAB isogonal sind. Dann gilt (BM 0 · BN )/(CM 0 · CN ) = AB 2 /AC 2 und
zusammen mit (3.1) daher BM 0 /CM 0 = BM/CM . Somit muss M 0 = M gelten.
Sei nun ABC ein Dreieck und X, Y und Z Punkte auf BC, CA und AB. Seien weiters
AX 0 , BY 0 , CZ 0 die zu AX, BY , CZ bezüglich der Winkel CAB, ABC, BCA isogonale
Geraden (mit X 0 auf BC, Y 0 auf CA und Z 0 auf AB).
Satz 3.8. Die Ecktransversalen AX 0 , BY 0 , CZ 0 schneiden sich dann und nur dann in einem
Punkt, wenn sich die Ecktransversalen AX, BY , CZ in einem Punkt schneiden.
60
A
Y
Z'
Z
P
I
Q
Y'
C
B
X
X'
Abbildung 3.12: Isogonal Konjugierte
Beweis. Nach Satz 3.7 gilt
BX · BX 0
AB 2
=
CX · CX 0
AC 2
⇔
BX
AB 2 CX 0
=
·
.
XC
CA2 X 0 B
Zyklische Multiplikation ergibt daher
CX 0 BZ 0 AY 0
AB 2 · BC 2 · CA2
CX 0 BZ 0 AY 0
BX CY AZ
·
·
=
· 0 · 0 · 0 = 0 · 0 · 0 .
XC Y A ZB
CA2 · AB 2 · BC 2
XB ZA Y C
XB ZA Y C
Die Behauptung folgt nun aus dem Satz von Ceva.
Es seien nun X, Y , Z so gewählt, dass sich AX, BY , CZ in einem Punkt P schneiden.
Sei Q der Schnittpunkt von AX 0 , BY 0 , CZ 0 , dessen Existenz wir soeben bewiesen haben.
Dann heißen die Punkte P und Q isogonal konjugiert in Bezug auf das Dreieck ABC.
Beispiel 3.3.1 (IMO 2004). In einem konvexen Viereck ABCD halbiere die Diagonale BD
weder den Winkel ∠ABC noch den Winkel CDA. Es sei P ein Punkt im Inneren des Vierecks
ABCD, der die Gleichungen
∠P BC = ∠DBA
und
∠P DC = ∠BDA
erfüllt. Man beweise, dass das Viereck ABCD dann und nur dann ein Sehnenviereck ist,
wenn AP = CP .
Lösung. Nehmen wir zunächst an, ABCD sei ein Sehnenviereck. Die Geraden BP und DP
schneiden den Umkreis von ABCD ein weiteres Mal in den Punkten E und F . Da aufgrund
61
D
C
P
A
B
Abbildung 3.13: Zeichnung zu Beispiel 3.3.1 (IMO 2004)
_
_
_
_
der gegebenen Bedingung AB = CF und AD = CE gilt, folgt, dass BF und DE zu AC
parallel sind. Somit ist BDEF ein gleichschenkeliges Trapez, dessen Diagonalen sich in P
schneiden. P muss daher auf dem Durchmesser des Umkreieses liegen, welcher normal auf
AC steht. Somit gilt AP = CP .
Nehmen wir nun AP = CP an. Die Geraden BP und DP schneiden AC in K und L. Die
Punkte A und C sind bezüglich 4BDP isogonal konjugiert, daher gilt ∠AP K = ∠CP L.
Wegen AP = CP sind daher K und L symmetrisch bezüglich der Streckensymmetrale von
AC.
Sei nun E der an der Streckensymmetrale von AC gespiegelte Punkt von D. Dann liegt
E auf BP und die Dreiecke AP D und CP E sind kongruent. Somit gilt ∠BDC = ∠ADP =
∠BEC, womit die Punkte B, C, E, D auf einem Kreis liegen. Analog liegen auch A, C, E,
D auf einem Kreis. Somit ist ABCD ein Sehnenviereck.
3.4
Symmedianen
Es sei ABC ein Dreieck mit Schwerpunkt S. Sei L der zu S isogonal konjugierte Punkt
(bezüglich 4ABC). Dieser Punkt L heißt auch Lemoine-Punkt des Dreiecks ABC. Die
Geraden AL, BL, CL heißen auch Symmedianen des Dreiecks ABC. Sie sind per Definition
zu den Schwerlinien isogonal.
Der Lemoine-Punkt, wie auch die Symmedianen, haben viele besondere und interessante
Eigenschaften. Für Wettbewerbe können vor allem aber folgende von großem Nutzen sein.
Satz 3.9. Die Symmediane durch A schneide BC im Punkt X. Dann gilt
AB 2
BX
=
.
CX
AC 2
Beweis. Dies ist eine triviale Konsequenz aus Satz 3.7, denn AX ist zur Schwerlinie durch
A isogonal, welche die Seite BC halbiert.
62
A
Mc
Mb
S
L
I
B
C
Ma
Abbildung 3.14: Der Lemoine-Punkt
Satz 3.10. Es seien tB und tC die Tangenten in B bzw. C an den Umkreis von 4ABC. Sei
P der Schnittpunkt von tB und tC . Dann liegt P auf der Symmediane von 4ABC durch A.
A
γ
β
B
C
γ
X
β
γ
γ
β
B'
P'
C'
P
β
Y
Abbildung 3.15: Symmedianen und der Tangentenschnittpunkt
Beweis. Es seien α = ∠CAB, β = ∠ABC, γ = ∠BCA. Wir betrachten eine Gerade g durch
P , die mit AB den Winkel γ und mit AC den Winkel β einschließt 2 (siehe Abbildung 3.15).
Diese schneide AB bzw. AC in X bzw. Y .
Da ein Peripheriewinkel auch an der Tangente zu sehen ist, gilt ∠P BC = ∠P CB = α.
Somit gilt ∠P BX = 180◦ − β − α = γ, daher ist wegen ∠P BX = ∠P XB = γ das Dreieck
2
Eine solche Gerade heißt auch antiparallel zu BC bezüglich der Geraden AB und AC.
63
P BX gleichschenklig und es gilt P B = P X, analog auch P C = P Y . Da aber P B = P C,
muss auch P X = P Y gelten, d.h. P ist der Halbierungspunkt von XY und somit ist AP
die Schwerlinie durch A im Dreieck AXY .
Die Spiegelung an der Winkelsymmetralen von ∠CAB führe die Punkte X, Y , P jeweils in
die Punkte C 0 , B 0 , P 0 über. Da die Geraden AB und AC bezüglich der Winkelsymmetrale von
∠CAB symmetrisch sind, muss C 0 auf AC und B 0 auf AB liegen. Nun gilt aber ∠AB 0 C 0 =
∠ABC = β, womit das Dreieck AB 0 C 0 zum Dreieck ABC ähnlich ist, insbesondere ist B 0 C 0
zu BC parallel. Andererseits muss aber P 0 der Halbierungspunkt von B 0 C 0 sein. Somit ist
AP 0 die Schwerlinie im Dreieck AB 0 C 0 durch A und ist wegen BC k B 0 C 0 daher auch eine
Schwerlinie im Dreieck ABC. Somit folgt, dass AP die zur Schwerlinie durch A isogonale
Gerade ist, welche per Definition die Symmediane ist.
Beispiel 3.4.1 (IMO Shortlist 2003). Es seien A, B, C verschiedene Punkte auf einer Geraden in dieser Reihenfolge. Sei k ein Kreis, der durch A und C geht und dessen Mittelpunkt
nicht auf AC liegt. Die Tangenten an k in A und C schneiden sich in P . Die Strecke P B
schneidet k in Q. Zeige, dass der Schnittpunkt von AC und der Winkelsymmetrale von
∠AQC unabhängig von k ist.
P
Q
A
B
C
Abbildung 3.16: Zeichnung zu Beispiel 3.4.1 (IMO Shortlist 2003)
Lösung. Eine Winkelsymmetrale teilt im Dreieck die gegenüberliegende Seite im Verhältnis
der anliegenden Seiten, es genügt daher zu zeigen, dass AQ/CQ unabhängig von der Wahl
von k ist.
Nach Satz 3.10 ist QB eine Symmediane im Dreieck ACQ. Nach Satz 3.9 gilt daher
r
AB
AQ2
AQ
AB
=
und somit
=
2
CB
CQ
CQ
CB
und dieses Verhältnis ist unabhängig von der Wahl von k.
64
3.5
Satz von Stewart
Der Satz von Stewart ist eine manchmal praktische, allerdings oft als brutal angesehene
Methode, mit der man die Länge einer Ecktransversale berechnen kann.
Satz 3.11 (Satz von Stewart). Sei ABC ein Dreieck und P ein Punkt auf der Strecke BC.
Sei a = BC, b = CA, c = AB, m = BP , n = P C, p = AP . Dann gilt
a(p2 + mn) = b2 m + c2 n.
(3.4)
A
c
b
p
B
m
P
n
C
a
Abbildung 3.17: Satz von Stewart
Beweis. Sei θ = ∠AP B. Durch Anwendung des Cosinussatzes in den Dreiecken AP B und
AP C erhalten wir
c2 = p2 + m2 − 2pm cos θ,
b2 = p2 + n2 + 2pn cos θ.
(3.5)
(3.6)
Multiplizieren wir (3.5) mit n und (3.6) mit m und addieren die beiden Gleichungen, so
erhalten wir unter der Berücksichtigung, dass m + n = a gilt, wie gewünscht (3.4).
Mit diesem Satz kann man nun beispielsweise die bekannte Formel für die Schwerlinie
herleiten (für m = n = a/2).
Satz 3.12. Sei sa die Schwerlinie durch A im Dreieck ABC. Dann gilt
sa =
1√ 2
2b + 2c2 − a2 .
2
65
A
O
S
H
x
B
Ma
C
Abbildung 3.18: Zeichnung zu Beispiel 3.5.1
Beispiel 3.5.1. Sei ABC ein Dreieck mit den Seitenlängen a, b, c, Höhenschnittpunkt H,
Umkreismittelpunkt O und Umkreisradius R. Beweise, dass OH 2 = 9R2 − a2 − b2 − c2 .
Lösung. Es sei Ma der Halbierungspunkt von BC und S der Schwerpunkt des Dreiecks ABC.
Nach Satz 3.3 liegt S auf OH (nämlich der Eulerschen Geraden) und teilt die Strecke OH
im Verhältnis 1 : 2. Somit gilt OH = 3OS.
Es sei x = SMa . Da der Schwerpunkt die Schwerlinie im Verhältnis 1 : 2 teilt, gilt
AS = 2x und AMa = 3x. Nach Satz 3.12 gilt
3x =
1√ 2
2b + 2c2 − a2 .
2
(3.7)
Da 4OMa B ein rechtwinkeliges Dreieck mit Hypotenuse OB = R ist, gilt nach Pythagoras
OMa2
a2
=R − .
4
2
(3.8)
Wenden wir den Satz von Stewart auf das Dreieck OAMa an, so folgt
AMa (OS 2 + AS · SMa ) = OMa2 · AS + OA2 · SMa
= 2x · OMa2 + R2 · x = x(2OMa2 + R2 )
und wegen AMa = 3x und SMa = x gilt
3 · OS 2 = 2 · OMa2 + R2 − 6x2 .
Setzen wir in (3.9) nun (3.7) und (3.8) ein, so erhalten wir
a2
1
+ R2 −
2b2 + 2c2 − a2
2
6
2
2
2
2
2
⇔ 9 · OS = 9R − a − b − c .
3 · OS 2 = 2R2 −
66
(3.9)
Kapitel 4
Transformationen
In Kapitel 3.1 haben wir öfters zentrische Streckungen verwendet (z.B. beim Beweis von
Satz 3.3). Diese ist eine Art von Transformationen, andere sind zum Beispiel die Translation
(Schiebung) oder die Rotation (Drehung).
Allgemein ist eine Transformation eine Abbildung der gesamten Ebene in sich selbst,
sodass jeder Punkt P ein eindeutiges Abbild P 0 hat und jeder Punkt Q0 Abbild eines eindeutigen Punktes Q ist (also eine bijektive Abbildung).
In diesem Kapitel sollen die für die Mathematikolympiade wichtigsten Transformationen
erläutert werden.
4.1
Translation
Bei einer Translation oder Schiebung bewegt sich jeder Punkt um eine feste Strecke in eine
feste Richtung. Eine Translation kann daher auch durch einen Vektor ~t beschrieben werden.
Die Translation mit dem Nullvektor wird auch als Identität bezeichnet.
A'
B'
A
B
Abbildung 4.1: Translation
67
Bei einer Translation entsteht aus einer gegebenen Strecke AB ein Parallelogram ABB 0 A0
(welches auch zu einer Strecke ausarten kann).
Mit Translationen alleine können nur selten ganze Olympiadeaufgaben gelöst werden, sie
können allerdings manchmal bei Konstruktionsproblemen von großem Nutzen sein (es gibt
manchmal Aufgaben, bei denen das Konstruieren einer genauen Zeichnung nicht trivial ist eine genaue Zeichnung ist bei den meisten Geometriebeispiel aber das Wichtigste!).
Demonstrieren wir zunächst, wie eine Translation prinzipiell angewendet werden kann:
Beispiel 4.1.1. Einem Kreis k mit Mittelpunkt O und Radius r ist ein Rechteck einzuschreiben, welches zwei gegenüberliegende Seiten hat, die zu einer gegebenen Strecke der Länge
a < 2r parallel und gleich lang sind.
a
D
k
C
O
A
k'
O'
B
Abbildung 4.2: Zeichnung zu Beispiel 4.1.1
Lösung. Wir verschieben den Kreis k um einen der beiden Vektoren, der durch a repräsentiert
werden. Bezeichnen wir das Rechteck mit ABCD, wobei AB = CD = a gelten möge, so
geht bei dieser Translation der Punkt A in den Punkt B über, welcher auf dem entstehenden
Kreis k 0 aber dem Punkt A auf k entspricht. Daher sind B und C die Schnittpunkte der
beiden Kreise k und k 0 . Schieben wir B und C wieder zurück, so erhalten wir das gesuchte
Rechteck ABCD.
Beispiel 4.1.2 (IMO 2005). Auf den Seiten eines gleichseitigen Dreiecks ABC werden A1
und A2 auf BC, B1 und B2 auf CA und C1 und C2 auf AB gewählt, sodass diese Punkte
die Eckpunkte eines konvexen Sechsecks mit gleich langen Seiten sind. Zeige, dass sich die
Diagonalen des Sechsecks in einem Punkt schneiden.
Lösung. Versuchen wir zunächst, ein solches Sechseck exakt zu konstruieren.
Wir beginnen mit einem Dreieck AC1 B2 , welches bei A einen Winkel von 60◦ hat. Sei
a = C1 B2 die Länge der Sechseckseite. Nun können wir auch ganz einfach die Punkte C2
68
k1
C
A2
B1
k3
A1
B2
P
k2
A
C1
C2
B
Abbildung 4.3: Zeichnung zu Beispiel 4.1.2 (IMO 2005)
und B1 konstruieren, indem wir einfach a auf AC1 und AB2 in die entsprechende Richtung
abtragen.
Auf dem Kreis k1 mit Mittelpunkt B1 und Radius a liegen B2 und A2 , auf dem Kreis k2
mit Mittelpunkt C2 und Radius a liegen C1 und A1 . Wir benötigen noch die Punkte A1 und
A2 .
Wir kennen die Richtung von A1 A2 , daher können wir k2 in diese Richtung um a verschieben. Der Punkt C2 gehe dabei in den Punkt P und der Kreis k2 in den Kreis k3 über.
Bei dieser Translation geht der Punkt A1 , der vorher auf k2 lag, in den Punkt A2 über, der
nun auf k3 liegt. Dieser liegt aber auch auf k1 und ist daher der Schnttpunkt von k1 und k3 .
Verschieben wir A2 zurück, so erhalten wir A1 . Damit ist die Konstruktion gelöst. Lösen wir
nun das eigentliche Beispiel, welches nach der Konstruktion relativ einfach ist.
Die Schieberichtung bezüglich AB beträgt 60◦ , daher ist das Dreieck C1 C2 P gleichseitig.
Somit gilt
C1 P = C2 P = C1 C2 = a.
Da aber P C2 k A1 A2 , ist A1 A2 P C2 ein Rhombus und analog ist auch B1 B2 C1 P ein Rhombus.
Somit gilt
P B1 = P A2 = a
womit auch 4A2 P B1 gleichseitig ist.
Insgesamt ist somit A1 A2 B1 B2 C1 C2 in A1 B2 symmetrisch, womit sich also C1 A2 und
B1 C2 auf der Symmetrieachse A1 B2 schneiden.
69
4.2
Rotation
Eine weitere Klasse von Transformationen bilden die Rotationen oder Drehungen. Dabei
wird die gesamte Ebene um einen festen Punkt, das sogenannte Drehzentrum, und um einen
bestimten Drehwinkel gedreht. Dabei bleiben sämtliche Längen gleich und alle Punkte bewegen sich auf konzentrischen Kreisen um das Drehzentrum, welches der einzige Fixpunkt
ist (die einzige Ausnahme hierzu bildet die Identität, welche eine Rotation um 0◦ ist).
A'
B'
B
A
Z
Abbildung 4.4: Rotation
Um die Anwendung einer Drehung zu demonstrieren, beweisen wir einen bekannten Satz.
Satz 4.1 (Fermat). Über den Seiten AB, BC, CA eines Dreiecks ABC werden nach außen
gleichseitige Dreiecke ABC 0 , BCA0 , CAB 0 errichtet. Dann schneiden sich AA0 , BB 0 , CC 0
in einem Punkt F , der auch Fermatpunkt des Dreiecks ABC heißt und auf den Umkreisen
der Dreiecke BCA0 , CAB 0 und ABC 0 liegt, unter einem Winkel von 60◦ . Des Weiteren gilt
AA0 = BB 0 = CC 0 .
C'
A
60°
60°
B'
F 60°
60°
60°
60°
B
60°
60°
C
A'
Abbildung 4.5: Fermatpunkt
70
Beweis. Sei F der Schnittpunkt von CC 0 und BB 0 . Eine Drehung um 60◦ um A führt C 0 in B
und C in B 0 und somit das Dreieck AC 0 C in das Dreieck ABB 0 über. Somit ist ∠BF C 0 = 60◦
und CC 0 = BB 0 . Analog gilt auch CC 0 = AA0 , also gilt insgesamt
AA0 = BB 0 = CC 0 .
Des Weiteren sind AC 0 BF und AB 0 CF Sehnenvierecke, daher gilt ∠AF B = ∠CF A = 120◦
und daher ∠BF C = 120◦ , womit auch BA0 CF ein Sehnenviereck ist. Somit gilt ∠BF A0 =
60◦ , womit also insgesamt ∠A0 F A = 180◦ gilt und F daher auch auf AA0 liegt.
Beispiel 4.2.1 (IMO 2005). Gegeben sei ein konvexes Viereck ABCD, in dem die Seiten BC
und AD gleich lang und nicht parallel sind. Auf den Seiten BC bzw. AD werden die inneren
Punkte E bzw. F so gewählt, dass BE = DF gilt. Die Geraden AC und BD schneiden sich
in P , die Geraden BD und EF schneiden sich in Q und die Geraden EF und AC schneiden
sich in R. Es werden alle Dreiecke P QR betrachtet, wenn E und F variieren. Man beweise,
dass die Umkreise dieser Dreiecke einen von P verschiedenen gemeinsamen Punkt haben.
D
C
F
P
R
Q
E
S
A
B
Abbildung 4.6: Zeichnung zu Beispiel 4.2.1 (IMO 2005)
Lösung. Es sei S der Schnittpunkt der Streckensymmetralen von AC und BD. Somit gilt
SA = SC
und SD = SB
und laut Angabe gilt AD = BC. Somit sind die Dreiecke ASD und CSB (gleichsinnig)
kongruent und daher gibt es eine Drehung um S, die das Dreieck ASD in das Dreieck CSB
überführt. Sei ϕ der Drehwinkel. Bei dieser Drehung geht der Punkt F in den Punkt E über,
daher gilt auch
SE = SF
71
womit die Dreiecke DF S und BES kongruent sind. Somit gilt
∠ASC = ∠DSB = ∠F SE = ϕ
und da die Dreiecke ASC, DSB, F SE alle gleichschenklig sind, gilt
ϕ
∠SAC = ∠SF E = ∠F ES = ∠DBS = 90◦ − .
2
Somit sind AF RS und BEQS Sehnenvierecke, daher gilt
∠P RS = 180◦ − ∠ARS = 180◦ − ∠AF S = ∠DF S = ∠BES = ∠BQS = 180◦ − ∠P QS
womit P QSR ein Sehnenviereck ist.
4.3
Geradenspiegelung
A
A'
B
B'
C
C'
Abbildung 4.7: Geradenspiegelung
Die Spiegelung oder Reflexion an einer Geraden ist eine weitere Transformation. Auch
diese erhält sämtliche Distanzen, jedoch ändern sämtliche Winkel ihre Orientierung (alles
wird “spiegelverkehrt“).
Es sei g eine Gerade, an der gespiegelt werden soll. Jeder Punkt auf g wird auf sich selbst
abgebildet, jeder Punkt A, der nicht auf g liegt, wird auf jenen Punkt A0 abgebildet, sodass
g die Streckensymmetrale von AA0 ist.
Die Spiegelung ist eine wichtige Transformation, denn die beiden bisher behandelten
Transformationen können als Produkt von Geradenspiegelungen dargestellt werden.
Satz 4.2. Jede Translation lässt sich als Produkt zweier Geradenspiegelungen darstellen,
wobei die Spiegelgeraden g1 und g2 normal auf den Translationsvektor stehen und g2 aus g1
durch eine Translation um den halbierten Vektor hervorgeht (siehe Abbildung 4.8).
Ändert man die Reihenfolge, in der an g1 und g2 gespiegelt wird, so wird in die entgegengesetzte Richtung geschoben.
72
t
t/2
A
A'
A''
B'
B
B''
C'
C
C''
Abbildung 4.8: Translation durch zwei Geradenspiegelungen ersetzen
Satz 4.3. Jede Rotation mit Zentrum Z und Drehwinkel θ lässt sich als Produkt zweier
Geradenspiegelungen darstellen, wobei sich die Spiegelgeraden g1 und g2 in Z schneiden und
einen Winkel von θ/2 einschließen (siehe Abbildung 4.9).
Ändert man die Reihenfolge, in der an g1 und g2 gespiegelt wird, so wird in die entgegengesetzte Richtung rotiert.
A'
A
B'
C'
A''
B''
C''
B
C
θ/2
θ
Z
Abbildung 4.9: Rotation durch zwei Geradenspiegelungen ersetzen
Als Anwendung dieser Sätze beweisen wir einen sehr bekannten Satz:
Satz 4.4 (Napoleon). Sei ABC ein beliebiges Dreieck. Es werden über den Dreiecksseiten
nach außen die gleichseitigen Dreiecke ABC 0 , BCA0 , CAB 0 mit den Mittelpunkten Oc , Oa ,
73
B'
C
Ob
A'
Oa
A
B
Oc
C'
Abbildung 4.10: Napoleondreieck
Ob errichtet. Dann ist 4Oa Ob Oc ein gleichseitiges Dreieck. Dieses Dreieck heißt auch Napoleondreieck des Dreiecks ABC.
Beweis. Eine Drehung um Oc um 120◦ führt A in B, eine Drehung um Oa um 120◦ führt B
in C und eine Drehung um Ob um 120◦ führt C wieder in A über.
Sei P ein Punkt auf derselben Seite von Oa Oc wie Ob , sodass Oa Oc P ein gleichseitiges
Dreieck ist. Wir wollen zeigen, dass Ob = P gilt.
Die Geraden P Oc und Oa Oc schneiden sich in Oc und schließen einen Winkel von 60◦
ein. Somit entspricht eine Drehung um 120◦ um Oc den Spiegelungen an P Oc und Oa Oc (in
dieser Reihenfolge).
Die Geraden Oc Oa und P Oa schneiden sich in Oa und schließen einen Winkel von 60◦
ein. Somit entspricht eine Drehung um 120◦ um Oa den Spiegelungen an Oc Oa und P Oa (in
dieser Reihenfolge).
Somit entspricht das Produkt der Drehung um Oc und Oa um jeweils 120◦ den Spiegelungen an den Geraden P Oc , Oa Oc , Oc Oa , P Oa (in dieser Reihenfolge). Die Spiegelungen an
Oa Oc bzw. Oc Oa heben sich aber auf, somit entspricht das Produkt dieser beiden Drehungen
dem Produkt der Spiegelungen an P Oc und P Oa (in dieser Reihenfolge).
Da sich aber P Oc und P Oa in P schneiden und einen Winkel von 60◦ einschießen, entsprechen die Spiegelungen an P Oc und P Oa einer Drehung um P um 120◦ , bei der A in C
übergeht. Eine Drehung um Ob um 120◦ in die entgegensetzte Richtung führt C aber wieder
in A zurück. Dies ist nur möglich, wenn die beiden Drehzentren zusammenfallen, wenn also
Ob = P gilt.
74
4.4
Zentrische Streckung
Die bisherigen Transformationen hatten eines gemeinsam: Sämtliche Längen bleiben während
der Transformation gleich. Solche Transformationen werden auch Kongruenzabbildungen genannt. Sie sind ein Spezialfall der sogenannten Ähnlichkeitsabbildungen, welche sämtlich Figuren in ähnliche Figuren überführen. Bei solchen Abbildungen bleiben also sämtliche Winkel
und Streckenverhältnisse erhalten. Bei einer Ähnlichkeitsabbildung geht eine Strecke AB in
eine Strecke A0 B 0 über, für die A0 B 0 = k ·AB gilt (für eine reelle Konstante k). Die einfachste
Ähnlichkeitsabbildung ist die Zentrische Streckung.
A'
C'
A
C
B'
A
C
Z
Z
B
B
C'
B'
A'
Abbildung 4.11: Zentrische Streckung
Gegeben sei nun ein Punkt Z und eine reelle Zahl k (welche auch negativ sein kann).
Eine Zentrische Streckung mit Streckungszentrum Z und Streckungsfaktor k wird dadurch
definiert, dass jeder Punkt P auf jenen Punkt P 0 auf ZP mit ZP 0 = k · ZP abgebildet wird.
Zentrische Streckungen haben die Eigenschaft, dass jede Strecke auf eine zu ihr parallele
Strecke abgebildet wird. Abgesehen von der Translation (welche als eine entartete zentrische
Streckung mit einem Streckungszentrum im Unendlichen“ betrachtet werden kann) charak”
terisiert diese Eigenschaft die zentrischen Streckungen unter allen Ähnlichkeitsabbildungen.
Es gilt also
Satz 4.5. Jede gleichsinnige Ähnlichkeitsabbildung, die keine Kongruenzabbildung ist und
alle Strecken auf parallele Strecken abbildet, ist eine zentrische Streckung.
Beweis. Wir betrachten eine gleichsinnige Ähnlichkeitsabbildung, die keine Kongruenzabbildung ist und alle Strecken auf parallele Strecken abbildet. Seien A, B, C verschiedene
Punkte und A0 , B 0 , C 0 deren Bilder. Dann sind die Strecken AB und A0 B 0 parallel und nicht
gleich lang, also sind die Geraden AA0 und BB 0 nicht parallel. Sei Z der Schnittpunkt von
AA0 und BB 0 . Ebenso sind die Strecken BC und B 0 C 0 parallel und nicht gleich lang, also
sind die Geraden BB 0 und CC 0 nicht parallel. Sei Z 0 der Schnittpunkt von BB 0 und CC 0 .
Da es sich um eine Ähnlichkeisabbildung handelt, sind die Dreiecke ABC und A0 B 0 C 0
75
ähnlich, daher gilt nach dem Strahlensatz (mit gerichteten Streckenverhältnissen)
ZB 0
A0 B 0
B0C 0
Z 0B0
=
=
= 0 ,
ZB
AB
BC
ZB
also muss Z = Z 0 gelten. Folglich gehen alle Geraden, die einen Punkt mit seinem Bildpunkt
verbinden, durch einen festen Punkt Z, womit es sich um eine zentrische Streckung handeln
muss.
Die klassische Anwendung einer zentrischen Streckung ist eine alte, aber berühmte Aufgabe:
Beispiel 4.4.1. Gegeben sei ein Dreieck ABC. Konstruiere das Quadrat P QRS, sodass P
und Q auf BC, R auf CA und S auf AB liegen.
A
S
B
B'
P
P'
R
Q
C
Q'
C'
Abbildung 4.12: Zeichnung zu Beispiel 4.4.1
Lösung. Wir errichten ein Quadrat P 0 Q0 CB über BC nach außen. Seien B 0 bzw C 0 die
Schnittpunkte von P 0 Q0 mit AB bzw AC. Die Dreiecke ABC und AB 0 C 0 sind sicherlich
zueinander ähnlich, somit ist A das Zentrum jener zentrischen Streckung, die das Dreieck
AB 0 C 0 in das Dreieck ABC überführt. Bei dieser zentrischen Streckung geht das Quadrat
P 0 Q0 CB in das gesuchte Quadrat P QRS über. Somit sind P und Q die Schnittpunkte von
BC mit AP 0 und AQ0 . Scheiden wir nun die Normalen auf BC durch P bzw Q mit AB bzw
AC, so erhalten wir die restlichen Quadrateckpunkte.
Beispiel 4.4.2. Zwei Kreise berühren einander von innen im Punkt A. Die Sehne M N des
größeren Kreises ist Tangente an den kleineren Kreis im Punkt P . Man beweise, dass die
Gerade AP eine Winkelsymmetrale des Winkels ∠M AN ist.
76
N
L
O1
A
O2
P
K
M
Abbildung 4.13: Zeichnung zu Beispiel 4.4.2
Lösung. Sei k1 der kleinere und k2 der größere der beiden Kreise und O1 bzw O2 jeweils die
Kreismittelpunkte. AM und AN schneiden k1 jeweils in K und L.
A ist Zentrum jener zentrischen Streckung, die k2 in k1 überführt. Bei dieser zentrischen
Streckung geht M in K und N in L über, daher sind KL und M N parallel. Wegen O1 P ⊥
M N gilt somit auch O1 P ⊥ KL und da KL aber eine Sehne von k1 ist, ist O1 P dessen
Streckensymmetrale. Nach dem Südpolsatz (Satz 1.8) ist somit AP die Winkelsymmetrale
von ∠M AN .
Beispiel 4.4.3 (IMO 1992). Sei C ein Kreis und L eine Tangente an C und M ein Punkt
auf L. Bestimme den geometrischen Ort jener Punkte P mit der folgenden Eigenschaft: Es
existieren zwei Punkte Q und R auf L, sodass M der Mittelpunkt von QR und C der Inkreis
von 4P QR ist.
Lösung. Sei N der Berührpunkt von C und L und X jener Punkt auf C, sodass N X ein
Durchmesser von C ist. Sei weiters C 0 der Ankreis von 4P QR, dessen Mittelpunkt dem
Punkt P gegenüberliegt und Y jener Punkt auf L, sodass M N = M Y gilt. Da M aber auch
der Mittelpunkt von QR ist, gilt QN = Y R. Nach Satz 1.5 und Satz 1.2 ist somit Y der
Ankreisberührpunkt von C 0 .
Da sowohl P Q als auch P R Tangenten an C und C 0 sind, ist P somit das Zentrum jener
zentrischen Streckung, die C in C 0 überführt. Bei dieser geht allerdings X in Y über. Somit
sind P , X und Y kollinear, d.h. P liegt auf der Gerade XY , welche allerdings eine feste
Gerade ist. Der gesuchte geometrische Ort ist somit der auf der Geraden XY verlaufende
Strahl mit Scheitel X, der Y nicht enthält.
77
P
X
C
L
Q
N
R
Y
M
C'
Ip
Abbildung 4.14: Zeichnung zu Beispiel 4.4.3 (IMO 1992)
4.5
Drehstreckung
Eine Drehstreckung ist eine Hintereinanderausführung einer zentrischen Streckung mit Zentrum Z und Faktor k und einer Drehung um Z um einen Winkel ϕ.
B'
B
A
A'
Z
Abbildung 4.15: Drehstreckung
Man kann sich nun fragen, was das Produkt einer Streckung und einer Rotation mit
verschiedenen Zentren ist oder was passiert, wenn noch eine Translation hinzukommt. Die
verblüffende und einfache Antwort lautet: eine Drehstreckung.
Satz 4.6. Jede gleichsinnige Ähnlichkeitsabbildung der Ebene ist entweder eine Translation
oder eine Drehstreckung.
Beweis. Wir betrachten eine gleichsinnige Ähnlichkeitsabbildung, welche die verschiedenen
Punkte A und B jeweils auf A0 und B 0 abbilde.
Nehmen wir zunächst an, die Strecken AB und A0 B 0 seien gleich lang, parallel und haben
dieselbe Orientierung. Sei C ein beliebiger Punkt, der nicht auf AB liegt, und C 0 dessen
Bild. Dann folgt aus der gleichsinnigen Ähnlichkeit, dass die Dreiecke ABC und A0 B 0 C 0
78
kongruent und entsprechende Seiten dieser Dreiecke parallel sind. Somit sind alle Strecken,
die einen Punkt mit seinem Bildpunkt verbinden, parallel und gleich lang, womit es sich bei
der Abbildung um eine Translation handelt.
Nehmen wir nun an, die Strecken AB und A0 B 0 seien nicht gleich lang oder nicht parallel.
Sei D der Schnittpunkt der Geraden AA0 und BB 0 (falls AA0 und BB 0 parallel sind, so können
wir A durch einen Punkt A1 , der nicht auf AB liegt, und A0 durch A01 , den Bildpunkt von A1 ,
ersetzen. Da es sich nicht um eine Translation handelt, sind BB 0 und A1 A01 nicht parallel)
und sei Z der zweite Schnittpunkt der Umkreise von 4ABD und 4A0 B 0 D (falls die Punkte
A, B, A0 , B 0 kein Viereck bilden, so können wir die Strecken AB und A0 B 0 durch eine
andere Strecke und ihre Bildstrecke ersetzen, sodass die vier Punkte ein Viereck bilden.
Liegt beispielsweise B 0 auf AA0 , so können wir A durch den Mittelpunkt von AB und A0
durch den Mittelpunkt von A0 B 0 ersetzen).
A'
B'
D
B
A
Z
Abbildung 4.16
Dann gilt (mit orientierten Winkeln modulo 180◦ )
]ZAB = ]ZDB = ]ZDB 0 = ]ZA0 B 0
und analog auch ]ABZ = ]A0 B 0 Z sowie ]BZA = ]B 0 Z 0 A0 , also sind 4ZAB und 4ZA0 B 0
gleichsinnig ähnlich. Somit führt die Drehstreckung mit Zentrum Z, Faktor k = ZA0 /ZA
und Winkel ϕ = ∠AZA0 das Dreieck ZAB in das Dreieck ZA0 B 0 über. Es folgt, dass jede
gleichsinnige Ähnlichkeitsabbildung, die keine Translation ist, einen Fixpunkt hat. Ferner ist
dieser Fixpunkt eindeutig, denn gäbe es zwei verschiedene Fixpunkte Z1 und Z2 , so gälte
k=
Z1 Z 2
=1
Z1 Z 2
womit die Abbildung eine Kongruenzabbildung wäre. Die einzige gleichsinnige Kongruenzabbildung, die zwei verschiedene Fixpunkte hat, ist aber die Identität, welche eine Translation
um den Nullvektor ist.
Beispiel 4.5.1 (Iran 2006). Sei ABC ein Dreieck und P , Q und R die Mittelpunkte von
BC, CA und AB. AP schneide RQ in E und den Umkreis von 4ABC in F . S und T seien
79
Punkte auf P Q und P R, sodass ES ⊥ P Q, ET ⊥ RP . F 0 sei jener Punkt auf dem Umkreis
von 4ABC, sodass F F 0 ein Durchmesser des Umkreises ist. AF 0 schneide BC in E 0 . S 0
und T 0 seien auf AB, AC, sodass E 0 S 0 ⊥ AB, E 0 T 0 ⊥ AC. Zeige, dass T S ⊥ T 0 S 0 .
T'
F'
A
E
R
Q
S
T
B
C
E'
P
S'
F
Abbildung 4.17: Zeichnung zu Beispiel 4.5.1 (Iran 2006)
Lösung. Sicherlich ist T ESP aufgrund zweier gegenüberliegender rechter Winkel ein Sehnenviereck, daher gilt
∠T 0 AS 0 = 180◦ − ∠CAB = 180◦ − ∠T P S = ∠T ES.
Da nach Thales E 0 A ⊥ AP und T 0 A ⊥ T 0 E 0 , gilt aufgrund von Normalwinkel
∠EST = ∠EP T = ∠P AQ = ∠AE 0 T 0 = ∠AS 0 T 0 .
Somit sind die Dreiecke AT 0 S 0 und ET S (gleichsinnig) ähnlich. Somit existiert eine Drehstreckung, die das Dreieck ET S in das Dreieck AT 0 S 0 überführt. Allerdings stehen die entsprechenden Seiten ES und AS 0 , sowie ET und AT 0 normal aufeinander, womit der Drehwinkel
der Drehstreckung ein rechter Winkel ist. Somit muss auch das dritte Paar entsprechender
Seiten, also T S und T 0 S 0 , normal aufeinander stehen.
Beispiel 4.5.2 (IMO Shortlist 2002). Es seien S1 und S2 Kreise, die sich in P und Q
schneiden. Seien A1 und B1 zwei verschiedene Punkte auf S1 (verschieden von P und Q).
Die Geraden A1 P und B1 P schneiden S2 in A2 und B2 . A1 B1 und A2 B2 schneiden sich
in C. Zeige dass, wenn sich A1 und B1 auf S1 bewegen, sich der Umkreismittelpunkt von
A1 A2 C auf einem fixen Kreis bewegt.
80
C
A1
O
P
B2
B1
A2
S1
S2
Q
Abbildung 4.18: Zeichnung zu Beispiel 4.5.2 (IMO Shortlist 2002)
Lösung. Es gilt
∠B1 A1 Q = ∠B1 P Q = 180◦ − ∠B2 P Q = ∠CA2 Q
womit CA1 QA2 ein Sehnenviereck ist. Somit können wir die Bi ignorieren, denn wir suchen
die Ortslinie von O, dem Umkreismittelpunkt von 4A1 A2 Q.
Da die Winkel ∠P A1 Q und ∠P A2 Q konstant sind, sind alle Dreiecke A1 A2 Q zueinander ähnlich. Somit sind auch alle Dreiecke A1 OQ zueinander ähnlich, d.h. das Verhältnis
QA1 /QO ist konstant. Betrachten wir nun die Drehstreckung mit Zentrum Q, die A1 auf O
abbildet, so liegt O auf jenem Kreis, der aus S1 ensteht. Dieser ist allerdings aufgrund des
konstanten Streckungsfaktors QA1 /QO und des konstanten Drehwinkels ∠A1 QO ein fester
Kreis.
Beispiel 4.5.3 (IMO Shortlist 2001). Sei ABC ein spitzwinkliges Dreieck und A1 der Mittelpunkt des Quadrats, das zwei Eckpunkte auf BC und je einen auf AB und AC hat. Ebenso
sind B1 und C1 die Mittelpunkte der Quadrate mit zwei Eckpunkten auf AC bzw. AB und je
einem auf den anderen beiden Seiten. Zeige, dass sich AA1 , BB1 und CC1 in einem Punkt
schneiden.
Lösung. Wir errichten nach außen über den Seiten BC, CA und AB die Quadrate BCX2 X1 ,
CAY2 Y1 und ABZ2 Z1 mit den Mittelpunkten X, Y und Z. Eine zentrische Streckung mit
Zentrum A führt das Quadrat mit zwei Punkten auf BC und je einen Punkt auf AB und
AC in das Quadrat BCX2 X1 und somit A1 in X über (siehe Beispiel 4.4.1). Ebenfalls führt
eine Zentrische Streckung mit Zentrum B bzw. C die Punkte B1 bzw. C1 in die Punkte Y
und Z über. Es genügt also zu zeigen, dass sich AX, BY und √
CZ in einem Punkt schneiden.
◦
Eine Drehstreckung um 45 mit Zentrum A und Faktor 2 führt das Dreieck AY Z in
das Dreieck AY2 B und somit Y Z in Y2 B über, daher schließen Y Z und Y2 B einen Winkel
von 45◦ ein.
81
Z1
Y2
A
Z
Z2
Y
Y1
C
B
X
X1
X2
Abbildung 4.19: Zeichnung zu Beispiel 4.5.3 (IMO Shortlist 2001)
√
Eine Drehstreckung um 45◦ mit Zentrum C und Faktor 1/ 2 führt das Dreieck CY2 B in
das Dreieck CAX und somit Y2 B in AX über, daher schließen Y2 B und AX einen Winkel
von 45◦ ein.
Somit entsteht AX aus Y Z durch zweimaliges Drehstrecken um jeweils 45◦ (jeweils gegen
den Uhrzeigersinn), daher schließen AX und Y Z einen rechten Winkel ein, d.h. AX ⊥ Y Z.
Analog gilt auch BY ⊥ ZX und CZ ⊥ XY . Somit schneiden sich AX, BY und CZ in
einem Punkt, nämlich dem Höhenschnittpunkt des Dreiecks XY Z.
82
Kapitel 5
Inversive Geometrie
5.1
Inversion
Die Inversion oder Spiegelung an einem Kreis ist ein mächtiges Werkzeug, sie ist eine geometrische Transformation, welche wie folgt definiert ist:
O
X
X'
Abbildung 5.1: Kreisinversion
Sei k ein Kreis mit Mittelpunkt O und Radius r. Ein Punkt X 6= O wird auf jenen Punkt
X 0 auf OX abgebildet, für den
OX · OX 0 = r2
gilt, wobei die Längen gerichtet zu nehmen sind (d.h. X und X 0 liegen auf derselben Seite
von O).
Offensichtlich wird X 0 wieder auf X abgebildet.
Wie man sieht, ist diese Abbildung für X = O nicht definiert. Wir beobachten aber,
dass sich X 0 umso weiter von O entfernt, je mehr sich X dem Punkt O nähert. Aus diesem
Grund erweitern wir die Euklidische Ebene um einen zusätzlichen Punkt, den sogenannten
83
Fernpunkt O∞ . Dieser kann als ein Punkt betrachtet werden, der unendlich weit entfernt ist.
Er liegt zusätzlich noch auf jeder Gerade.
Nun können wir zusätzlich definieren, dass O bei der Kreisinversion auf O∞ und O∞ auf
O abgebildet wird.
Alternativ kann man X 0 (für X verschieden von O und O∞ ) auch folgendermaßen konstruieren:
Falls X außerhalb von k liegt, so lege man die Tangenten von X an k. Die Verbindungsgerade der Tangentenberührpunkte schneiden OX dann in X 0 . Liegt X innerhalb von k,
so errichte man die Normale auf OX durch X. Die Tangenten in den Schnittpunkten der
Normalen mit dem Kreis schneiden sich in X 0 .
Diese Konstruktionsvorschrift folgt unmittelbar aus dem Kathetensatz.
5.2
Eigenschaften der Inversion
Um die Kreisinversion produktiv anwenden zu können, ist eine Reihe von Sätzen notwendig.
Im Folgenden betrachten wir eine Kreisinversion an einem Kreis mit Zentrum O und Radius
r. Wir verwenden dabei die Notation und einige Ergebnisse für orientierte Winkel modulo
180◦ (siehe Anhang A).
Satz 5.1. Wird ein Punkt A auf A0 und ein Punkt B auf B 0 abgebildet, so sind die Dreiecke
OAB und OB 0 A0 gegensinnig ähnlich mit einem Ähnlichkeitsfaktor r2 /(OA · OB) = OA0 ·
OB 0 /r2 . Insbesondere gilt ]OAB = −]OB 0 A0 sowie
A0 B 0 =
AB · r2
.
OA · OB
B'
B
B'
O
B
A'
O
A
Abbildung 5.2
Satz 5.2. Eine Gerade, die durch O geht, wird auf sich selbst abgebildet.
84
A'
A
Satz 5.3. Eine Gerade g, die nicht durch O geht, wird auf einen Kreis k abgebildet, der
durch O geht. Ebenso wird ein Kreis k, der durch O geht, auf eine Gerade g abgebildet, die
nicht durch O geht. Die Tangente in O an k ist dabei zu g parallel.
A'
A
A'
A
L'
O
L
L
O
L'
g
k
k
g
Abbildung 5.3
Beweis. Sei g eine Gerade, die nicht durch O geht und sei L die Normalprojektion von O
auf g. Sei weiters A ein beliebiger Punkt auf g. L0 und A0 seien die Abbilder von A und
L durch die Inversion. Nach Satz 5.1 gilt ∠OA0 L0 = 90◦ , womit A0 auf dem Kreis k mit
Durchmesser OL0 liegt. Umgekehrt wird allerdings jedes A0 auf k auf ein entsprechendes A
auf g abgebildet (O wird auf O∞ ∈ g abgebildet), womit g tatsächlich in den gesamten Kreis
übergeht. Da OL0 ⊥ g, ist die Tangente in O an k zu g parallel.
Die Umkehrung erfolgt analog.
Satz 5.4. Ein Kreis, der nicht durch O geht, wird auf einen Kreis abgebildet, der nicht durch
O geht.
C
D
D'
C'
O
B
B'
A'
A
Abbildung 5.4
85
Beweis. Es seien A, B, C, D vier Punkte, die auf einem Kreis liegen, und es seien A0 , B 0 , C 0 ,
D0 deren Abbilder durch die Inversion. Verwenden wir nun Satz 5.1, so gilt (mit orientierten
Winkeln modulo 180◦ )
]A0 B 0 C 0 = ]A0 B 0 O + ]OB 0 C 0
= ]OAB + ]BCO
= ]OAD + ]DAB + ]BCD + ]DCO
= ]OAD + ]DCO
= ]A0 D0 O + ]OD0 C 0
= ]A0 D0 C 0
womit die Punkte A0 , B 0 , C 0 , D0 ebenfalls auf einem Kreis liegen. Dieser kann nach Satz 5.3
nicht durch O gehen, da er sonst auf auf eine Gerade abgebildet würde und nicht auf den
Kreis durch A, B, C und D.
Aus Symmetriegründen folgt weiters
Satz 5.5. Wird bei einer Inversion der Kreis k auf einen Kreis k 0 abgebildet, so sind das
Inversionszentrum O und die Mittelpunkte von k und k 0 kollinear. Vorsicht: Der Mittelpunkt
von k wird NIEMALS auf den Mittelpunkt von k 0 abgebildet!
In der Literatur ist es üblich, bei der Inversion eine Gerade als einen speziellen Kreis zu
betrachten. Wir fassen diese beiden Begriffe zum Begriff verallgemeinerter Kreis zusammen.
Wir sagen nun, dass sich zwei verallgemeinerte Kreise (also Kreis oder Gerade) schneiden, berühren oder nicht schneiden, je nachdem, ob sie zwei, einen oder keinen gemeinsamen
Punkt haben (dabei wird O∞ mitgezählt). Ist nun P ein gemeinsamer Punkt zweier verallgemeinerter Kreise k1 und k2 , so ist natürlich P 0 auch ein gemeinsamer Punkt der Bildkreise
k10 und k20 .
Satz 5.6. Es seien k1 und k2 zwei verallgemeinerte Kreise, die bei der Inversion in k10 und
k20 übergehen. Dann gilt
• Schneiden sich k1 und k2 , so schneiden sich auch k10 und k20 .
• Berühren sich k1 und k2 , so berühren sich auch k10 und k20 .
• Haben k1 und k2 keinen gemeinsamen Punkt, so haben auch k10 und k20 keinen gemeinsamen Punkt.
Zum letzten Punkt sollte noch bemerkt werden, dass zwei Geraden immer einen gemeinsamen Punkt, nämlich O∞ haben. Zwei parallele Geraden werden daher auf zwei verallgemeinerte Kreise abgebildet, die sich in O berühren.
Von der Tatsache, dass verallgemeinerte Kreise bei der Inversion erhalten bleiben, ist es
nur ein kleiner Schritt zur Erkenntnis, dass auch Winkel erhalten bleiben.
86
Doch zunächst müssen wir noch den Schnittwinkel zweier einander schneidender verallgemeinerter Kreise definieren: Der Schnittwinkel zweier Kreise k1 und k2 ist jener Winkel
θ, den die Tangenten der beiden Kreise in einem der Schnittpunkte miteinander einschließen. Aus Symmetriegründen sind die Schnittwinkel der Tangenten in beiden Schnittpunkten
gleich. Der Schnittwinkel zwischen einer Gerade g und einem Kreis k ist jener Winkel θ,
den die Tangente an k in einem der Schnittpunkte mit g einschließt. Auch hier sind aus
Symmetriegründen die Schnittwinkel in beiden Schnittpunkten gleich.
P
g2
θ
g1
θ
P'
g2'
O
θ
g1'
Abbildung 5.5: Winkeltreue bei der Inversion
Um zu sehen, wie Winkel von der Inversion beeinflusst werden, betrachten wir zwei
Geraden g1 und g2 , die nicht durch O gehen und sich in P unter einem Winkel θ schneiden.
Diese werden bei der Inversion auf die Kreise g10 und g20 abgebildet, die sich in O und P 0
schneiden (P 0 ist das Bild von P unter der Inversion). Nach Satz 5.3 ist die Tangente in O
an g10 zu g1 parallel und die Tangente in O an g20 zu g2 parallel. Somit ist θ der Schnittwinkel
der Tangenten der beiden Kreise in O und dieser Winkel tritt auch als Schnittwinkel bei P 0
auf.
Falls eine der Geraden durch O geht, so wird diese auf sich selbst und die andere Gerade
auf einen Kreis, der durch O geht und dessen Tangente in O zur ursprünglichen Geraden
parallel ist, abgebildet. Somit bleibt auch hier der Schnittwinkel erhalten. Falls beide Geraden
durch O gehen, also P = O gilt, so werden beide auf sich selbst abgebildet, wodurch der
Winkel selbstverständlich gleich bleibt.
Um zu sehen, wie sich eine Inversion auf den Schnittwinkel zweier sich schneidender
Kreise auswirkt, können wir annehmen, dass sie sich in P schneiden und g1 und g2 jeweils
ihre Tangenten sind. Dann berühren die Bildkreise g10 und g20 in P 0 , wodurch sie jeweils mit g10
und g20 eine gemeinsame Tangente haben. Somit bleibt auch hier der Schnittwinkel erhalten.
87
Wir fassen zusammen:
Satz 5.7. Ist der Schnittwinkel zweier (verallgemeinerter) Kreise θ, so schneiden sich deren
Bilder ebenfalls unter einem Winkel von θ.
5.3
Anwendungen der Inversion
Beispiel 5.3.1. Gegeben seien ein Punkt A und zwei Kreise k1 und k2 . Man konstruiere
einen dritten Kreis k3 , der durch A geht und k1 und k2 berührt.
k3
k1
O2
O1
A
k2
Abbildung 5.6: Zeichnung zu Beispiel 5.3.1
Lösung. Wir betrachten eine Inversion mit Zentrum A und beliebigem Radius. Diese bilde
k1 auf k10 und k2 auf k20 ab. Wir konstruieren eine gemeinsame Tangente l von k10 und k20 . l
geht bei dieser Inversion in einen Kreis k3 über, der durch A geht. Da aber l die Kreise k10
und k20 berührt, muss dessen Abbild, also k3 , auch die Abbilder dieser beiden Kreise, also k1
und k2 , berühren.
Beispiel 5.3.2. Gegeben seien zwei Punkte A und B sowie ein Kreis k1 . Man konstruiere
einen weiteren Kreis k2 , sodass k2 durch A und B geht und k1 berührt.
A
k1
k2
B
O2
O1
Abbildung 5.7: Zeichnung zu Beispiel 5.3.2
88
Lösung. Wir betrachten eine Inversion mit Zentrum A und beliebigem Radius. Diese Bilde
B auf B 0 und k1 auf k10 ab. Wir konstruieren eine Tangente l von B 0 an k10 . k2 sei das Abbild
von l durch diese Inversion. Sicherlich geht k2 durch A und B und da aber l den Kreis k10
berührt, muss das Abbild von l, also k2 , das Abbild von k10 , also k1 , berühren.
Beispiel 5.3.3 (IMO 1996). Sei P ein Punkt innerhalb des Dreiecks 4ABC, sodass ∠AP B−
∠ACB = ∠AP C − ∠ABC. Die Punkte D und E seien die Inkreismittelpunkte der Dreiecke AP B und AP C. Zeige, dass die Geraden AP , BD und CE einen gemeinsamen Punkt
haben.
C
P
C'
A
P'
B
B'
Abbildung 5.8: Zeichnung zu Beispiel 5.3.3 (IMO 1996)
Lösung 2. 1 In jedem Dreieck teilt die Innenwinkelsymmetrale die gegenüberliegende Seite
im Verhältnis der anliegenden Seiten. Wollen wir also zeigen, dass sich BD und CE auf AP
schneiden, sie also AP im gleichen Verhältnis teilen, so genügt es zu zeigen, dass AB/BP =
AC/CP gilt.
Wir betrachten eine Inversion mit Zentrum A und beliebigem Radius. Dabei gehe B in
B 0 , C in C 0 und P in P 0 über. Die Dreiecke AP B und AB 0 P 0 sowie AP C und AC 0 P 0 sind
jeweils ähnlich. Somit gilt
∠C 0 B 0 P 0 = ∠AB 0 P 0 − ∠AB 0 C 0
= ∠AP B − ∠ACB
= ∠AP C − ∠ABC
= ∠AC 0 P 0 − ∠AC 0 B 0
= ∠B 0 C 0 P 0
1
Eine alternative Lösung dieser Aufgabe befindet sich auf Seite 43.
89
womit das Dreieck B 0 P 0 C 0 gleichschenklig mit P 0 B 0 = P 0 C 0 ist. Somit gilt wiederum aufgrund
der Ähnlichkeit der Dreiecke AP B und AB 0 P 0 bzw. AP C und AC 0 P 0
AP 0
AP 0
AC
AB
= 0 0 = 0 0 =
.
BP
PB
PC
CP
Beispiel 5.3.4 (IMO Longlist 1994). Der Inkreis des Dreiecks ABC berühre BC, CA, AB
in D, E, F . X sei ein Punkt im Inneren des Dreiecks ABC sodass der Inkreis des Dreiecks
XBC die Seite BC in D sowie CX in Y und XB in Z berührt. Zeige, dass EF ZY ein
Sehnenviereck ist.
Z'
Y'
g2
k3
k4
A
F'
E'
g1
k2
k1
X
F
Z
B'
B
Y
E
C
D
C'
Abbildung 5.9: Zeichnung zu Beispiel 5.3.4 (IMO Longlist 1994)
Lösung. Wir betrachten eine Inversion mit Zentrum D und beliebigem Radius. Diese führe
den Inkreis von 4ABC in die Gerade g1 , den Inkreis von 4XBC in die Gerade g2 , B in B 0 ,
C in C 0 , E in E 0 , F in F 0 , Y in Y 0 , Z in Z 0 , AB in den Kreis k1 , AC in den Kreis k2 , XB
in den Kreis k3 und XC in den Kreis k4 über.
Nach Satz 5.2 liegen B 0 und C 0 auf der Geraden BC, welche in sich selbst übergeführt
wird. Des Weiteren gilt nach Satz 5.3 g1 k g2 k BC.
Der Kreis k1 ist aus einer Geraden entstanden und somit muss er durch das Inversionszentrum D gehen. Des Weiteren geht k1 durch B 0 , da die ursprüngliche Gerade AB durch
B ging, und berührt g1 in F 0 , da die ursprüngliche Gerade AB den Inkreis von 4ABC in F
berührte.
90
Ebenso geht k3 durch D, da k3 aus einer Geraden entstanden ist, und durch B 0 , da die
usprüngliche Gerade XB durch B ging, und sie berührt g2 in Z 0 , da die ursprüngliche Gerade
XB den Inkreis von 4XBC in Z berührte.
Die Kreise k1 und k3 haben die Punkte D und B 0 gemeinsam und sind somit bezüglich der
Streckensymmetralen von DB 0 symmetrisch. Da diese Kreise aber jeweils zu DB 0 parallele
Tangenten in F 0 und Z 0 haben, müssen auch F 0 und Z 0 auf der Streckensymmetrale von DB 0
liegen. Somit gilt F 0 Z 0 ⊥ g1 und F 0 Z 0 ⊥ g2 . Analog gilt auch E 0 Y 0 ⊥ g1 und E 0 Y 0 ⊥ g2 .
Somit ist E 0 F 0 Z 0 Y 0 ein Rechteck. Da es aber somit einen Kreis durch E 0 , F 0 , Z 0 , Y 0 gibt,
muss es auch einen Kreis oder eine Gerade durch E, F , Z, Y geben. Eine Gerade ist aber
unmöglich.
A
k
H
k'
S
H'
P
P'
A'
Q'
C
B
B'
Q
Abbildung 5.10: Zeichnung zu Beispiel 5.3.5 (Israel 1995)
Beispiel 5.3.5 (Israel 1995). Sei P Q der Durchmesser eines Halbkreises H. Ein Kreis k
berühre H von innen und berühre P Q in C. Seien A ein Punkt auf H und B ein Punkt auf
P Q sodass AB normal auf P Q steht und eine Tangente an k ist. Zeige, dass AC den Winkel
∠P AB halbiert.
Lösung. Wir betrachten eine Inversion mit Zentrum C und beliebigem Radius. Bei dieser
gehen die Punkte A, B, P , Q jeweils in A0 , B 0 , P 0 , Q0 über.
Da der Kreis k durch das Inversionszentrum C geht und P Q berührt, geht k in eine
Gerade k 0 über, die zu P Q parallel ist. Der Halbkreis H geht in den Halbkreis H 0 mit
Durchmesser P 0 Q0 über, welcher nach Satz 5.6 k 0 berührt. Die Gerade AB geht in einen
Kreis S über, der durch C, A0 , B 0 geht und nach Satz 5.6 auch k 0 berührt.
Sowohl H 0 als auch S haben ihren Durchmesser auf P Q und sie haben eine gemeinsame
Tangente k 0 , die zu P Q parallel ist. Somit sind die Kreise H 0 und S kongruent und daher
sind sie bezüglich der Streckensymmetralen von CQ0 , also der Normalen zu P Q durch A0 ,
symmetrisch. Somit gilt
∠A0 P 0 B 0 = ∠A0 B 0 P 0
und daher
91
∠CP 0 A0 = ∠CB 0 A0 .
Nach Satz 5.1 gilt allerdings ∠CAP = ∠CP 0 A0 und ∠CAB = ∠CB 0 A0 und daher ∠CAP =
∠CAB.
Beispiel 5.3.6 (Iran 1995). Sei ABC ein Dreieck mit Inkreismittelpunkt I und Umkreismittelpunkt O. Der Inkreis des Dreiecks ABC berühre die Seiten BC, CA und AB jeweils
in M , N und P . Zeige, dass I, O und der Höhenschnittpunkt des Dreiecks M N P auf einer
Geraden liegen.
A
P
A'
N
O
I
F
B'
B
M
H
C'
C
Abbildung 5.11: Zeichnung zu Beispiel 5.3.6 (Iran 1995)
Lösung. Der Inkreismittelpunkt I des Dreiecks ABC ist der Umkreismittelpunkt des Dreiecks M N P , daher ist die Gerade durch I und den Höhenschnittpunkt von 4M N P die
Eulersche Gerade des Dreiecks M N P .
Betrachten wir nun eine Kreisinversion am Inkreis von 4ABC. Bei dieser gehen die
Eckpunkte A, B und C in die Mittelpunkte A0 , B 0 und C 0 der Strecken N P , P M und M N
über. Somit geht der Umkreis des Dreiecks ABC in den Umkreis des Dreiecks A0 B 0 C 0 über.
Das Dreieck A0 B 0 C 0 ist aber das Mittendreieck des Dreiecks M N P , somit ist der Umkreis von
4A0 B 0 C 0 der Feuerbachkreis von 4M N P . Dessen Mittelpunkt liegt nach Satz 3.6 auch auf
der Eulerschen Gerade des Dreiecks M N P . Somit liegt auch O auf der Eulerschen Geraden
von 4M N P , denn der Mittelpunkt eines Kreises, der Mittelpunkt des Bildkreises und das
Inversionszentrum sind kollinear.
Beispiel 5.3.7 (Russland 1995). Gegeben sei ein Halbkreis mit Durchmesser AB und Mittelpunkt O. Eine Gerade schneide den Halbkreis in C und D und die Gerade AB in M (mit
M B < M A und M D < M C). Sei K der zweite Schnittpunkt der Umkreise der Dreiecke
AOC und DOB. Zeige, dass ∠M KO ein rechter Winkel ist.
92
P
C
K
A
D
O
M'
B
M
Abbildung 5.12: Zeichnung zu Beispiel 5.3.7 (Russland 1995)
Lösung. Die Geraden AC, BD und OK schneiden sich in einem Punkt P , dem Potenzzentrum der Umkreise von 4OAC, 4OBD und ABDC.
Betrachten wir die Inversion mit Zentrum O und Radius OA. Bei dieser gehen die Punkte
A, B, C, D in sich selbst, der Umkreis von 4OAC in AC und der Umkreis von 4OBD in
BD über. M gehe in den Punkt M 0 über.
K ist der zweite Schnittpunkt der Umkreise von 4OAC und 4OBD, deshalb ist das
Bild von K der Schnittpunkt der Bilder dieser Kreise, also der Schnittpunkt von AC und
BD, also P . Nach Satz 5.1 gilt somit ∠M KO = 90◦ genau dann, wenn ∠P M 0 O = 90◦ .
Die Gerade CD geht bei dieser Inversion in den Kreis k durch O, C und D über. Dieser
Kreis ist aber der Feuerbachkreis des Dreiecks ABP , denn C und D sind die Höhenfußpunkte
von A auf BP bzw B auf AP (Thaleskreis über AB) und O ist per Definition der Halbierungspunkt von AB. Da M aber auf AB und CD liegt, muss dessen Abbild M 0 auch auf
deren Abbilder, also AB und k liegen. Somit ist M 0 der zweite Schnittpunkt von k und AB.
Da aber k der Feuerbachkreis des Dreiecks ABP ist, ist dieser Punkt der Höhenfußpunkt
von P auf AB. Somit gilt ∠P M 0 O = 90◦ .
93
Kapitel 6
Geometrische Ungleichungen
6.1
Dreiecksungleichung
Eine der grundlegendsten und wichtigsten geometrischen Ungleichungen ist die sogenannte
Dreiecksungleichung, die höchstwahrscheinlich bereits bekannt ist:
Satz 6.1. Für drei Punkte A, B, C gilt
AC + BC ≥ AB
mit Gleichheit genau dann, wenn C auf der Strecke AB liegt.
Ebenfalls bekannt sollte die Hypotenusen-Ungleichung sein:
Satz 6.2. Ist ∠ABC ein rechter Winkel, so gilt AC > BC.
Obwohl diese Ungleichungen trivial sind, lassen sich doch eine Fülle von geometrischen
Ungleichungen mit diesen einfachen Ideen beweisen.
D
C
E
B
A
Abbildung 6.1: Zeichnung zu Beispiel 6.1.1 (Kürschák 1954)
Beispiel 6.1.1 (Kürschák 1954). Sei ABCD ein konvexes Viereck mit AB + BD ≤ AC +
CD. Zeige, dass AB < AC gilt.
94
Beweis. Sei E der Schnittunkt von AC und BD. Nach der Dreiecksungleichung gilt AE +
BE > AB und CE+DE > CD und daher (AE+CE)+(BE+DE) = AC+BD > AB+CD.
Somit gilt AC − AB > CD − BD ≥ AB − AC und daher AC > AB.
Beispiel 6.1.2. Sei ABCD ein konvexes Viereck und seien M und N die Mittelpunkte von
AD und BC. Zeige, dass
AB + CD
≥ MN
2
gilt, wobei Gleichheit genau dann gilt, wenn AB und CD parallel sind.
D
C
M
N
P
A
B
Abbildung 6.2: Zeichnung zu Beispiel 6.1.2
Lösung. Sei P der Mittelpunkt der Diagonalen AC. Somit sind M P bzw P N parallel zu
CD bzw AB und es gilt M P = CD/2 sowie N P = AB/2. Somit gilt nach der Dreiecksungleichung
AB + CD
= MP + NP ≥ MN
2
mit Gleichheit genau dann, wenn P auf der Strecke M N liegt, wenn also die Geraden
M P, N P und M N zusammenfallen. Wegen M P k CD und N P k AB ist dies genau dann
der Fall, wenn AB k CD.
Beispiel 6.1.3. Zeige, dass in jedem Dreieck die Summe der Höhen kleiner ist als der
Dreiecksumfang.
Lösung. Sei ABC ein Dreieck mit Höhenfußpunkten Ha , Hb , Hc (mit Ha auf BC, Hb auf
CA und Hc auf AB). Dann ist 4ABHa ein rechtwinkeliges Dreieck mit Hypotenuse AB.
Somit gilt AB > AHa und analog auch BC > BHb und CA > CHc . Aufsummieren dieser
Ungleichungen liefert die zu zeigende Ungleichung.
95
6.2
Algebraische Techniken
Einige (oftmals nur scheinbar) geometrische Ungleichungen lassen sich schnell auf algebraische Ungleichungen zurückführen bzw. sind oft algebraisch lösbar, ohne dass viel geometrische Arbeit vonnöten ist.
Oftmals ist bei Aufgaben eine Ungleichung für die postitiven reellen Zahlen a, b, c zu zeigen, wobei a, b, c die Seitenlängen eines Dreiecks sind. Kommt außer a, b, c keine andere Größe
in der Ungleichung vor, so haben diese Ungleichungen oft wenig bis keinen geometrischen
Hintergund. Man kann dann, um die Geometrie bei einer solchen Ungleichung endgültig zu
entfernen, die sogenannte Ravi-Substitution durchführen. Dabei substituiert man a = x + y,
b = y + z, c = z + x, wobei x, y, z positive Zahlen sind (wir wählen x = (c + a − b)/2,
y = (a + b − c)/2, z = (b + c − a)/2, welche nach der Dreiecksungleichung sicherlich positiv
sind). Kommen neben den Seitenlängen a, b, c auch noch andere Größen wie Flächeninhalt,
Inkreisradius, Umkreisradius, etc vor, so kann man versuchen, diese Größen durch die Seitenlängen auszudrücken. Oftmals gibt es Formeln für die zusätzliche Größen, um diese aus
den Seitenlängen auszurechnen. Hier die geläufigsten (die meisten sind wahrscheinlich bekannt):
Satz 6.3. Sei ABC ein Dreieck mit den Seitenlängen a, b, c, dem Flächeninhalt F , dem
Umkreisradius R und dem Inkreisradius r. Sei weiters s = (a + b + c)/2 der halbe Umfang
des Dreiecks. Dann gilt
p
• (Heronsche Flächenformel) F = s(s − a)(s − b)(s − c)
• R=
abc
4F
• r=
F
(vergleiche Satz 1.3).
s
Ebenfalls bekannt sollten folgende Ungleichungen sein:
• (Arithmetisch-geometrische Mittelungleichung) Für alle nichtnegativen reellen Zahlen
x1 , . . . , xn gilt
√
x1 + . . . + xn
≥ n x1 · . . . · xn .
n
• (Cauchy-Schwarz-Ungleichung) Für alle reellen Zahlen x1 , . . . , xn und y1 , . . . , yn gilt
(x21 + . . . + x2n )(y12 + . . . + yn2 ) ≥ (x1 y1 + . . . + xn yn )2 .
• (Schur-Ungleichung) Für alle reellen Zahlen r und alle positiven reellen Zahlen x, y, z
gilt
xr (x − y)(x − z) + y r (y − z)(y − x) + z r (z − x)(z − y) ≥ 0.
Übrigens kann man für r = 1 die Schur-Ungleichung auch folgendermaßen schreiben:
96
• xyz ≥ (x + y − z)(y + z − x)(z + x − y)
• x3 + y 3 + z 3 + 3xyz ≥ x2 y + x2 z + y 2 z + y 2 x + z 2 x + z 2 y.
Beispiel 6.2.1 (Ungleichung von Euler). Seien R der Umkreisradius und r der Inkreisradius
eines Dreiecks. Man beweise die Ungleichung
R ≥ 2r.
Lösung. Seien a, b, c die Seitenlängen und F der Flächeninhaltpdes Dreiecks und sei s =
(a + b + c)/2. Dann gilt R = abc/(4F ), r = F/s sowie F = s(s − a)(s − b)s − c). Die
Ungleichung schreibt sich nun als
F
abc
≥2
4F
s
F2
⇔ abc ≥ 8
s
⇔ abc ≥ 8(s − a)(s − b)(s − c).
Nun substituieren wir x = s − a, y = s − b, z = s − c mit x, y, z > 0 (also die RaviSubstitution). Dann schreibt sich die Ungleichung als
⇔
(x + y)(y + z)(z + x) ≥ 8xyz
x+y
y+z
z+x
≥ xyz
2
2
2
und diese Ungleichung folgt nun aus der arithmetisch-geometrischen Mittelungleichung.
Beispiel 6.2.2. Seien α, β, γ die Winkel eines Dreiecks. Zeige folgende Ungleichung:
3
cos α + cos β + cos γ ≤ .
2
Lösung. Es seien a, b, c die Seiten dieses Dreiecks. Nach dem Cosinussatz gilt
a2 = b2 + c2 − 2bc cos α
⇔
cos α =
b 2 + c 2 − a2
.
2bc
Die Ungleichung schreibt sich somit als
b 2 + c 2 − a2 c 2 + a2 − b 2 a2 + b 2 − c 2
3
+
+
≤
2bc
2ca
2ab
2
⇔ a(b2 + c2 − a2 ) + b(c2 + a2 − b2 ) + c(a2 + b2 − c2 ) ≤ 3abc
⇔ a2 b + a2 c + b2 c + b2 a + c2 a + c2 b ≤ a3 + b3 + c3 + 3abc
und dies ist die Schur-Ungleichung.
97
Beispiel 6.2.3 (Weitzenböck-Ungleichung). Seien a, b, c die Seitenlängen eines Dreiecks mit
Flächeninhalt F . Dann gilt
√
a2 + b2 + c2 ≥ 4 3F.
Lösung. Wir erhalten unter Verwendung der Heronschen Flächenformel sowie der arithmetischgeometrischen Mittelungleichung
3
a+b+c
2
16F = (a + b + c)(a + b − c)(b + c − a)(c + a − b) ≤ (a + b + c)
.
3
Wir erhalten somit unter Verwendung der arithmetisch-quadratischen Mittelungleichung
2
√ a2 + b 2 + c 2
(a + b + c)2 √
a+b+c
√
4F ≤
= 3·
.
≤ 3·
3
3
3· 3
Eine etwas stärkere Ungleichung ist die Hadwiger-Finsler-Ungleichung:
Beispiel 6.2.4 (Hadwiger-Finsler-Ungleichung). Seien a, b, c die Seitenlängen eines Dreiecks
mit Flächeninhalt F . Dann gilt
√
a2 + b2 + c2 ≥ 4 3F + (a − b)2 + (b − c)2 + (c − a)2 .
Lösung. Es gilt a2 − (b − c)2 = (c + a − b)(a + b − c). Substituieren wir nun a = x + y,
b = y + z, c = z + x mit x, y, z > 0, so schreibt sich die Ungleichung mithilfe der Heron’schen
Flächenformel als
p
xy + yz + zx ≥ 3(x + y + z)xyz
⇔ (xy + yz + zx)2 ≥ 3(x + y + z)xyz
⇔ (xy + yz + zx)2 − 3(x + y + z)xyz ≥ 0
(xy − yz)2 + (yz − zx)2 + (zx − xy)2
⇔
≥ 0.
2
Manchmal kann man eine Ungleichung auch schnell beweisen, indem man Streckenverhältnisse verwendet. Vor allem, wenn Flächeninhalte ins Spiel kommen, kann dies von
großem Vorteil sein. Der folgende Satz ist zwar eigentlich trivial, aber dennoch häufig nützlich.
Satz 6.4. Seien ABC und XY Z Dreiecke mit ∠CAB = ∠ZXY . Sei weiters λ1 = XY /AB
und λ2 = ZX/CA. Dann gilt [XY Z] = λ1 λ2 · [ABC].
Sind insbesondere 4ABC und 4XY Z ähnlich mit λ = XY /AB = Y Z/BC = ZX/CA,
so gilt [XY Z] = λ2 [ABC].
Beweis. Es gilt
[XY Z] =
XY · ZX · sin ∠ZXY
λ1 AB · λ2 CA · sin ∠CAB
=
= λ1 λ2 · [ABC].
2
2
98
A
B2
C1
C2
B
P
A1
B1
A2
C
Abbildung 6.3: Zeichnung zu Beispiel 6.2.5 (Mediterranean MC 2006)
Beispiel 6.2.5 (Mediterranean MC 2006). Sei P ein innerer Punkt des Dreiecks ABC und
seien A1 B2 , B1 C2 und C1 A2 der Reihe nach die parallelen Geraden zu den Seiten AB, BC
und CA durch den Punkt P , wobei A1 und A2 auf BC, B1 und B2 auf CA und C1 und C2
auf AB liegen. Zeige, dass [A1 A2 B1 B2 C1 C2 ] ≥ 32 [ABC] gilt, wobei [. . .] jeweils die Fläche
der Figur angibt.
Lösung. Seien a, b, c die Seitenlängen und F der Flächeninhalt des Dreiecks ABC. Wegen
A1 B2 k AB, B1 C2 k BC und C1 A2 k CA sind die Dreiecke P A1 A2 , P B1 B2 und P C1 C2 zum
Dreieck ABC ählich. Seien nun λ1 , λ2 , λ3 die Ähnlichkeitsfaktoren der Dreiecke P A1 A2 ,
P B1 B2 und P C1 C2 zum Dreieck ABC, d.h. es gelte
A1 A2 = λ1 a,
B1 B2 = λ2 b,
C1 C2 = λ3 c,
A2 P = λ1 b,
B2 P = λ2 c,
C2 P = λ3 a,
P A1 = λ1 c,
P B1 = λ2 a,
P C1 = λ3 b.
Allerdings sind P B2 AC1 , P C2 BA1 und P A2 CB1 Parallelogramme, daher gilt
BA1 = C2 P = λ3 a,
CB1 = A2 P = λ1 b,
AC1 = B2 P = λ2 c,
A2 C = P B1 = λ2 a,
B2 A = P C1 = λ3 b,
C2 B = P A1 = λ1 c.
Somit gilt
λ3 a + λ1 a + λ2 a = BA1 + A1 A2 + A2 C = AB = a
⇔ λ1 + λ2 + λ3 = 1.
Andererseits gilt nach Satz 6.4
[P A1 A2 ] = λ21 F,
[P B2 C1 ] = λ2 λ3 F,
[P B1 B2 ] = λ22 F,
[P C2 A1 ] = λ3 λ1 F,
99
[P C1 C2 ] = λ23 F,
[P A2 B1 ] = λ1 λ2 F.
Somit gilt
2
[A1 A2 B1 B2 C1 C2 ] ≥ F
3
2
⇔ [P A1 A2 ] + [P B1 B2 ] + [P C1 C2 ] + [P A2 B1 ] + [P C2 A1 ] + [P B2 C1 ] ≥ F
3
2
⇔ λ21 + λ22 + λ23 + λ1 λ2 + λ2 λ3 + λ3 λ1 ≥
3
2
⇔ λ21 + λ22 + λ23 + λ1 λ2 + λ2 λ3 + λ3 λ1 ≥ (λ1 + λ2 + λ3 )2
3
2
2
2
⇔ 3 λ1 + λ2 + λ3 + λ1 λ2 + λ2 λ3 + λ3 λ1 ≥ 2 (λ1 + λ2 + λ3 )2 .
Allerdings gilt
3 λ21 + λ22 + λ23 + λ1 λ2 + λ2 λ3 + λ3 λ1 − 2 (λ1 + λ2 + λ3 )2
= λ21 + λ22 + λ23 − λ1 λ2 − λ2 λ3 − λ3 λ1
(λ1 − λ2 )2 + (λ2 − λ3 )2 + (λ3 − λ1 )2
=
≥ 0.
2
Beispiel 6.2.6 (IMO 1966). Auf den Seiten BC, CA, AB eines Dreiecks werden jeweils
ein innerer Punkt K, L, M , beliebig, aber verschieden von den Eckpunkten gewählt. Zeige,
dass die Flächeninhalte der Dreiecke AM L, BKM , CLK nicht alle größer als ein Viertel
des Flächeninhalt des Dreiecks ABC sein können.
A
ν1
F1
M
µ2
L
ν2
µ1
F2
B
λ1
F3
K
λ2
C
Abbildung 6.4: Zeichnung zu Beispiel 6.2.6 (IMO 1966)
Lösung. Seien F = [ABC], F1 = [AM L], F2 = [BKM ] und F3 = [CLK]. Seien weiters λ1 =
BK/BC, λ2 = KC/BC, µ1 = CL/CA, µ2 = LA/CA, ν1 = AM/AB und ν2 = M B/AB.
Dann gilt
λ1 + λ2 = µ1 + µ2 = ν1 + ν2 = 1.
100
Weiters gilt nach Satz 6.4
F1 = µ2 ν1 F,
F2 = ν2 λ1 F,
F3 = λ2 µ1 F.
Nach der arithmetisch-geometrischen Mittelungleichung gilt allerdings
6 3
λ1 + λ2 + µ1 + µ2 + ν1 + ν2
1
µ2 ν1 · ν2 λ1 · λ2 µ1 ≤
=
.
6
4
Somit ist mindestens eine der Zahlen µ2 ν1 , ν2 λ1 , λ2 µ1 kleiner oder gleich 1/4 und somit
mindestens eine der Flächen F1 , F2 , F3 kleiner oder gleich F/4.
6.3
Jensen-Ungleichung
Es sei I ⊆ R ein Intervall und f : I → R eine Funktion. Dann heißt die Funktion f konvex
im Intervall I, wenn für alle x1 , x2 ∈ I und für alle t ∈ [0; 1] die Ungleichung
tf (x1 ) + (1 − t)f (x2 ) ≥ f (tx1 + (1 − t)x2 )
(6.1)
erfüllt ist.
P2
O
P1
Abbildung 6.5: Konvexe Funktionen
Geometrisch kann dies so interpretiert werden, dass für zwei Punkte P1 (x1 /y1 ) und
P2 (x2 /y2 ) auf dem Graphen von f (mit x1 , x2 ∈ I) die Sehne P1 P2 immer über dem Graphen
liegt.
Falls in (6.1) ≤ statt ≥ gilt, so heißt die Funktion auch konkav im Intervall I.
Falls f im Intervall I zwei Mal differenzierbar ist, so ist f im Intervall I genau dann
konvex, wenn f 00 (x) ≥ 0 (bzw genau dann konkav, wenn f 00 (x) ≤ 0) für alle x ∈ I gilt. Auf
den Beweis wird hier nicht eingegangen.
Man kann sich nun fragen, was passiert, wenn man (6.1) auf mehr als zwei Variablen ausdehnen will. In der Tat liefert eine kurze Induktion die äußerst nützliche Jensen-Ungleichung.
101
Satz 6.5 (Jensen-Ungleichung). Sei f : I → R eine im Intervall I konvexe Funktion. Dann
gilt für alle x1 , . . . , xn ∈ I und alle t1 , . . . , tn ∈ [0; 1] mit t1 +. . .+tn = 1 folgende Ungleichung:
t1 f (x1 ) + . . . + tn f (xn ) ≥ f (t1 x1 + . . . + tn xn ).
Falls f im Intervall I konkav ist, so gilt die umgekehrte Ungleichung
t1 f (x1 ) + . . . + tn f (xn ) ≤ f (t1 x1 + . . . + tn xn ).
Mit der Jensen-Ungleichung kann man nun beispielsweise sehr einfach die Mittelungleichungen beweisen (für AM-GM nehme man beispielsweise f (x) = ln(x)).
Für geometrische Ungleichungen sind oft die Konvexität bzw. Konkavität von Winkelfunktionen sehr nützlich (zB ist sin x auf [0◦ ; 180◦ ] konkav, cos x auf [−90◦ ; 90◦ ] konkav,
usw).
Beispiel 6.3.1 (IMO Shortlist 1988). Die Innenwinkelsymmetralen eines Dreiecks ABC
schneiden den Umkreis des Dreiecks jeweils ein weiteres Mal in den Punkten A0 , B 0 , C 0 .
Zeige, dass [A0 B 0 C 0 ] ≥ [ABC] gilt.
A
C'
B'
I
C
B
A'
Abbildung 6.6: Zeichnung zu Beispiel 6.3.1 (IMO Shortlist 1988)
Lösung. Für ein Dreieck mit den Seitenlängen a, b, c, den Innenwinkeln α, β, γ, dem Umkreisradius R und dem Flächeninhalt F gilt wegen
F =
abc
4R
und
a
b
c
=
=
= 2R
sin α
sin β
sin γ
für den Flächeninhalt des Dreiecks die Formel
F = 2R2 sin α sin β sin γ.
102
(6.2)
Seien nun α = ∠CAB, β = ∠ABC, γ = ∠BCA. Nach dem Peripheriewinkelsatz gilt für
die Innenwinkel des Dreiecks A0 B 0 C 0
∠C 0 A0 B 0 = ∠C 0 A0 A + ∠AA0 B 0 = ∠C 0 CA + ∠ABB 0 =
β+γ
γ β
+ =
2
2
2
und analog auch
α+β
γ+α
sowie ∠B 0 C 0 A0 =
.
2
2
Da die Dreiecke ABC und A0 B 0 C 0 denselben Umkreisradius haben, genügt es nach (6.2),
die Ungleichung
γ+α
α+β
β+γ
sin
sin
≥ sin α sin β sin γ
sin
2
2
2
zu zeigen.
Wenden wir die Jensen-Ungleichung auf sin x an, so gilt
∠A0 B 0 C 0 =
sin
β+γ
γ+α
α+β
sin β + sin γ sin γ + sin α sin α + sin β
· sin
· sin
≥
·
·
2
2
2
2
2
p
p 2
p
≥ sin β sin γ · sin γ sin α · sin α sin β
= sin α sin β sin γ.
Beispiel 6.3.2 (IMO 1991). Sei ABC ein Dreieck und P ein Punkt im Inneren des Dreiecks.
Zeige, dass zumindest einer der Winkel ∠P AB, ∠P BC, ∠P CA kleiner oder gleich 30◦ ist.
C
P
A
B
Abbildung 6.7: Zeichnung zu Beispiel 6.3.2 (IMO 1991)
Lösung. Wenden wir den Sinussatz auf die Dreiecke P AB, P BC, P CA an, so erhalten wir
PB
PA
=
,
sin ∠P AB
sin ∠P BA
PC
PB
=
,
sin ∠P BC
sin ∠P CB
PA
PC
=
.
sin ∠P CA
sin ∠P AC
Durch Multiplikation erhalten wir somit
sin ∠P AB · sin ∠P BC · sin ∠P CA = sin ∠P BA · sin ∠P CB · sin ∠P AC.
103
Wenden wir die Jensen-Ungleichung auf sin x an, so gilt
sin ∠P AB · sin ∠P BC · sin ∠P CA
√
= sin ∠P AB · sin ∠P BC · sin ∠P CA · sin ∠P BA · sin ∠P CB · sin ∠P AC
3
sin ∠P AB + sin ∠P BC + sin ∠P CA + sin ∠P BA + sin ∠P CB + sin ∠P AC
≤
6
3
∠P AB + ∠P BC + ∠P CA + ∠P BA + ∠P CB + ∠P AC
≤ sin
6
1
= (sin 30◦ )3 = .
8
Somit gilt für mindestens einen der Winkel θ ∈ {∠P AB, ∠P BC, ∠P CA}, dass sin θ ∈
(0; 1/2]. Somit ist entweder θ ∈ (0◦ ; 30◦ ] oder θ ∈ [150◦ ; 180◦ ). Falls Ersteres gilt, so ist die
Aufgabe gelöst. Falls Letzteres gilt, so ist mindestens einer der anderen Winkel kleiner oder
gleich 30◦ .
6.4
Satz von Ptolemäus
Satz 6.6 (Satz von Ptolemäus). Seien A, B, C, D vier Punkte, nicht alle auf einer Gerade.
Dann gilt
AB · CD + BC · DA ≥ AC · BD
mit Gleichheit genau dann, wenn ABCD ein konvexes Sehnenviereck ist.
D'
D
C'
C
B'
A
B
Abbildung 6.8: Satz von Ptolemäus
104
Beweis. Wir betrachten eine Kreisinversion mit Mittelpunkt A und beliebigem Radius r.
Dabei gehe B in B 0 , C in C 0 und D in D0 über. Nach Satz 5.1 gilt
B0C 0 =
BC · r2
,
AB · AC
CD · r2
,
AC · AD
C 0 D0 =
B 0 D0 =
BD · r2
.
AB · AD
Nach der Dreiecksungleichung gilt allerdings
B 0 C 0 + C 0 D0 ≥ B 0 D0
also
(6.3)
BC · r2
CD · r2
BD · r2
+
≥
AB · AC AC · AD
AB · AD
und daher
BC · DA + AB · CD ≥ AC · BD.
Gleichheit gilt genau dann, wenn bei (6.3) Gleichheit gilt. Dies ist allerdings genau dann der
Fall, wenn B 0 , C 0 , D0 in dieser Reihenfolge auf einer Geraden liegen, wenn also A, B, C, D
in dieser Reihenfolge auf einem Kreis liegen.
Beispiel 6.4.1. Sei ABC ein Dreieck und M ein Punkt auf dessen Umkreis. Zeige folgende
Ungleichung:
MA MB MC
+
+
≥ 2.
BC
CA
AB
A
B
C
M
Abbildung 6.9: Zeichnung zu Beispiel 6.4.1
Lösung. Seien a, b, c die Seitenlängen des Dreiecks. Wir können oBdA annehmen, dass M
_
auf dem Bogen BC (der A nicht enthält) liegt. Dann gilt nach Ptolemäus
b · M B + c · M C = a · M A.
Somit gilt
MA
b
c
= MB · 2 + MC · 2
a
a
a
105
und daher
1
b
1
c
+
+ MC ·
+
b a2
c a2
a2 + c 2
a2 + b 2
+
M
C
·
= MB ·
a2 b
a2 c
2ab
2ac
≥ MB · 2 + MC · 2
ab
ac
MB + MC
=2·
≥2
a
MA MB MC
+
+
= MB ·
a
b
c
denn es gilt a2 +b2 ≥ 2ab, a2 +c2 ≥ 2ac und nach der Dreiecksungleichung M B+M C ≥ a.
Beispiel 6.4.2 (IMO Shortlist 1996). Sei ABC ein spitzwinkeliges Dreieck mit Umkreismittelpunkt O und Umkreisradius R. Sei A0 der zweite Schnittpunkt der Geraden AO mit dem
Umkreis von 4BOC. Ebenso seien B 0 , C 0 die zweiten Schnittpunkte von BO bzw CO mit
den Umkreisen von 4COA bzw. 4AOB. Zeige, dass
OA0 · OB 0 · OC 0 ≥ 8 · R3
und untersuche, wann Gleichheit gilt.
C'
A
B'
O
C
B
A'
Abbildung 6.10: Zeichnung zu Beispiel 6.4.2 (IMO Shortlist 1996)
Lösung. Es sei ϕ = ∠COB 0 = ∠BOC 0 , χ = ∠AOC 0 = ∠COA0 sowie ψ = ∠BOA0 =
∠AOB 0 . Da BOCA0 ein Sehnenviereck ist, gilt
∠CA0 B = ϕ,
∠A0 BC = χ,
106
∠BCA0 = ψ.
Nach dem Satz von Ptolemäus gilt allerdings
OA0 · BC = OB · A0 C + OC · A0 B
und daher
0
OA = R ·
A0 C A0 B
+
BC
BC
.
Nach dem Sinussatz gilt aber
A0 C
sin χ
=
BC
sin ϕ
und daher
0
OA = R ·
sowie
A0 B
sin ψ
=
BC
sin ϕ
sin χ sin ψ
+
sin ϕ sin ϕ
.
Zyklische Multiplikation ergibt
sin χ sin ψ
sin ψ sin ϕ
sin ϕ sin χ
0
0
0
3
OA · OB · OC = R ·
+
·
+
·
+
.
sin ϕ sin ϕ
sin χ sin χ
sin ψ sin ψ
Sicherlich gilt ϕ + χ + ψ = 180◦ und daher sin ϕ, sin χ, sin ψ ∈ (0; 1). Nach arithmetischgeometrischer Mittelungleichung gilt somit
sin ψ sin ϕ
sin ϕ sin χ
sin χ sin ψ
+
·
+
·
+
≥8
sin ϕ sin ϕ
sin χ sin χ
sin ψ sin ψ
was die behauptete Ungleichung beweist. Gleichheit tritt genau dann ein, wenn sin ϕ =
sin χ = sin ψ gilt. Wegen ϕ + χ + ψ = 180◦ ist dies genau dann der Fall, wenn ϕ = χ = ψ =
60◦ .
107
Anhang A
Orientierte Winkel modulo 180◦
A.1
Definition und Eigenschaften von Kreiswinkeln
Orientierte Winkel modulo 180◦ , auch Kreiswinkel oder Kreuze genannt, sind ein äußerst
nützliches Werkzeug, um sich lästige Fallunterscheidungen bezüglich Punkteandordnungen
zu ersparen. Möchte man beispielsweise beweisen, dass vier Punkte A, B, C, D auf einem Kreis liegen, so müsste man prinzipiell zwei Fälle unterscheiden. Im ersten Fall nimmt
man an, dass die Punkte C und D auf derselben Seite von AB liegen und zeigt, dass
∠ACB = ∠ADB gilt. Im zweiten Fall nimmt man dann an, dass die Punkte C und D
auf verschiedenen Seiten von AB liegen und zeigt, dass ∠ACB = 180◦ − ∠ADB gilt. Bei
Beispielen, wo mit sehr vielen Punkten gearbeitet wird, deren Anordnung nicht festgelegt
ist, kann die Anzahl solcher Fallunterscheidungen sehr stark wachsen. Die Verwendung von
Orientierten Winkeln modulo 180◦ ist eine Möglichkeit, sich diese Fallunterscheidungen zu
ersparen und den Beweis zumindest formal stark zu verkürzen. Oftmals können dadurch Beispiele sogar ohne Zeichnung gelöst werden. Eine sehr gute und ausführliche Einführung in
diesen Winkeltyp mit vielen Beispielen ist unter [5] zu finden. Ich erlaube mir, im Folgenden
diese Einführung stark verkürzt wiederzugeben.
Ein gewöhnlicher Winkel ∠ASB ist vorzeichenlos, es gilt also ∠ASB = ∠BSA. Um mit
Kreiswinkeln arbeiten zu können, müssen wir Winkel zunächst mit Vorzeichen versehen, sodass ]ASB = −]BSA gilt. Dabei heiße ein Winkel positiv, falls er gegen den Uhrzeigersinn
läuft und negativ, falls er im Uhrzeigersinn läuft.
Ist α nun ein gewöhnlicher gerichteter Winkel, so bezeichnet der Kreiswinkel Kα genau
genommen eine Winkelklasse. Diese besteht aus jenen Winkeln, die sich aus α plus einem
ganzzahligen Vielfachen von 180◦ zusammensetzen, also
Kα = {. . . , α − 360◦ , α − 180◦ , α, α + 180◦ , α + 360◦ , . . .}.
Wenn wir im Folgenden mit Kreiswinkeln arbeiten, so sei unter einem Kreiswinkel α immer
ein Repräsetant α ∈ Kα gemeint. Wir werden schnell feststellen, dass es beim Arbeiten mit
Kreiswinkeln irrelevant ist, welcher Repräsentant genau gemeint ist.
Sowohl für das Addieren wie auch das Subtrahieren zweier Kreiswinkel ist die Wahl des
konkreten Repräsetanten unerheblich, denn diese unterscheiden sich nur in den Vielfachen
108
von 180◦ und diese sind für den Kreiswinkel nicht von Bedeutung. Auch für das Multiplizieren
mit ganzzahligen Vielfachen ist es unerheblich. Um es gleich vorweg zu nehmen, dividieren
kann man Kreiswinkel nicht ordentlich, denn das Ergebnis ist üblicherweise nicht eindeutig
(es gibt beispielsweise, zwei Kreiswinkel, die mit 2 multipliziert 0◦ ergeben, nämlich 0◦ und
90◦ , da wir ja 180◦ = 0◦ festgelegt haben). Übrigens folgt aus 180◦ = 0◦ , dass 90◦ = −90◦ .
Im folgenden seien die wichtigsten Eigenschaften von Kreiswinkeln angemerkt. Die meisten davon sind relativ offensichtlich, sodass ich sie nicht explizit beweisen werde.
Satz A.1. Seien g und h zwei Geraden durch einen Punkt S. Seien A, A0 Punkte auf g und
B, B 0 Punkte auf h. Dann gilt
]ASB = ]A0 SB 0 .
B'
B
B
α
S
S
A
α
A'
A
A'
α
B'
B
B'
B
α
A'
A'
S
α
S
α
A
A
α
B'
Abbildung A.1: Anordnungsunabhängigkeit von Kreiswinkel
Wir stellen fest, dass diese Gleichheit unabhängig von der Punkteanordnung gilt. Würden
wir mit normalen ungerichteten Winkeln arbeiten, so gälte ∠ASB = ∠A0 SB 0 oder ∠ASB =
180◦ − ∠A0 SB 0 , abhängig von der Anordnung der Punkte.
Bei Kreiswinkeln ist also der Winkel nur von der Wahl der Geraden g und h, nicht
aber von der Wahl der Punkte A, B abhängig. Somit können wir Kreiswinkel auch als
einen Winkel zwischen zwei Geraden interpretieren (nicht wie bei gewöhnlichen Winkeln
zwischen zwei Halbgeraden), wir können also statt ]ASB auch ](g; h) schreiben. Man kann
den Kreiswinkel ](g; h) auch so interpretieren, dass dies jener Winkel ist, um den man die
Gerade g in bzw gegen den Uhrzeigersinn (jenachdem, ob der Winkel negativ oder positiv
ist) drehen muss, sodass das Abbild von g zu h parallel ist. Für zwei parallele Geraden g, h
definieren wir ](g; h) = 0◦ . Offensichtlich gilt wegen ]ASB = −]BSA auch
109
Satz A.2. ](g; h) = −](h; g).
Auch Parallelwinkel können leicht auf Kreiswinkel übertragen werden:
Satz A.3. Seien g und g 0 zwei parallele Geraden sowie h und h0 zwei parallele Geraden.
Dann gilt ](g; h) = ](g 0 ; h0 ).
Kommen wir nun zu den additiven Eigenschaften, die gegenüber euklidischen Winkeln
ebenfalls einen großen Vorteil darstellen:
Satz A.4. Für beliebige Punkte A1 , A2 , . . . , An , S gilt
]A1 SA2 + ]A2 SA3 + . . . + ]An−1 SAn = ]A1 SAn
bzw
]A1 SA2 + ]A2 SA3 + . . . + ]An−1 SAn + ]An SA1 = 0◦ .
Daraus lässt sich leicht folgern:
Satz A.5. Für n beliebige Geraden g1 , . . . , gn gilt
](g1 ; g2 ) + ](g2 ; g3 ) + . . . + ](gn−1 ; gn ) = ](g1 ; gn )
bzw
](g1 ; g2 ) + ](g2 ; g3 ) + . . . + ](gn−1 ; gn ) + ](gn ; g1 ) = 0◦ .
Übrigens bedeutet dies für den Fall n = 3, dass die Winkelsumme in einem Dreieck 0◦
beträgt.
Satz A.6. Drei Punkte A, B, C liegen genau dann auf einer Geraden, wenn ]ABC = 0◦
gilt.
Satz A.7. Spiegelt man die Punkte A, B, S an einer Geraden und sind A0 , B 0 , S 0 die gespiegelten Punkte, so gilt ]ASB = −]A0 S 0 B 0 .
Dies hat zur Folge, dass sich mithilfe von Kreiswinkel leicht erkennen lässt, ob ähnliche
Dreiecke gleichsinnig oder gegensinnig ähnlich sind.
Satz A.8. Zwei Dreiecke ABC und A0 B 0 C 0 sind gleichsinnig ähnlich, wenn
]CAB = ]C 0 A0 B 0 ,
]ABC = ]A0 B 0 C 0 ,
]BCA = ]B 0 C 0 A0 .
Zwei Dreiecke ABC und A0 B 0 C 0 sind gegensinnig ähnlich, wenn
]CAB = −]C 0 A0 B 0 ,
]ABC = −]A0 B 0 C 0 ,
]BCA = −]B 0 C 0 A0 .
Kommen wir nun zum Peripheriewinkelsatz für Kreiswinkel. Wir erinnern uns, dass für
Euklidische Winkel vier Punkte A, B, C, D genau dann auf einem Kreis liegen, wenn entweder C und D auf derselben Seite von AB liegen und ∠ACB = ∠ADB gilt, oder aber,
wenn C und D auf verschiedenen Seiten von AB liegen und die Winkel ∠ACB und ∠ADB
supplementär sind. Für Kreiswinkel ist diese Fallunterscheidung überflüssig:
110
C
C
α
D
α
α
B
A
B
A
α
D
Abbildung A.2: Peripheriewinkelsatz für Kreiswinkel
Satz A.9. Vier paarweise verschiedene Punkte A, B, C, D liegen genau dann auf einem
Kreis, wenn
]ACB = ]ADB
gilt.
Auch der Peripheriewinkel-Tangentenwinkelsatz und der Peripheriewinkel-Zentriwinkelsatz
lassen sich für Kreiswinkel übernehmen:
Satz A.10. Ist t die Tangente in B an dem Umkreis von 4ABC, so gilt
]ACB = ](AB; t).
C
α
B
A
α
Abbildung A.3: Peripheriewinkel-Tangentenwinkelsatz für Kreiswinkel
Satz A.11. Sei O der Umkreismittelpunkt von 4ABC und α = ]ACB. Dann gilt
]AOB = 2 · α
sowie
]BAO = ]OBA = 90◦ − α.
111
C
α
O
2α
90°-α
90°-α
B
A
Abbildung A.4: Peripheriewinkel-Zentriwinkelsatz für Kreiswinkel
Kreiswinkel haben allerdings auch Nachteile. Einerseits kann man, wie anfangs schon
erwähnt, Kreiswinkel nicht eindeutig dividieren, denn es gibt im Allgemeinen für eine positive
ganze Zahl k und einen Kreiswinkel α insgesamt k verschiede Kreiswinkel β, sodass α = k · β
gilt. Des Weiteren ist das Arbeiten mit Kreiswinkeln üblicherweise nicht möglich, sobald
Trigonometrie ins Spiel kommt, da es für Kreiswinkel keine Sinus und Kosinus gibt (beispielsweise gilt cos 180◦ 6= cos 0◦ ). Auch können Kreiswinkel keiner Größenordnungsrelation
unterzogen werden, d.h. es gibt keine sinnvolle Definition, wann für zwei Kreiswinkel α, β die
Ungleichung α < β gilt. Folglich können Kreiswinkel vor allem nicht für geometrische Ungleichungen verwendet werden, was allerdings schon aufgrund der Tatsache evident ist, dass
Anordnungen bei geometrischen Ungleichungen meistens wesentlich sind, der Sinn von Kreiswinkeln aber genau darin besteht, sich über Anordnungsangelegenheiten keine Gedanken zu
machen.
A.2
Anwendungen von Kreiswinkeln
Die Mächtigkeit von Kreiswinkeln wurde schon in Kapitel 2.4 demonstriert. Es sei aber
noch deutlich darauf hinweisen, dass Kreiswinkel zwar ein Werkzeug darstellen, welches die
Formulierungen und Rechnungen verkürzt, sicherlich aber keine konkrete Lösungsstrategie
liefern. Eine schwere Aufgabe wird durch die Verwendung von Kreiswinkeln nicht einfacher,
lediglich lässt sich die Lösung (hoffentlich) etwas kürzer schreiben.
Satz A.12. Spiegelt man den Höhenschnittpunkt H eines Dreiecks ABC an den Dreiecksseiten, so liegen die gespiegelten Punkte auf dem Umkreis von 4ABC.
Beweis. Es sei Ha das Spiegelbild von H an BC. Sicherlich gilt BH ⊥ CA und CH ⊥ AB
112
A
Hc
Hb
H
B
C
Ha
Abbildung A.5: Höhenschnittpunkt an Seiten gespiegelt
und daher gilt
]BHC = ](BH; CH)
= ](BH; CA) + ](CA; AB) + ](AB; CH)
= 90◦ + ](CA; AB) + 90◦ = 180◦ + ](CA; AB)
= ](CA; AB) = ]CAB.
Da aber Ha das Spiegelbild von H an BC ist, gilt ]BHa C = −]BHC und daher
]BHa C = −]BHC = −]CAB = ]BAC.
Somit liegen die Punkte A, B, C, Ha auf einem Kreis. Analoges Vorgehen für die andere
Seiten beweist den Satz (genau genommen deckt dieser Beweis ein rechtwinkeliges Dreieck
nicht ab, dieser Fall ist allerdings trivial).
Wir sehen, dass wir im Grunde genommen gar keine Zeichnung gebraucht hätten.
Beispiel A.2.1. Sei P ein beliebiger Punkt in der Ebene des Dreiecks ABC, der aber nicht
auf dessen Umkreis liegt. Die Fußpunkte der Lote von P auf die Geraden BC, CA und AB
seien X, Y , Z. Die Geraden AP , BP und CP schneiden den Umkreis des Dreiecks ABC
zum zweiten Mal in den Punkten D, E und F . Man zeige, dass die Dreiecke XY Z und DEF
gleichsinnig ähnlich sind.
Lösung. Da P XCY und P XBZ Sehnenvierecke sind, gilt
]ZXY = ]ZXP + ]P XY
= ]ZBP + ]P CY = ]ABE + ]F CA
= ]ADE + ]F DA = ]F DA + ]ADE
= ]F DE.
113
A
F
Z
E
Y
P
B
C
X
D
Abbildung A.6: Zeichnung zu Beispiel A.2.1
Analog zeigt man, dass ]XY Z = ]DEF und ]Y ZX = ]EF D gilt, womit die Dreiecke
XY Z und DEF gleichsinnig ähnlich sind.
Beispiel A.2.2. Seien g1 , g2 , g3 , g4 vier Geraden, sodass keine zwei der Geraden parallel
sind und keine drei der Geraden durch einen gemeinsamen Punkt gehen. Diese vier Geraden
begrenzen vier Dreiecke. Zeige, dass die Umkreise dieser Dreiecke durch einen gemeinsamen
Punkt gehen.
P23
P24
T
P34
g1
P14
P13
P12
g4
g3
g2
Abbildung A.7: Zeichnung zu Beispiel A.2.2
114
Lösung. Für 1 ≤ i < j ≤ 4 sei Pij der Schnittpunkt der Geraden gi und gj . Sei T der zweite
Schnittpunkt der Umkreise von 4P12 P13 P23 und 4P23 P24 P34 . Dann gilt
]T P34 P14 = ]T P34 P24 = ]T P23 P24 = ]T P23 P12 = ]T P13 P12 = ]T P13 P14 ,
also liegt T auf dem Umkreis von 4P13 P14 P34 . Analog liegt T auch auf dem Umkreis von
4P12 P14 P24 .
Beispiel A.2.3. Seien A, B, C drei Punkte, die nicht auf einer Gerade liegen. Seien S1
und S2 zwei Kreise, sodass der Kreis S1 die Gerade AB in B berührt, der Kreis S2 die
Gerade AC in C berührt und die Kreise S1 und S2 einander in D berühren. Zeige, dass der
Umkreismittelpunkt des Dreiecks BCD auf dem Umkreis von 4ABC liegt.
t
C
O
B
D
S1
S2
A
Abbildung A.8: Zeichnung zu Beispiel A.2.3
Lösung. Es sei t die gemeinsame Tangente von S1 und S2 in D und O der Umkreismittelpunkt
von 4BCD. Wir wollen zeigen, dass ]BOC = ]BAC gilt.
Nach Peripheriewinkel-Zentriwinkelsatz gilt
]BOC = 2 · ]BDC.
Andererseits gilt ](BD; t) = ](AB; BD) und ](t; CD) = ](DC; CA), daher gilt
]BDC = ](BD; t) + ](t; CD) = ](AB; BD) + ](DC; CA) = ](AB; BD) + ](CD; CA).
Somit gilt insgesamt
]BOC = 2 · ]BDC = ](BD; CD) + ](BD; CD)
= (](BD; BC) + ](BC; CD)) + (](AB; BD) + ](CD; CA))
= ](AB; BD) + ](BD; BC) + ](BC; CD) + ](CD; CA)
= ](AB; CA) = ](BA; AC) = ]BAC.
115
Anhang B
Beispiele, Beispiele, Beispiele, . . .
Zum Schluss sei noch eine Liste mit Aufgaben zum selbstständigen Lösen angefügt. Die
Aufgaben sind allerdings weder nach Schwierigkeitsgrad noch nach Themengebiet sortiert.
1. Sei ABC ein beliebiges Dreieck. Über den Seiten AB und AC errichten wir nach außen
die Quadrate mit den Mittelpunkten K und L. Sei M der Mittelpunkt der Seite BC.
Zeige, dass 4KLM ein gleichschenklig rechtwinkeliges Dreieck ist.
2. Sei ABCD ein konvexes Viereck. Über den Seiten AB, BC, CD und DA errichten wir
nach außen die Quadrate mit den Mittelpunkten K, L, M und N . Zeige, dass KM
normal auf LN steht.
3. (Russland 2006) Sei ABC ein Dreieck mit Umkreismittelpunkt O. Seien A0 , B 0 , C 0
Punkte, sodass CA0 = CA und AA0 ⊥ BC, CB 0 = CB und BB 0 ⊥ AC, BC 0 = BC
und CC 0 ⊥ AB. Sei A1 der Schnittpunkt von OA0 und BC, B1 der Schnittpunkt von
OB 0 und CA, C1 der Schnittpunkt von OC 0 und AB. Zeige, dass sich AA1 , BB1 , CC1
in einem Punkt schneiden.
4. (IMO Shortlist 2004) Sei ABC ein spitzwinkeliges Dreieck mit ∠ABC < ∠ACB und
Umkreismittelpunkt O. Sei D der Schnittpunkt von AO und BC und seien E und
F jeweils die Umkreismittelpunkte der Dreiecke ABD und ACD. Sei weiters G ein
Punkt auf der Verlängerung von AB über A hinaus, sodass AG = AC, und sei H ein
Punkt auf der Verlängerung von AC über A hinaus, sodass AH = AB. Zeige, dass das
Viereck EF GH dann und nur dann ein Rechteck ist, wenn ∠ACB − ∠ABC = 60◦ .
5. Auf dem Halbkreis über einer Strecke AB bewegen sich zwei Punkte C und D, deren
Abstand konstant ist. E sei der Schnittpunkt von AC und BD, F sei der Schnittpunkt
von AD und BC. Beweise, dass der Flächeninhalt des Vierecks AEBF konstant ist.
6. Gegeben ist ein Dreieck ABC und ein innerer Punkt P . Die Abstände von P zu den
Dreiecksseiten seien mit x, y, z bezeichnet. Für welche Lage von P ist das Produkt xyz
maximal?
116
7. In einem Kreis mit dem Mittelpunkt O steht der Radius ON auf die Sehne AB normal.
ON schneidet AB im Punkt M . Der Punkt P liegt auf der Verlängerung von ON . Die
Gerade P M schneidet den Kreis im Punkt Q, die Gerade P N die Sehne AB im Punkt
R. Man beweise, dass die Sehne RN immer länger als die Sehne M Q ist.
8. Es sei BC die längste Seite des Dreiecks ABC und P ein Punkt im Inneren dieses
Dreiecks. A0 , B 0 und C 0 seien die Schnittpunkte der Geraden P A, P B und P C mit den
Seiten BC, CA und AB. Man beweise
P A0 + P B 0 + P C 0 < BC.
9. Gegeben seien eine Gerade g, ein auf g liegender Punkt P und ein nicht auf g liegender
Punkt Q. Ermittle alle auf g liegende Punkte R, für die
PQ + PR
QR
maximal ist.
10. Gegeben sei ein Dreieck mit einer beliebigen Geraden g durch dessen Schwerpunkt.
Liegen zwei Eckpunkte des Dreiecks auf der gleichen Seite von g, so ist die Summe
ihrer Abstände von g gleich dem Abstand des dritten Eckpunktes von g.
11. Für die Ankreisradien ra , rb , rc sowie den Inkreisradius r eines Dreiecks gilt
1
1
1
1
+ + = .
ra rb rc
r
12. Sei ABCD ein Tangentenviereck mit Inkreismittelpunkt I. Beweise:
AI · BI
AB
=
.
CD
CI · DI
13. Sei ABC ein Dreieck und P ein Punkt, der nicht auf dem Umkreis von 4ABC liegt.
Seien X, Y , Z die gespiegelten Punkte von P an BC, CA, AB. Sei Q der Umkreismittelpunkt von 4XY Z. Zeige, dass P und Q isogonal konjugiert bezüglich 4ABC
sind.
14. (IMO 1998) Sei I der Inkreismittelpunkt eines Dreiecks ABC. Seien K, L und M
jeweils die Inkreisberührpunkte auf den Seiten BC, CA bzw AB. Die Gerade durch B
parallel zu M K schneidet die Geraden LM bzw LK in den Punkten R bzw S. Zeige,
dass ∠RIS ein spitzer Winkel ist.
15. (USA 1993) Sei ABCD ein konvexes Viereck in dem die Diagonalen AC und BD
normal aufeinander stehen. Sei O deren Schnittpunkt. Zeige, dass die gespiegelten
Punkte von O an AB, BC, CD, DA auf einem Kreis liegen.
117
16. (Rumänien 2006) Seien ABC und AM N zwei ähnliche Dreiecke mit derselben Orientierung sodass AB = AC und AM = AN und die beiden Dreiecke keine gemeinsame
Schnittfläche haben. Sei O der Umkreismittelpunkt des Dreiecks M AB. Zeige, dass die
Punkte O, C, N , A genau dann auf einem Kreis liegen, wenn ABC gleichseitig ist.
17. (IMO 1993) Für drei Punkte P , Q, R sei m(P QR) die Länge der kürzesten Höhe des
Dreiecks P QR (falls P, Q, R auf einer Geraden liegen, so sei m(P QR) = 0). Zeige für
vier beliebige Punke A, B, C, X die Ungleichung
m(ABC) ≤ m(ABX) + m(AXC) + m(XBC).
18. Sei ABC ein spitzwinkeliges Dreieck und P ein Punkt im Inneren des Dreiecks. AP
schneidet den Umkreis von 4BP C ein weiteres mal in A0 , BP schneidet den Umkreis
von 4CP A ein weiteres mal in B0 und CP schneidet den Umkreis von 4AP B ein
weiteres mal in C0 . Beweise folgende Ungleichungen:
• P A0 · P B0 · P C0 ≥ 8 · P A · P B · P C
P A0 P B0 P C0
+
+
≥ 6.
•
PA
PB
PC
19. (IMO 1994) Sei ABC ein gleichschenkliges Dreieck mit AB = AC. M ist der Mittelpunkt der Strecke BC und O ist ein Punkt auf der Geraden AM , sodass OB normal
auf AB steht. Q ist ein beliebiger Punkt auf BC verschieden von B und C. E sei ein
Punkt auf AB und F sei ein Punkt auf AC sodass E, Q, F verschieden und kollinear
sind. Zeige, dass OQ genau dann normal auf EF steht, wenn QE = QF .
20. (Korea 2003) Sei ABC ein Dreieck, dessen Inkreis die Seiten AB, BC, CA jeweils in
P , Q, R berührt. Zeige folgende Ungleichung:
BC CA AB
+
+
≥ 6.
P Q QR RP
21. Sei ABC ein spitzwinkeliges Dreieck. Bestimme den Ort der Punkte M , N und P auf
den Seiten BC, CA und AB, sodass der Umfang des Dreiecks M N P minimal ist.
22. (Balkan 2003) Es sei ABC ein Dreieck mit AB 6= AC. Die Tangente am Umkreis
von 4ABC in A schneidet BC in D. Die Normale zu BC durch B schneidet die
Streckensymmetrale von AB in E und die Normale zu BC durch C schneidet die
Streckensymmetrale von AC in F . Zeige, dass D, E und F kollinear sind.
23. (IMO 2002) Sei k ein Kreis mit Mittelpunkt O und
Sei A ein Punkt auf k, sodass ∠AOB < 120◦ . Sei
_
AB, welcher C nicht enthält. Die zu DA parallele
in I. Die Streckensymmetralen von OA schneide k
Inkreismittelpunkt des Dreiecks CEF ist.
118
sei BC ein Durchmesser von k.
D der Mittelpunkt des Bogens
Gerade durch O schneidet AC
in E und F . Zeige, dass I der
24. Die Kreise k1 und k2 mit den Mittelpunkten M1 bzw M2 berühren einander von außen
im Punkt T . Diese zwei Kreise berühren den Kreis k mit Mittelpunkt M von innen,
und zwar k1 im Punkt A und k2 im Punkt B. Die gemeinsame Tangente der Kreise k1
und k2 im Punkt T schneidet k in den Punkten K und L. Sei D der Mittelpunkt der
Strecke KL. Zeige, dass ∠M1 M M2 = ∠ADB gilt.
25. (Bulgarien 2004) Seien I der Inkreismittelpunkt und O der Umkreismittelpunkt des
Dreiecks ABC und seien A1 , B1 , C1 Punkte auf den Strecken AI, BI, CI. Die Streckensymmetralen von AA1 , BB1 , CC1 schneiden sich jeweils in A2 , B2 , C2 . Zeige, dass O
genau dann der Umkreismittelpunkt von 4A2 B2 C2 ist, wenn I der Höhenschnittpunkt
von A1 B1 C1 ist.
26. (IMO 1998) In einem konvexen Viereck ABCD stehen die Diagonalen AC und BD
senkrecht aufeinander und die gegenüberliegenden Seiten AB und CD sind nicht parallel. Die Streckensymmetralen von AB und CD schneiden sich im Punkt P innerhalb
von ABCD. Man beweise: ABCD ist dann und nur dann ein Sehnenviereck, wenn die
Dreiecke ABP und CDP gleichen Flächeninhalt haben.
27. Gegeben sei ein spitzwinkeliges Dreieck ABC mit dem Umkreismittelpunkt O. Die
Gerade AO schneidet BC in D. Die Punkte E und F liegen auf den Seiten AB bzw
AC derart, dass A, E, D, F auf einem Kreis liegen. E0 und F0 seien die Fußpunkte
der Normalen von E und F auf BC. Zeige, dass die Länge von E0 F0 von der Wahl von
E und F unabhängig ist.
28. Sei ABCD ein Rechteck und P ein Punkt auf dessen Umkreis, verschieden von den
Eckpunkten. Seien X, Y , Z und W die Projektionen von P auf die Geraden AB, BC,
CD und DA. Zeige, dass einer der Punkte X, Y , Z, W der Höhenschnittpunkt jenes
Dreiecks ist, das die anderen drei Punkte bilden.
29. (Satz von Archimedes) Sei ABC ein Dreieck mit AC ≥ BC und sei M der Mittelpunkt
des Bogens ACB. Sei weiters D die Normalprojektion von M auf AC. Zeige, dass
AD = DC + CB gilt.
_
30. (IMO Shortlist 2002) Sei ABC ein Dreieck, sodass ein innerer Punkt F existiert, für
den ∠AF B = ∠BF C = ∠CF A gilt. Die Geraden BF und CF schneiden AC und AB
jeweils in D und E. Zeige, dass AB + AC ≥ 4DE.
31. (Russland 1996) Sei ABCD ein konvexes Viereck und es seien E und F Punkte auf
der Seite BC, wobei E näher bei B ist als F . Es sei bekannt, dass ∠BAE = ∠CDF
und ∠EAF = ∠F DE. Zeige, dass ∠F AC = ∠EDB.
32. (Rumänien 1992) Sei ABC ein spitzwinkeliges Dreieck und sei T ein Punkt im Inneren
des Dreiecks, sodass ∠AT B = ∠BT C = ∠CT A. Seien M , N , P die Projektionen
von T auf BC, CA, AB. Der Umkreis des Dreiecks M N T schneide BC, CA, AB ein
weiters mal in M 0 , N 0 , P 0 . Zeige, dass das Dreieck M 0 N 0 P 0 gleichseitig ist.
119
33. Sei A ein fixer Punkt auf dem Strahl Ox eines Winkels xOy. Ein variabler Kreis k
berührt Ox und Oy. Sei D der Berührpunkt von k un Oy. Die zweite Tangente von A
an k berühre k in E. Zeige, dass DE durch einen fixen Punkt geht.
34. Zwei Kreise k1 und k2 schneiden sich in A und B. Eine Gerade durch A schneidet k1
in C und k2 in D. Seien P und Q die Normalprojektionen von B auf die Tangenten in
C und D an k1 bzw k2 . Zeige, dass P Q eine Tangente am Kreis mit Durchmesser AB
ist.
35. (Eulersche Formel) Sei O der Umkreismittelpunkt, I der Inkreismittelpunkt, R der
Umkreisradius und r der Inkreisradius eines Dreiecks. Dann gilt OI 2 = R2 − 2Rr.
36. (Balkan 2005) Es sei ABC ein beliebiges spitzwinkeliges Dreieck, dessen Inkreis die
Seiten AB und AC in den Punkten D bzw. E berührt. Die Gerade durch D und E
schneidet die Winkelsymmetralen von ∠ACB und ∠ABC in den Punkten X bzw Y .
Z ist der Mittelpunkt der Seite BC. Man zeige: Das Dreieck XY Z ist genau dann
gleichseitig, wenn ∠BAC = 60◦ gilt.
37. (IMO 2001) Sei ABC ein spitzwinkeliges Dreieck mit ∠BCA ≥ ∠ABC + 30◦ . Sei O
der Umkreismittelpunkt des Dreiecks und sei P der Höhenfußpunkt von A auf BC.
Zeige, dass ∠CAB + ∠COP < 90◦ .
38. (Russland 2006) Gegeben seien ein Kreis und ein Punkt P , der nicht auf dem Kreis
liegt. Bestimme den geometrischen Ort des Höhenschnittpunktes von 4P AB, wenn
AB ein beliebiger Durchmesser des Kreises ist und variiert.
39. Es seien k1 , k2 , k3 und k4 Kreise, sodass sich k1 und k2 in A, k2 und k3 in B, k3 und
k4 in C und k4 und k1 in D berühren. Zeige, dass die Punkte A, B, C und D entweder
auf einer Geraden oder auf einem Kreis liegen.
40. (Rumänien 2005) Zwei Kreise k1 und k2 schneiden einander in den Punkten A und
B. Die Tangente in A an k2 schneidet k1 im Punkt C, und die Tangente in A an k1
schneidet k2 im Punkt D. Eine Gerade durch A innerhalb des Winkels ∠CAD schneidet
k1 im Punkt M , k2 im Punkt N und den Umkreis des Dreiecks ACD im Punkt P .
Zeige, dass AM = N P gilt.
41. In einem Parallelogramm ABCD werden auf den Seiten AB und BC die Punkte E
und F so gewählt, dass sie mit keinem Eckpunkt zusammenfallen und die Strecken AE
und F C gleich lang sind. Der Schnittpunkt von AE und F C sei G. Zeige, dass DG
den Winkel ∠ADC halbiert.
42. Die Winkelsymmetralen des Dreiecks ABC schneiden seinen Umkreis in den Punkten
A1 , B1 , C1 . Sei I der Inkreismittelpunkt des Dreiecks ABC. AA1 schneide BC in N
und BB1 schneide A1 C1 in P . Sei O der Umkreismittelpunkt des Dreiecks IP C1 . OP
schneide BC in M . Es sei bekannt, dass BM = M N und ∠BAC = 2 · ∠ABC gilt.
Bestimme die Winkel des Dreiecks ABC.
120
43. (IMO Shortlist 2003) Sei ABC ein gleichschenkliges Dreieck mit AC = BC und Inkreismittelpunkt I. P sei ein Punkt auf dem Umkreis des Dreiecks AIB, der im Inneren
des Dreiecks ABC liegt. Die zu CA und CB parallele Geraden durch P schneiden AB
jeweils in D und E. Die zu AB parallele Gerade durch P schneide CA in F und CB
in G. Zeige, dass sich die Geraden DF und EG auf dem Umkreis des Dreiecks ABC
schneiden.
44. Sei ABCD ein Sehnenviereck. Zeige, dass die Inkreismittelpunkte der Dreiecke ABC,
BCD, CDA und DAB die Eckpunkte eines Rechtecks sind.
45. Sei ABCD ein Sehnenviereck und K, L, M , N jeweils die Mittelpunkte der Seiten AB,
BC, CD, DA. Zeige, dass die Höhenschnittpunkte der Dreiecke AN K, BKL, CLM ,
DM N Eckpunkte eines Parallelogramms sind.
46. (USA 1997) Es sei ABC ein beliebiges Dreieck. Wir errichten über den Dreiecksseiten
nach außen die gleichschenkligen Dreiecke BCD, CAE, ABF mit BD = CD, CE =
AE, AF = BF . Zeige, dass sich die jeweils zu BC, CA, AB normale Geraden durch
A, B, C in einem Punkt schneiden.
47. (IMO Shortlist 2003) Seien Γ1 , Γ2 , Γ3 , Γ4 vier verschiedene Kreise, sodass sich Γ1 und
Γ3 von außen im Punkt P berühren, und sich Γ2 und Γ4 von außen im selben Punkt
P berühren. Γ1 schneidet Γ2 ein weiteres Mal in A, Γ2 schneidet Γ3 ein weiteres Mal
in B, Γ3 schneidet Γ4 ein weiteres Mal in C und Γ4 schneidet Γ1 ein weiteres Mal in
D, wobei A, B, C und D alle von P verschieden sind. Zeige, dass
P B2
AB · BC
=
.
AD · DC
P D2
48. (Bulgarien 1997) Sei ABC ein Dreieck mit Höhenschnittpunkt H und seien M und
K die Mittelpunkte von AB und CH. Zeige, dass sich die Winkelsymmetralen von
∠CAH und ∠CBH auf der Gerade M K schneiden.
49. (USA 1998) Seien ω1 und ω2 zwei konzentrische Kreise, wobei ω2 im Inneren von ω1
sei. Eine Sehne AC von ω1 sei Tangente an ω2 in B. D sei der Mittelpunkt von AB.
Eine Gerade durch A schneidet ω2 in E und F , sodass sich die Streckensymmetralen
von DE und CF auf AB im Punkt M schneiden. Man bestimme AM/M C.
50. (IMO 1985) Ein Kreis mit Mittelpunkt O geht durch die Eckpunkte A und C eines
Dreiecks ABC und schneidet die Strecken AB und BC jeweils ein weiters Mal in
den verschiedenen Punkten K und N . Die Umkreise der Dreiecke ABC und KBN
schneiden sich in Punkten B und M , wobei B und M voneinander verschieden sind.
Zeige, dass ∠OM B = 90◦ .
51. Eine Gerade durch den Inkreismittelpunkt I eines Dreiecks ABC schneide den Umkreis
des Dreiecks in den Punkten F und G und den Inkreis des Dreiecks in den Punkten
D und E, wobei D zwischen I und F liege. Man zeige F D · EG ≥ r2 , wobei r der
Inkreisradius des Dreiecks ABC sei. Wann tritt Gleichheit auf?
121
52. (Großbritannien 2007) Sei ABC ein spitzwinkliges Dreieck mit AB > AC und ∠BAC =
60◦ . Seien O der Umkreismittelpunkt und H der Höhenschnittpunkt des Dreiecks
und seien P und Q jeweils die Schnittpunkte von OH mit AB und AC. Zeige, dass
P O = HQ.
53. Sei ABC ein Dreieck und seien X, Y , Z jeweils Punkte auf den Strecken BC, CA,
AB, sodass die Geraden AX, BY , CZ durch einen gemeinsamen Punkt gehen. Zeige,
dass die Fläche von 4XY Z nicht größer als ein Viertel der Fläche von 4ABC ist.
54. Sei ABC ein Dreieck und seien B 0 und C 0 zwei Punkte auf dem Umkreis des Dreiecks,
sodass ∠C 0 BA = ∠B 0 CA. Die zu AC parallele Gerade zu C 0 schneide die zu AB
parallele Gerade durch B 0 in A0 . Sei A00 der Schnittpunkt der Geraden BC 0 und CB 0 .
Zeige, dass die Punkte A, A0 und A00 auf einer Geraden liegen.
55. (Iran 2006) Sei ABC ein Dreieck mit AB = AC und sei P ein Punkt auf der Geraden
BC. X und Y seien jeweils Punkte auf AB und AC, sodass P X k AC und P Y k AB.
Sei weiters T der Schnittpunkt der Streckensymmetrale von BC mit dem Umkreis von
4ABC, wobei T verschieden von A sei. Zeige, dass P T normal auf XY steht.
56. Sei H der Höhenschnittpunkt eines spitzwinkligen Dreiecks ABC. Die Geraden AH,
BH, CH schneiden den Umkreis des Dreiecks jeweils ein weiteres Mal in A0 , B 0 , C 0 .
Man beweise folgende Ungleichung:
AA0 · BB 0 · CC 0
≥ 8.
AH · BH · CH
57. (Balkan 1997) Die Kreise C1 und C2 berühren sich von außen im Punkt D und berühren
einen weiteren Kreis ω von innen jeweils in B und C. Sei A einer der Schnittpunkte
von ω und der gemeinsamen Tangente an C1 und C2 in D. Die Geraden AB und AC
schneiden C1 und C2 jeweils ein weiteres Mal in K und L. Die Gerade BC schneide
C1 ein weiteres Mal in M und C2 ein weiteres Mal in N . Zeige, dass die Geraden AD,
KM , LN durch einen gemeinsamen Punkt gehen.
58. (Türkei 2006) Sei Q ein Punkt auf einem Kreis mit Durchmesser AB, verschieden von
A und B. H sei der Fußpunkt des Lotes von Q auf AB. Der Kreis mit dem Mittelpunkt
Q und dem Radius QH schneide den Kreis mit dem Durchmesser AB in C und D.
Zeige, dass QH von CD halbiert wird.
59. (Korea 2004) Sei ABC ein spitzwinkliges Dreieck, sodass ∠CAB der größte Innenwinkel des Dreiecks ist. Seien weiters R der Umkreisradius und r der Inkreisradius des
Dreiecks, M der Mittelpunkt der Strecke BC und X der Schnittpunkt der Tangenten
in B und C am Umkreis des Dreiecks ABC. Man zeige die Ungleichung
AM
r
≥
.
R
AX
122
60. (Großbritannien 1996) Zwei Kreise S1 und S2 berühren einander von außen im Punkt
K. Ein weiterer Kreis S berühre die Kreise S1 und S2 von innen, jeweils in den Punkten
A1 und A2 . Sei P einer der Schnittpunkte von S mit der gemeinsamen Tangente an
S1 und S2 in K. Die Gerade P A1 schneide S1 ein weiteres Mal in B1 , die Gerade P A2
schneide S2 ein weiteres Mal in B2 . Zeige, dass B1 B2 eine gemeinsame Tangente an S1
und S2 ist.
61. (Taiwan 2002) Sei ABC ein Dreieck und D ein variabler Punkt auf dem Umkreis von
4ABC, verschieden von A, B, C. Zeige, dass die Simson-Geraden von A, B, C, D
jeweils bezüglich 4BCD, 4CDA, 4DAB, 4ABC durch einen gemeinsamen Punkt
gehen und bestimmte den geometrischen Ort dieses Punktes, wenn D variiert.
62. (Hong Kong 2006) Sei ABCD ein Sehnenviereck mit BC = CD. Die Diagonalen AC
und BD schneiden sich in E. X, Y , Z, W seien jeweils die Inkreismittelpunkte der
Dreiecke ABE, ADE, ABC, ADC. Zeige, dass die Punkte X, Y , Z, W dann und nur
dann auf einem Kreis liegen, wenn AB = AD.
63. (Kanada 2006) Sei ABC ein spitzwinkeliges Dreieck. Diesem Dreieck wird ein Rechteck
DEF G eingeschrieben, sodass D ∈ AB, E ∈ AC, F ∈ BC, G ∈ BC. Bestimme den
geometrischen Ort des Diagonalenschnittpunktes dieses Rechtecks, wenn DEF G alle
solche Rechtecke durchläuft.
64. (Vietnam 2007) Einem Kreis k wird ein Trapez ABCD mit AD k BC eingeschrieben.
Sei P ein variabler Punkt auf der Geraden BC. P A schneide k ein weiteres Mal in N ,
der Kreis mit dem Durchmesser P D schneide k ein weiteres Mal in E. DE schneide
BC in M . Zeige, dass die Gerade M N durch einen festen Punkt geht.
65. (Satz von Kiepert) Sei ABC ein Dreieck. Über den Seiten BC, CA, AB werden nach
außen zueinander ähnliche, gleichschenklige Dreiecke BCA0 , CAB 0 , ABC 0 mit A0 B =
A0 C, B 0 C = B 0 A, C 0 A = C 0 B errichtet. Dann gehen die Geraden AA0 , BB 0 , CC 0 durch
einen gemeinsamen Punkt.
66. (IMO 1982) Im regelmäßigen Sechseck ABCDEF sind M , N Punkte auf AC bzw.
CE. Bestimme r ∈ R, wenn AM/AC = CN/CE = r und B, M , N kollinear sind.
67. (ÖMO 1992) Eine Gerade durch den Schwerpunkt S eines Dreiecks ABC schneide die
Seiten BC und AC jeweils in den Punkten P und Q. Sei weiters M der Mittelpunkt
der Seite AB. Man zeige die Ungleichung
2
[M P Q] ≥ [ABC].
9
68. Sei ABCD ein konvexes Sehnenviereck. Zeige, dass AB · BC · AC + AC · CD · AD =
BC · CD · BD + AB · BD · AD.
123
69. (USA 1994) Ein konvexes Sechseck ABCDEF sei einem Kreis eingeschrieben, sodass
AB = CD = EF und die Diagonalen AD, BE, CF sich in einem Punkt schneiden.
Sei P der Schnittpunkt von AD und CE. Zeige, dass CP/P E = (AC/CE)2 .
70. (USA 1995) Sei ABC ein nicht-gleichschenkliges und nicht-rechtwinkliges Dreieck mit
dem Umkreismittelpunkt O und seien A1 , B1 , C1 jeweils die Mittelpunkte der Seiten
BC, CA, AB. A2 , B2 , C2 seien jeweils jene Punkte auf den Strahlen OA, OB, OC,
sodass 4OAA1 ∼ 4OA2 A, 4OBB1 ∼ 4OB2 B, 4OCC1 ∼ 4OC2 C. Zeige, dass
AA2 , BB2 , CC2 durch einen gemeinsamen Punkt gehen.
71. (USA 1999) Sei ABCD ein gleichschenkliges Trapez mit AB k CD. Der Inkreis des
Dreiecks BCD berühre CD in E. F sei jener Punkt auf der Innenwinkelsymmetrale
von ∠DAC, sodass EF ⊥ CD. Der Umkreis des Dreiecks ACF schneide CD in C und
G. Zeige, dass das Dreieck AF G gleichschenklig ist.
72. Seien k ein Kreis mit dem Mittelpunkt O und A ein Punkt außerhalb von k. Eine
Gerade g durch A, verschieden von AO, schneide k in B und G, wobei B zwischen
A und G liege. Die zu g bezüglich AO symmetrische Gerade h schneide k in C und
H, wobei C zwischen A und H liege. Zeige, dass die Diagonalen des Vierecks BCHG
durch einen Punkt gehen, der unabhängig von der Wahl von g ist.
73. Sei l eine Gerade, die durch den Höhenschnittpunkt H eines Dreiecks ABC geht. Zeige,
dass die Spiegelgeraden von l bezüglich BC, CA, AB durch einen gemeinsamen Punkt
gehen. Zeige weiters, dass dieser Punkt auf dem Umkeis des Dreiecks ABC liegt.
74. (IMO Shortlist 2005) Sei ABC ein Dreieck, sodass AC + BC = 3 · AB. Der Inkreis des
Dreiecks mit Mittelpunkt I berühre die Seiten BC und CA jeweils in D und E. K und
L seien jeweils die Spiegelpunkte von D und E bezüglich I. Zeige, dass die Punkte A,
B, K, L auf einem Kreis liegen.
75. (IMO 1989) Die Innenwinkelsymmetralen des Dreiecks ABC schneiden seinen Umkreis jeweils in den Punkten A1 , B1 , C1 . A0 , B0 , C0 seien die Ankreismittelpunkte des
Dreiecks ABC. Zeige, dass [A0 B0 C0 ] = 2 · [AC1 BA1 CB1 ] ≥ 4 · [ABC].
124
Literaturverzeichnis
[1] Harold S. M. Coxeter und Samuel L. Greitzer, Geometry Revisited, Mathematical Association of America (NML 19), 1967.
[2] Kiran Kedlaya, Geometry Unbound,
http://www-math.mit.edu/~kedlaya/geometryunbound, 2006
[3] Kiran Kedlaya, Notes on Euclidean Geometry,
http://www-math.mit.edu/~kedlaya/geometryunbound/geom-080399.pdf, 1999
[4] Darij Grinberg, Über einige Sätze und Aufgaben aus der Dreiecksgeometrie,
http://de.geocities.com/darij_grinberg/Dreigeom/Dreigeom.zip, 2003
[5] Darij Grinberg, Orientierte Winkel modulo 180◦ ,
http://de.geocities.com/darij_grinberg/Dreigeom/OrWinkel.zip, 2004
[6] Titu Andreescu, Mathematical Olympiad Challenges, Birkhäuser Boston, 2001
[7] Titu Andreescu, Mathematical Olympiad Treasures, Birkhäuser Boston, 2003
125
Herunterladen